Corporation.classification Of Corp .syllabus .cases2.docx

  • Uploaded by: Nick Cunanan
  • 0
  • 0
  • October 2019
  • PDF

This document was uploaded by user and they confirmed that they have the permission to share it. If you are author or own the copyright of this book, please report to us by using this DMCA report form. Report DMCA


Overview

Download & View Corporation.classification Of Corp .syllabus .cases2.docx as PDF for free.

More details

  • Words: 47,847
  • Pages: 95
CASE TITLE

National Coal Company vs Collector of Internal Revenue 46 Phil 583 [GR No. L-22619 December 2, 1924]

FACTS

ISSUE

RULING

IN RELATION TO THE STATE: PRIVATE The plaintiff corporation was created on the 10th Whether or not Yes. The plaintiff is a private corporation. The mere fact that the day of March 1917, by Act No. 2705, for the plaintiff is a government happens to the majority stockholder does not make it a purpose of developing the coal industry in the private public corporation. Act 2705, as amended by Act 2822, makes it subject Philippine Islands, in harmony with the general corporation. to all the provisions of the corporation law, in so far as they are not plan of the government to encourage the inconsistent with said act. No provisions of Act 2705 are found to be development of natural resources of the country, inconsistent with the provisions of the corporation law. As a private and to provide facilities therefore. By the said act, corporation, it has no greater rights, powers or privileges than any the company was granted the general powers of a other corporation which might be organized for the same purpose corporation and such other powers as may be under the corporation law, and certainly it was not the intention of the necessary to enable it to prosecute the business legislature to give it a preference or right or privilege over other of developing coal deposits in the Philippine legitimate private corporations in the mining of coal. While it is true that Islands of mining, extracting, transporting, and said proclamation no. 39 withdrew from settlement entry, sale or other selling the coal contained in said deposits. By the disposition of coal-bearing public lands within the province of same law, the government of the Philippine Zamboanga, and the islands of Polillo, it made no provision for the Islands is made the majority stockholder, occupation and operation by the plaintiff, to the exclusion of other evidently in order to ensure proper government persons or corporations who might under proper permission, enter supervision and control and thus to place the upon to operate the coal mines. government in a position to render all possible encouragement, assistance, and help in the prosecution and furtherance of the company’s business. On May 14, 1917, two months after the passage of Act no. 2705, creating the national coal company, the Philippine legislature passed Act 2719, “to provide for the leasing and development of coal lands in the Philippine islands.” On October 18, 1917, upon petition of the national coal company, the governor-general, by proclamation no. 39, withdrew from settlement, entry, sale or other deposition, all coal-bearing public lands within the province of Zamboanga, Department of Mindanao and Sulu, and the island of Polillo, Province of Tayabas. Almost immediately after the issuance of said proclamation the national coal company took possession of the coal lands within the said reservation with an area of about 400 hectares, without any further formality, contract of lease. Of

PHILIPPINE SOCIETY FOR THE PREVENTION OF CRUELTY TO ANIMALS vs. COA. G.R. No. 169752 September 25, 2007

the 30,000 shares of stock issued by the company, the government of the Philippine islands is the owner of 29,809 shares, that is, of 99 1/3 per centum of the whole capital stock. The petitioner was incorporated as a juridical entity over one hundred years ago by virtue of Act No. 1285, enacted on January 19, 1905, by the Philippine Commission. The petitioner, at the time it was created, was composed of animal aficionados and animal propagandists. The objects of the petitioner, as stated in Section 2 of its charter, shall be to enforce laws relating to cruelty inflicted upon animals or the protection of animals in the Philippine Islands, and generally, to do and perform all things which may tend in any way to alleviate the suffering of animals and promote their welfare. At the time of the enactment of Act No. 1285, the original Corporation Law, Act No. 1459, was not yet in existence. Act No. 1285 antedated both the Corporation Law and the constitution of the SEC. For the purpose of enhancing its powers in promoting animal welfare and enforcing laws for the protection of animals, the petitioner was initially imbued under its charter with the power to apprehend violators of animal welfare laws. In addition, the petitioner was to share 1/2 of the fines imposed and collected through its efforts for violations of the laws related thereto. Subsequently, however, the power to make arrests as well as the privilege to retain a portion of the fines collected for violation of animalrelated laws were recalled by virtue of C.A. No. 148. Whereas, the cruel treatment of animals is now an offense against the State, penalized under our statutes, which the Government is duty bound to enforce; When the COA was to perform an audit on them they refuse to do so, by the reason that they are a private entity and not under the said commission. It argued that COA covers only government entities. On the other hand the COA decided that

WON the said petitioner is a private entity.

YES. First, the Court agrees with the petitioner that the “charter test” cannot be applied. Essentially, the “charter test” provides that the test to determine whether a corporation is government owned or controlled, or private in nature is simple. Is it created by its own charter for the exercise of a public function, or by incorporation under the general corporation law? Those with special charters are government corporations subject to its provisions, and its employees are under the jurisdiction of the CSC, and are compulsory members of the GSIS. And since the “charter test” had been introduced by the 1935 Constitution and not earlier, it follows that the test cannot apply to the petitioner, which was incorporated by virtue of Act No. 1285, enacted on January 19, 1905. Settled is the rule that laws in general have no retroactive effect, unless the contrary is provided. All statutes are to be construed as having only a prospective operation, unless the purpose and intention of the legislature to give them a retrospective effect is expressly declared or is necessarily implied from the language used. In case of doubt, the doubt must be resolved against the retrospective effect. Second, a reading of petitioner’s charter shows that it is not subject to control or supervision by any agency of the State, unlike GOCCs. No government representative sits on the board of trustees of the petitioner. Like all private corporations, the successors of its members are determined voluntarily and solely by the petitioner in accordance with its by-laws, and may exercise those powers generally accorded to private corporations, such as the powers to hold property, to sue and be sued, to use a common seal, and so forth. It may adopt by-laws for its internal operations: the petitioner shall be managed or operated by its officers “in accordance with its by-laws in force.” Third. The employees of the petitioner are registered and covered by the SSS at the latter’s initiative, and not through the GSIS, which should be the case if the employees are considered government employees. This is another indication of petitioner’s nature as a private entity. Fourth. The respondents contend that the petitioner is a “body politic” because its primary purpose is to secure the protection and welfare of animals which, in turn, redounds to the public good. This argument, is not tenable. The fact that a certain juridical entity is impressed with public interest does not, by that circumstance alone, make the entity a

it is a government entity. WON the said petitioner is a private entity.

public corporation, inasmuch as a corporation may be private although its charter contains provisions of a public character, incorporated solely for the public good. This class of corporations may be considered quasi-public corporations, which are private corporations that render public service, supply public wants, or pursue other eleemosynary objectives. While purposely organized for the gain or benefit of its members, they are required by law to discharge functions for the public benefit. Examples of these corporations are utility, railroad, warehouse, telegraph, telephone, water supply corporations and transportation companies. It must be stressed that a quasi-public corporation is a species of private corporations, but the qualifying factor is the type of service the former renders to the public: if it performs a public service, then it becomes a quasi-public corporation. Authorities are of the view that the purpose alone of the corporation cannot be taken as a safe guide, for the fact is that almost all corporations are nowadays created to promote the interest, good, or convenience of the public. A bank, for example, is a private corporation; yet, it is created for a public benefit. Private schools and universities are likewise private corporations; and yet, they are rendering public service. Private hospitals and wards are charged with heavy social responsibilities. More so with all common carriers. On the other hand, there may exist a public corporation even if it is endowed with gifts or donations from private individuals. The true criterion, therefore, to determine whether a corporation is public or private is found in the totality of the relation of the corporation to the State. If the corporation is created by the State as the latter’s own agency or instrumentality to help it in carrying out its governmental functions, then that corporation is considered public; otherwise, it is private. Applying the above test, provinces, chartered cities, and barangays can best exemplify public corporations. They are created by the State as its own device and agency for the accomplishment of parts of its own public works. Fifth. The respondents argue that since the charter of the petitioner requires the latter to render periodic reports to the Civil Governor, whose functions have been inherited by the President, the petitioner is, therefore, a government instrumentality. This contention is inconclusive. By virtue of the fiction that all corporations owe their very existence and powers to the State, the reportorial requirement is applicable to all corporations of whatever nature, whether they are public, quasi-public, or private corporations— as creatures of the State, there is a reserved right in the legislature to investigate the activities of a corporation to determine whether it acted

within its powers. In other words, the reportorial requirement is the principal means by which the State may see to it that its creature acted according to the powers and functions conferred upon it.

Feliciano vs. COA (G.R. No. 147402, January 14, 2004

IN RELATION TO THE STATE: QUASI PUBLIC A special audit team from COA Regional office no. Whether or not No. Private corporations may exist only under a general law. If the VIII audited the accounts of LMWD. Subsequently, petitioner LMWD corporation is private, it must necessarily exist under a general law. LMWD received a letter from COA dated July 19, is a private Stated differently, only corporations created under a general law can 1999 requesting payment of auditing fees. As corporation qualify as private corporations under existing laws, that general law is general manager of LMWD, petitioner sent a reply exempt from the the corporation code, except that the cooperative code governs the dated October 12, 1999 informing COA’s regional auditing incorporation of cooperatives. director that the water district could not pay the jurisdiction of auditing fees. Petitioner cited as basis for his COA. Obviously, LWDs are not private corporations because they are not action section 6 and 20 of Presidential Decree no. created under the corporation code. LWDs are registered with the 198 as well as section 18 of RA 6758. The regional Securities and Exchange Commission (SEC). Section 14 of the director referred petitioner to reply o the COA corporation code states that all corporations under this code shall file Chairman on October 18, 1999. On October 19, with the SEC articles of incorporation. LWDs have no articles of 1999, petitioner wrote COA through the Regional incorporation, no incorporators and no stockholders or members. There Director asking for refund of all auditing fees are no stockholders or members to elect the board of directors of LWDs LMWD previously paid to COA. On March 16, as in the case of all corporations registered with the SEC. The local 2000, petitioner received COA Chairman Celso D. mayor or the provincial governor appoints the directors of LWDs for a Gangans resolution dated January 3, 2o00 fixed term of office. This court has ruled that LWDs are not created denying his requests. Petitioner filed a motion for under the corporation code. reconsideration on March 31, 2000, which COA denied on January 30, 2001. The determining factor of COA’s audit jurisdiction is government ownership or control of the corporation. The criterion of ownership and control is more important than the issue of original charter. Certainly, the government owns and controls LWDs. The government organizes LWDs in accordance with a specific law, PD 198. There is no private party involved as co-owner in the creation of and LWD. Just prior to the creation of LWDs, the national or local government owns and controls all their assets. The government controls LWDs because under PD 198 the municipal or city mayor, or the provincial governor, appoints all the board of directors of an LWD for a fixed term of six (6) years. The board of directors of LWDs are not co-owners of the LWDs. LWD have no private stockholders or members. The board of directors and other personnel of LWDs are government employees subject to civil

service laws, anti-graft laws. Section 18 of RA 6758 prohibits COA Personnel from receiving any kind of compensation from any government except compensation paid directly by COA out of its appropriations and contributions. Thus, RA 6758 itself recognizes an exception to the statutory ban by COA personnel receiving compensation from GOCCs.

Liban v. Gordon G.R. 175352 July 15, 2009

Dante V. Liban, together with other petitioners, petitioned in Court to declare Richard J. Gordon as “having forfeited his seat in the Senate.” The petitioners were officers of the Board of Directors of the Quezon City Red Cross Chapter, while respondent is Chairman of the Philippine National Red Cross (PNRC) Board of Governors. During Gordon’s incumbency as a member of the Senate of the Philippines, he was elected Chairman of the PNRC during the February 23, 2006 meeting of the PNRC Board of Governors, in which the petitioners alleged that by accepting the responsibility, Gordon deemed ceased to be a member of the Senate as provided in Sec. 13, Article VI of the Constitution: Sec. 13. No Senator or Member of the House of Representatives may hold any other office or employment in the Government, or any subdivision, agency, or instrumentality thereof, including government-owned or controlled corporations or their subsidiaries, during his term without forfeiting his seat…. Respondent contested that the petitioners’ citation of a constitutional provision had no basis, since PNRC is not a government-owned or controlled corporation. Thus, prohibition under Sec. 13, Art. VI of the Constitution did not apply to his case. Furthermore, service rendered in PNRC is a

By accepting the PNRC Chair, did Gordon forfeit his Senate Seat?

No. The Philippine National Red Cross is a private organization performing public functions. It does not have government assets and does not receive any appropriation from the Philippine Congress. The PNRC is financed primarily by contributions from private individuals and private entities obtained through solicitation campaigns organized by its Board of Governors. Apart from that, PNRC must not only be, but must also be seen to be, autonomous, neutral and independent to be able to conduct its activities in accord to their fundamental principles of humanity, impartiality, neutrality, independence, voluntary service, unity, and universality. Hence, Article VI, Section 13 could not apply to Gordon’s case, in accepting the position in the PNRC. The petition was deemed to have no merit.

volunteer service to which is neither an office nor an employment. DANTE V. LIBAN, et al. v. RICHARD J. GORDON 2011

Respondent filed a motion for partial reconsideration on a Supreme Court decision which ruled that being chairman of the Philippine National Red Cross (PNRC) did not disqualify him from being a Senator, and that the charter creating PNRC is unconstitutional as the PNRC is a private corporation and the Congress is precluded by the Constitution to create such. The Court then ordered the PNRC to incorporate itself with the SEC as a private corporation. Respondent takes exception to the second part of the ruling, which addressed the constitutionality of the statute creating the PNRC as a private corporation. Respondent avers that the issue of constitutionality was only touched upon in the issue of locus standi. It is a rule that the constitutionality will not be touched upon if it is not the lis mota of the case.

Was it proper for the Court to have ruled on the constitutionality of the PNRC statute?

In the case at bar, the constitutionality of the PNRC statute was raised in the issue of standing. As such, the Court should not have declared certain provisions of such as unconstitutional. On the substantive issue, the PNRC is sui generis. It is unlike the private corporations that the Constitution wants to prevent Congress from creating. First, the PNRC is not organized for profit. It is an organization dedicated to assist victims of war and administer relief to those who have been devastated by calamities, among others. It is entirely devoted to public service. It is not covered by the prohibition since the Constitution aims to eliminate abuse by the Congress, which tend to favor personal gain. Secondly, the PNRC was created in order to participate in the mitigation of the effects of war, as embodied in the Geneva Convention. The creation of the PNRC is compliance with international treaty obligations. Lastly, the PNRC is a National Society, an auxiliary of the government. It is not like government instrumentalities and GOCC. The PNRC is regulated directly by international humanitarian law, as opposed to local law regulating the other mentioned entities. As such, it was improper for the Court to have declared certain portions of the PNRC statute as unconstitutional. However, it is the stand of Justice Carpio that there is no mandate for the Government to create a National Society to this effect. He also raises the fact that the PNRC is not sui generis in being a private corporation organized for public needs. Justice Abad is of the opinion that the PNRC is neither private or governmental, hence it was within the power of Congress to create. It has been consistently held in Jurisprudence that the Court should exercise judicial restraint when it comes to issues of constitutionality where it is not the lis mota of the case.

REGISTER OF DEEDS vs UNG SIU SI TEMPLE GR. No. L-6776 May 21,1955

EXPLOITATION OF NATURAL RESOURCES A Filipino citizen executed a deed of donation in Sec. 5, Art. 13 of the Constitution provides that save in cases of favor of the Ung Siu Si Temple, an unregistered Whether a deed of hereditary succession, no private agricultural land shall be religious organization that operated through three donation of a transferred or assigned except to individuals, corporations, or trustees all of Chinese nationality. The Register of parcel of land associations qualified to hold lands of the public domain in the Deeds refused to record the deed of donation executed in favor Philippines. The Constitution does not make any exception in executed in due form arguing that the Consitution of a religious favor of religious associations.

provides that acquisition of land is limited to Filipino citizens, or to corporations or associations at least 60% of which is owned by such citizens.

organization whose founder, trustees and administrator are Chinese citizens should be registered or not.

The fact that appellant has no capital stock does not exempt it from the Constitutional inhibition, since its member are of foreign nationality. The purpose of the 60% requirement is to ensure that corporations or associations allowed to acquire agricultural lands or to exploit natural resources shall be controlled by Filipinos; and the spirit of the Constitution demands that in the absence of capital stock, controlling membership should be composed of Filipino citizens. As to the complaint that the disqualification under Art. 13 of the Constitution violated the freedom of religion, the Court was not convinced that land tenure is indispensable to the free exercise and enjoyment of religious profession or worship.

People v. Quasha (1953) G.R. No. L-6055 June 12, 1953

PUBLIC UTILITIES For a corporation William H. Quasha to be entitled to ▪ a member of the Philippine bar, operate a public committed a crime of falsification of a utility is it public and commercial document for necessary that it causing it to appear that Arsenio Baylon, a Filipino citizen, had subscribed be organized with 60 per cent of its to and was the owner of 60.005 % of capital owned by the subscribed capital stock of Pacific Filipinos from the Airways Corp. (Pacific) when in reality start? the money paid belongs to an American citizen whose name did not appear in the article of incorporation, o to circumvent the constitutional mandate that no corp. shall be authorize to operate as a public utility in the Philippines unless 60% of its capital stock is owned by Filipinos. ▪ Found guilty after trial and sentenced to a term of imprisonment and a fine ● Quasha appealed to this Court ● Primary purpose: to carry on the business of a common carrier by air, land or water ● Baylon did not have the controlling vote because of the difference in voting power



No. For a corporation to be entitled to operate a public utility it is not necessary that it be organized with 60 per cent of its capital owned by Filipinos from the start. A corporation formed with capital that is entirely alien may subsequently change the nationality of its capital through transfer of shares to Filipino citizens. Conversely, a corporation originally formed with Filipino capital may subsequently change the national status of said capital through transfer of shares to foreigners. What need is there then for a corporation that intends to operate a public utility to have, at the time of its formation, 60 per cent of its capital owned by Filipinos alone? That condition may anytime be attained thru the necessary transfer of stocks. The moment for determining whether a corporation is entitled to operate as a public utility is when it applies for a franchise, certificate, or any other form of authorization for that purpose. And that can be done after the corporation has already come into being and not while it is still being formed. And at that moment, the corporation must show that it has complied not only with the requirement of the Constitution as to the nationality of its capital, but also with the requirements of the Civil Aviation Law if it is a common carrier by air, the Revised Administrative Code if it is a common carrier by water, and the Public Service Law if it is a common carrier by land or other kind of public service.

between the preferred shares and the common shares

FILIPINAS COMPANIA DE SEGUROS vs. CHRISTERN HUENEFELD and CO., INC. 89 Phil 54

Roman Catholic Apostolic Administrator of Davao, Inc. v. The Land Registration Commission and the Register of Deeds of Davao

On October 1, 1941, the respondent corporation, Christern Huenefeld and Co., Inc., after payment of corresponding premium, obtained from the petitioner, Filipinas Cia de Seguros fire policy covering merchandise contained in a building located at Binondo, Manila. On February 27, 1942 or during the Japanese military occupation, the building and insured merchandise were burned. In due time, the respondent submitted to the petitioner its claim under the policy. The petitioner refused to pay the claim on the ground that the policy in favor of the respondent that ceased to be a force on the date the United States declared war against Germany, the respondent corporation (through organized under and by virtue of the laws of Philippines) being controlled by German subjects and the petitioner being a company under American jurisdiction when said policy was issued on October 1, 1941. The theory of the petitioner is that the insured merchandise was burned after the policy issued in 1941 had ceased to be effective because the outbreak of the war between United States and Germany on December 10, 1941, and that the payment made by the petitioner to the respondent corporation during the Japanese military occupation was under pressure.

WARTIME-TEST Whether or not the respondent corporation is a corporation of public enemy.

CORPORATION SOLE On October 4, 1954, Mateo L. Rodis, a Filipino Whether or not citizen and resident of the City of Davao, executed the Universal a deed of sale of a parcel of land located in the Roman Catholic same city covered by Transfer Certificate No. Apostolic Church 2263, in favor of the Roman Catholic Apostolic in the Philippines, Administrator of Davao Inc.,(RCADI) is corporation or better still, the sole organized and existing in accordance with corporation sole Philippine Laws, with Msgr. Clovis Thibault, a named the Roman

Since the majority of stockholders of the respondent corporation were German subjects, the respondent became an enemy of the state upon the outbreak of the war between US and Germany. The English and American cases relied upon by the Court of Appeals lost in force upon the latest decision of the Supreme Court of US in which the control test has adopted. Since World War I, the determination of enemy nationality of corporations has been discussed in many countries, belligerent and neutral. A corporation was subject to enemy legislation when it was controlled by enemies, namely managed under the influence of individuals or corporations themselves considered as enemies… The Philippine Insurance Law (Act No 2427, as amended), in Section 8, provides that “anyone except a public enemy may be insured”. It stands to reason that an insurance policy ceases to be allowable as soon as an insured becomes a public enemy. The respondent having an enemy corporation on December 10, 1941, the insurance policy issued in its favor on October 1, 1941, by the petitioner had ceased to be valid and enforceable, and since the insured good were burned during the war, the respondent was not entitled to any indemnity under said policy from the petitioner. However, elementary rule of justice (in the absence of specific provisions in the Insurance Law) require that the premium paid by the respondent for the period covered by its policy from December 11, 1941, should be returned by the petitioner.

RCADI is qualified. While it is true and We have to concede that in the profession of their faith, the Roman Pontiff is the supreme head; that in the religious matters, in the exercise of their belief, the Catholic congregation of the faithful throughout the world seeks the guidance and direction of their Spiritual Father in the Vatican, yet it cannot be said that there is a merger of personalities resultant therein. Neither can it be said that the political and civil rights of the faithful, inherent or acquired under the

City, G.R. No. L8451, December 20,1957

Canadian citizen, as actual incumbent. Registry of Deeds Davao (RD) required RCADI to submit affidavit declaring that 60% of its members were Filipino Citizens. As the RD entertained some doubts as to the registerability of the deed of sale, the matter was referred to the Land Registration Commissioner (LRC) en consulta for resolution. LRC hold that pursuant to provisions of sections 1 and 5 of Article XII of the Philippine Constitution, RCADI is not qualified to acquire land in the Philippines in the absence of proof that at leat 60% of the capital, properties or assets of the RCADI is actually owned or controlled by Filipino citizens. LRC also denied the registration of the Deed of Sale in the absence of proof of compliance with such requisite. RCADI’s Motion for Reconsideration was denied. Aggrieved, the latter filed a petition for mandamus.

Catholic Apostolic Administrator of Davao, Inc., is qualified to acquire private agricultural lands in the Philippines pursuant to the provisions of Article XIII of the Constitution.

laws of their country, are affected by that relationship with the Pope. The fact that the Roman Catholic Church in almost every country springs from that society that saw its beginning in Europe and the fact that the clergy of this faith derive their authorities and receive orders from the Holy See do not give or bestow the citizenship of the Pope upon these branches. Citizenship is a political right which cannot be acquired by a sort of “radiation”. We have to realize that although there is a fraternity among all the catholic countries and the dioceses therein all over the globe, the universality that the word “catholic” implies, merely characterize their faith, a uniformity in the practice and the interpretation of their dogma and in the exercise of their belief, but certainly they are separate and independent from one another in jurisdiction, governed by different laws under which they are incorporated, and entirely independent on the others in the management and ownership of their temporalities. To allow theory that the Roman Catholic Churches all over the world follow the citizenship of their Supreme Head, the Pontifical Father, would lead to the absurdity of finding the citizens of a country who embrace the Catholic faith and become members of that religious society, likewise citizens of the Vatican or of Italy. And this is more so if We consider that the Pope himself may be an Italian or national of any other country of the world. The same thing be said with regard to the nationality or citizenship of the corporation sole created under the laws of the Philippines, which is not altered by the change of citizenship of the incumbent bishops or head of said corporation sole. We must therefore, declare that although a branch of the Universal Roman Catholic Apostolic Church, every Roman Catholic Church in different countries, if it exercises its mission and is lawfully incorporated in accordance with the laws of the country where it is located, is considered an entity or person with all the rights and privileges granted to such artificial being under the laws of that country, separate and distinct from the personality of the Roman Pontiff or the Holy See, without prejudice to its religious relations with the latter which are governed by the Canon Law or their rules and regulations. It has been shown before that: (1) the corporation sole, unlike the ordinary corporations which are formed by no less than 5 incorporators, is composed of only one persons, usually the head or bishop of the diocese, a unit which is not subject to expansion for the purpose of determining any percentage whatsoever; (2) the corporation sole is only the administrator and not the owner of the temporalities located in the territory comprised by said corporation sole; (3) such temporalities are administered for and on behalf of the faithful residing in the diocese

or territory of the corporation sole; and (4) the latter, as such, has no nationality and the citizenship of the incumbent Ordinary has nothing to do with the operation, management or administration of the corporation sole, nor effects the citizenship of the faithful connected with their respective dioceses or corporation sole. In view of these peculiarities of the corporation sole, it would seem obvious that when the specific provision of the Constitution invoked by respondent Commissioner (section 1, Art. XIII), was under consideration, the framers of the same did not have in mind or overlooked this particular form of corporation. If this were so, as the facts and circumstances already indicated tend to prove it to be so, then the inescapable conclusion would be that this requirement of at least 60 per cent of Filipino capital was never intended to apply to corporations sole, and the existence or not a vested right becomes unquestionably immaterial.

Republic v. Villanueva

AS TO EXISTENCE OF SHARES: NON STOCK Lots Nos. 568 and 569, located at Barrio Dampol, Whether or not Section 11, Article XIV of the Constitution stated that "no private Plaridel, Bulacan, with an area of 313 square the trial court corporation or association may hold alienable lands of the public meters and an assessed value of P1,350 were erred in ordering domain except by lease not to exceed one thousand hectares in area". acquired by INC on January 9, 1953 from Andres the registration of Perez in exchange for a lot with an area of 247 two lots. INC, as a corporation sole or a juridical person, is disqualified to acquire square meters owned by the said church. or hold alienable lands of the public domain, like the two lots in question, because of the constitutional prohibition already mentioned The said lots were already possessed by Perez in and because the said church is not entitled to avail itself of the benefits 1933. They are not included in any military of section 48(b) which applies only to Filipino citizens or natural reservation. They are inside an area which was persons. A corporation sole has no nationality (Roman Catholic certified as alienable or disposable by the Bureau Apostolic Adm. of Davao, Inc. vs. Land Registration Commission, 102 of Forestry in 1927. The lots are planted to santol Phil. 596. See Register of Deeds vs. Ung Siu Si Temple, 97 Phil. 58 and and mango trees and banana plants. A chapel sec. 49 of the Public Land Law). exists on the said land. The land had been declared for realty tax purposes. Realty taxes had The contention in the comments of the INC that the two lots are private been paid therefor. lands, following the rule laid down in Susi vs. Razon and Director of On September 13, 1977, INC filed with the CFI Bulacan an application for the registration of the two lots. It alleged that it and its predecessors-ininterest had possessed the land for more than thirty years. The trial court ordered the registration of the two lots.

Lands, 48 Phil. 424, is not correct. What was considered private land in the Susi case was a parcel of land possessed by a Filipino citizen since time immemorial, as in Cariño vs. Insular Government, 212 U.S. 449, 53 L. ed. 594, 41 Phil. 935 and 7 Phil. 132. The lots sought to be registered in this case do not fall within that category. They are still public lands. A land registration proceeding under section 48(b) "presupposes that the land is public" (Mindanao vs. Director of Lands, L-19535, July 10, 1967, 20 SCRA 641, 644).

As held in Oh Cho vs. Director of Lands, 75 Phil. 890, "all lands that were not acquired from the Government, either by purchase or by grant, belong to the public domain. An exception to the rule would be any land that should have been in the possession of an occupant and of his predecessors-in-interest since time immemorial, for such possession would justify the presumption that the land had never been part of the public domain or that it had been a private property even before the Spanish conquest." In Uy Un vs. Perez, 71 Phil. 508, it was noted that the right of an occupant of public agricultural land to obtain a confirmation of his title under section 48(b) of the Public Land Law is a "derecho dominical incoativo" and that before the issuance of the certificate of title the occupant is not in the juridical sense the true owner of the land since it still pertains to the State. The lower court's judgment is reversed and set aside. The application for registration of INC is dismissed with costs against said applicant. Collector of Internal Revenue vs. Club Filipino Inc. De Cebu

As found by the Court of Tax Appeals, the "Club Filipino, Inc. de Cebu," (Club, for short), is a civic corporation organized under the laws of the Philippines with an original authorized capital stock of P22,000. 00, which was subsequently increased to P200,000. 00. The Club owns and operates a club house, a bowling alley, a golf course (on a lot leased from the government), and a bar-restaurant where it sells wines and liquors, soft drinks, meals and short orders to its members and their guests. The bar-restaurant was a necessary incident to the operation of the club and its golf-course. The club is operated mainly with funds derived from membership fees and dues. In 1951. as a result of a capital surplus, arising from the re-valuation of its real properties, the value or price of which increased, the Club declared stock dividends; but no actual cash dividends were distributed to the stockholders. In 1952, a BIR agent discovered that the Club has never paid percentage tax on the gross receipts of its bar and restaurant,

Whether or not the Club is a stock-corporation? If so, can it be subject to tax

For a stock corporation to exist, two requisites must be complied with, to wit: (1) a capital stock divided into shares and (2) an authority to distribute to the holders of such shares, dividends or allotments of the surplus profits on the basis of the shares held (sec. 3, Act No. 1459). In the case at bar, nowhere in its articles of incorporation or by-laws could be found an authority for the distribution of its dividends or surplus profits. Strictly speaking, it cannot, therefore, be considered a stock corporation, within the contemplation of the corporation law. The bar and restaurant are necessary adjuncts of the Club to foster its purposes and the profits derived therefrom are necessarily incidental to the primary object of developing and cultivating sports for the healthful recreation and entertainment of the stockholders and members. Having arrived at the conclusion that respondent Club is not engaged in the business as an operator of a bar and restaurant, and therefore, not liable for fixed and percentage taxes, it follows that it is not liable for any penalty, much less of a compromise penalty.

although it secured B-4, B-9(a) and B-7 licenses. In a letter dated December 22, 1852, the Collector of Internal Revenue assessed against and demanded from the Club CLOSE CORPORATION: NO NECESSITY OF BOARD DULAY ENTERPRISES, INC. vs. COURT OF APPEALS FACTS

Dulay Enterprises, Inc., a domestic corporation with the following as members of its Board of Directors: Manuel R. Dulay designated as president, treasurer and general manager; Atty. Virgilio E. Dulay designated as vice-president; Linda E. Dulay; Celia Dulay-Mendoza and Atty. Plaridel C. Jose designated as secretary, owned a property known as Dulay Apartment located at Seventh Street (now Buendia Extension) and F. B. Harrison Street, Pasay City. The corporation through its president, Manuel Dulay, obtained various loans for the construction of its hotel project, Dulay Continental Hotel (now Frederick Hotel). It even had to borrow money from Virgilio Dulay to be able to continue the hotel project. As a result of said loan, Virgilio Dulay occupied one of the unit apartments of the subject property since 1973 while at the same time managing the Dulay Apartment as his shareholdings in the corporation was subsequently increased by his father. Manuel Dulay by virtue of Board Resolution 18 of the corporation sold the subject property to spouses Maria Theresa and Castrense Veloso. Subsequently, Manuel Dulay and the spouses Veloso executed a Memorandum to the Deed of Absolute Sale, giving Manuel Dulay within 2 years to repurchase the subject property which was, however, not annotated. Maria Veloso, without the knowledge of Manuel Dulay, mortgaged the subject property to Manuel A. Torres. Upon the failure of Maria Veloso to pay Torres, the subject property was sold to Torres as the highest bidder in an extrajudicial foreclosure sale

Whether the sale of the subject property between spouses Veloso and Manuel Dulay has no binding effect on the corporation as Board Resolution 18 which authorized the sale of the subject property was resolved without the approval of all the members of the board of directors and said Board Resolution was prepared by a person not designated by the corporation to be its secretary.

Section 101 of the Corporation Code of the Philippines provides that "When board meeting is unnecessary or improperly held. Unless the bylaws provide otherwise, any action by the directors of a close corporation without a meeting shall nevertheless be deemed valid if: (1) Before or after such action is taken, written consent thereto is signed by all the directors; or (2) All the stockholders have actual or implied knowledge of the action and make no prompt objection thereto in writing; or (3) The directors are accustomed to take informal action with the express or implied acquiesce of all the stockholders; or (4) All the directors have express or implied knowledge of the action in question and none of them makes prompt objection thereto in writing. If a directors' meeting is held without proper call or notice, an action taken therein within the corporate powers is deemed ratified by a director who failed to attend, unless he promptly files his written objection with the secretary of the corporation after having knowledge thereof. " Herein, the corporation is classified as a close corporation and consequently a board resolution authorizing the sale or mortgage of the subject property is not necessary to bind the corporation for the action of its president. At any rate, a corporate action taken at a board meeting without proper call or notice in a close corporation is deemed ratified by the absent director unless the latter promptly files his written objection with the secretary of the corporation after having knowledge of the meeting which, in this case, Virgilio Dulay failed to do. The corporation's claim that the sale of the subject property by its president, Manuel Dulay, to spouses Veloso is null and void as the alleged Board Resolution 18 was passed without the knowledge and consent of the other members of the board of directors cannot be sustained. Virgilio E. Dulay's protestations of complete innocence to the effect that he never participated nor was even aware of any meeting or resolution authorizing the mortgage or sale of the subject premises is difficult to believe. On the contrary, he is very much privy to the transactions involved. To begin with, he is an incorporator and one of the board of directors designated at the time of the organization of

as evidenced by the Certificate of Sheriff's Sale. Maria Veloso executed a Deed of Absolute Assignment of the Right to Redeem in favor of Manuel Dulay assigning her right to repurchase the subject property from Torres as a result of the extrajudicial sale. As neither Maria Veloso nor her assignee Manuel Dulay was able to redeem the subject property within the one year statutory period for redemption, Torres filed an Affidavit of Consolidation of Ownership with the Registry of Deeds of Pasay City. Torres filed a petition for the issuance of a writ of possession against spouses Veloso and Manuel Dulay in LRC. However, when Virgilio Dulay appeared in court to intervene in said case alleging that Manuel Dulay was never authorized by the corporation to sell or mortgage the subject property, the trial court ordered Torres to implead the corporation as an indispensable party but the latter moved for the dismissal of his petition which was granted. Torres and Edgardo Pabalan, real estate administrator of Torres, filed an action against the corporation, Virgilio Dulay and Nepomuceno Redovan, a tenant of Dulay Apartment for the recovery of possession, sum of money and damages with preliminary injunction. The corporation filed an action against spouses Veloso and Torres for the cancellation of the Certificate of Sheriff's Sale. Pabalan and Torres filed an action against spouses Florentino and Elvira Manalastas, a tenant of Dulay Apartment, with the corporation as intervenor for ejectment Metropolitan Trial Court of Pasay City which rendered a decision in favor of Pabalan, et al. , ordering the spouses Manalastas and all persons claiming possession under them to vacate the premises; and to pay the rents until they shall have vacated the premises with interest at the legal rate; and to pay attorney's fees as other expenses of litigation and for them to pay the costs of the suit. Thereafter, the corporation and Virgilio Dulay filed an action against the

Manuel R. Dulay Enterprises, Inc. In ordinary parlance, the said entity is loosely referred to as a "family corporation. " The nomenclature, if imprecise, however, fairly reflects the cohesiveness of a group and the parochial instincts of the individual members of such an aggrupation of which Manuel R. Dulay Enterprises, Inc. is typical: four-fifths of its incorporators being close relatives namely, 3 children and their father whose name identifies their corporation. Besides, the fact that Virgilio Dulay on 24 June 1975 executed an affidavit that he was a signatory witness to the execution of the post-dated Deed of Absolute Sale of the subject property in favor.

presiding judge of the Metropolitan Trial Court of Pasay City, Pabalan and Torres for the annulment of said decision with the Regional Trial Court of Pasay. Thereafter, the 3 cases were jointly tried and the trial court rendered a decision in favor of Pabalan and Torres. The corporation, et al. filed the petition for review on certiorari. During the pendency of the petition, Torres died and named Torres-Pabalan Realty & Development Corporation as his heir in his holographic will.

FINANCING CORPORATION OF THE PHILIPPINES v. TEODORO G.R. No. L-4900; August 31, 1953; Montemayor, J. Digest prepared by Jackie Canlas

CLOSE CORPORATION: WITHDRAWL AND DISSOLUTION Lizares et al., in their own behalf and in WON the YES. behalf of the other minority stockholders of appointment of PETITION DENIED. WRIT OF PRELIMINARY INJUCTION the Financing Corporation of the Philippines a receiver by DISSOLVED. sued the Corporation and J Amado Araneta, the lower court its President and GM, alleging gross was proper? ● GENERAL RULE: Minority shareholders of a corporation cannot sue mismanagement and fraudulent conduct of and demand dissolution in a private suit. The action should be the corporate affairs by Araneta and asking brought by the Government through its legal officer, via a quo that (1) the corporation be dissolved, (2) warranto proceeding. Araneta be declared personally accountable o EXCEPTION: cases wherein the intervention of the State for the unauthorized and fraudulent cannot be obtained because the complaint is a matter strictly disbursements of the corporate assets and between the shareholders and the corporation and does not violations of the Corporation Law and the byinvolve issues which involve acts/omissions warranting a quo laws of the Corporation, and (3) the best warranto. means to protect and preserve the assets of o When such action is brought, the trial court has jurisdiction Corporation is the appointment of a receiver. and has discretion to grant the prayer or not. Having such ● Among the allegations specified in the jurisdiction, the appointment of a receiver pendente lite is left complaint were: to the sound discretion of the trial court. 1. wrongful and unauthorized diversion o The appointment of a receiver upon petition by the minority from corporate purposes and use for shareholders is a power that must be exercised with great personal benefit; caution, and should be exercised when necessary to protect 2. unauthorized and profitless pledging of their rights, especially when they cannot obtain redress securities owned by Corporation to through or within the corporation. secure obligations amounting to P588,645.34 of another corporation controlled by Araneta; 3. unauthorized and profitless using of the name of the Corporation in the shipping of sugar belonging to other corporations





controlled by Araneta to the benefit of said corporations in the amount of at least P104,343.36; 4. refusal by Araneta to endorse to the Corporation shares of stock and other securities belonging to it but which are still in his name; 5. negligent failure to endorse other shares of stock belonging to Corporation but still in the names of the respective vendors; and 6. illegal and unauthorized transfer and deposit in the US of 6,426,281 shares of the Atok-Big Wedge Mining Company; 7. refusal to allow minority stockholders to examine the books and records of the Corporation; 8. failure to call and hold stockholders' and directors' meetings; 9. virtual disregard and ignoring of the board of directors who has been and is conducting the affairs of the Corporation under his absolute control and for his personal benefit and for the benefit of the corporations controlled by him; and 10. irregularity in the keeping and errors and omissions in the books and failure of the same to reflect the real and actual transactions of the Corporation. ● Judge Teodoro granted petition for appointment of a receiver (Yulo). The Corporation filed the present petition for certiorari with preliminary injunction to revoke and set aside the order appointing a receiver, alleging that: 1. The appointment of a receiver was merely an auxiliary remedy; 2. The principal remedy sought by Lizares et al. was dissolution; 3. A suit for the dissolution of a corporation can be brought and maintained only by the State

through its legal counsel, and Lizares et al., much less the minority shareholders, have no right or personality to maintain the action for dissolution. 4. Since the action cannot be maintained legally by Lizares et al., the auxiliary remedy of appointment of a receiver has no basis. ● The lower court granted the writ of preliminary injunction upon the filing of a bond by the Corporation.

CASE TITLE

FACTS

ISSUE

RULING

Organizing the Corporation Promoters (Section 2 [r] of the Revised Securities Act [BP 178]) (Sections 60 and 61 of the Corporation Code) CAGAYAN FISHING DEVELOPMENT CO., INC., vs.TEODORO SANDIKO G.R. No. 43350 December 23, 1937 Corporation, Incorporation, Promoters of Corporation

Manuel Tabora is the registered owner of four parcels of land and he wanted to build a Fishery. He loaned from PNB P8, 000 and to guarantee the payment of the loan, he mortgaged the said parcels of land. Three subsequent mortgages were executed in favor of the same bank and to Severina Buzon, whom Tabora is indebted to. Tabora sold the four parcels of land to the plaintiff company, said to be under process of incorporation, in consideration of one peso (P1) subject to the mortgages in favor of PNB and Severina Buzon and, to the condition that the certificate of title to said lands shall not be transferred to the name of the plaintiff company until the latter has fully and completely paid Tabora’s indebtedness to PNB. The articles of incorporation were filed and the company sold the parcels of land to Sandiko on the reciprocal obligation that Sandiko will shoulder the three mortgages. A deed of sale executed before a notary public by the terms of which the plaintiff sold, ceded and transferred to the defendant all its rights, titles and interest in and to the four parcels of land. He executed a promissory note that he shall be 25,300 after a year with interest and on the promissory notes, the parcels were mortgage as security. A promissory note for P25,300 was drawn by the defendant in favor of the plaintiff, payable after one year from the date thereof. Further, a deed of mortgage executed before a notary public in accordance with which the four parcels of land were given as security for the payment of the said promissory note. All these three instruments were dated

1. Whether Cagayan Fishing Dev’t. has juridical capacity to enter into the contract. 2. Can promoters of a corporation act as agents of a corporation?

1. The transfer made by Tabora to the Cagayan Fishing Development Co., Inc., plaintiff herein, was effected on May 31, 1930 and the actual incorporation of said company was effected later on October 22, 1930. In other words, the transfer was made almost five months before the incorporation of the company. A duly organized corporation has the power to purchase and hold such real property as the purposes for which such corporation was formed may permit and for this purpose may enter into such contracts as may be necessary. But before a corporation may be said to be lawfully organized, many things have to be done. Among other things, the law requires the filing of articles of incorporation. Although there is a presumption that all the requirements of law have been complied with, in the case before us it cannot be denied that the plaintiff was not yet incorporated when it entered into the contract of sale. The contract itself referred to the plaintiff as “una sociedad en vias de incorporacion.” It was not even a de facto corporation at the time. Not being in legal existence then, it did not possess juridical capacity to enter into the contract. “Corporations are creatures of the law, and can only come into existence in the manner prescribed by law. As has already been stated, general laws authorizing the formation of corporations are general offers to any persons who may bring themselves within their provisions; and if conditions precedent are prescribed in the statute,

February 15, 1932. Sandiko failed to pay, thus the action for payment. The lower court held that deed of sale was invalid. The corporation filed a motion for reconsideration.

or certain acts are required to be done, they are terms of the offer, and must be complied with substantially before legal corporate existence can be acquired.” “That a corporation should have a full and complete organization and existence as an entity before it can enter into any kind of a contract or transact any business, would seem to be selfevident. A corporation, until organized, has no being, franchises or faculties. Nor do those engaged in bringing it into being have any power to bind it by contract, unless so authorized by the charter. Until organized as authorized by the charter there is not a corporation, nor does it possess franchises or faculties for it or others to exercise, until it acquires a complete existence.” 2. The contract here was entered into not only between Manuel Tabora and a non-existent corporation but between Manuel Tabora as owner of four parcels of land on the one hand and the same Manuel Tabora, his wife and others, as mere promoters of a corporation on the other hand. For reasons that are self-evident, these promoters could not have acted as agents for a projected corporation since that which had no legal existence could have no agent. A corporation, until organized, has no life and therefore no faculties. It is, as it were, a child in ventre sa mere. This is not saying that under no circumstances may the acts of promoters of a corporation be ratified by the corporation if and when subsequently organized. There are, of course, exceptions, but under the peculiar facts and circumstances of the present case we decline to extend the doctrine of ratification which would result in the commission of injustice or fraud to the candid and unwary.

The transfer by Manuel Tabora to the Cagayan Fishing Development Company, Inc. was null because at the time it was effected the corporation was non-existent, we deem it unnecessary to discuss this point. 25 SCRA 285 - RIZAL LIGHT & ICE CO., INC. vs. PSC and Morong Elec. Co. (1968)

1. 2.

3. 4.

Morong Electric applied for a CPCN with the PSC to provide for electric service in Morong, Rizal. In May 1962, Morong Electric was granted a franchise. PSC found that “Morong Electric is a corporation duly organized and existing under the laws of the Philippines, the stockholders of which are Filipino citizens, that it is financially capable of operating an electric light, heat and power service, and that at the time the decision was rendered there was absence of electric service in Morong, Rizal” It was only on October 17 of the same year that the SEC issued Morong Electric’s certificate of incorporation. Rizal Light, a prior operator, contends that Morong should not have been granted the CPCN because: o it did not have a corporate personality at the time it was granted a franchise and when it applied for said certificate o it is not financially capable of undertaking an electric service o petitioner was rendering efficient service before its electric plant was burned, and therefore, being a prior operator its investment should be protected and no new party should be granted a franchise and certificate of public convenience and necessity to operate an electric service in the same locality.

WON the franchise granted to Morong Electric is valid despite the fact that such was granted before Morong Electric’s certificate of incorporation was issued by the SEC –

Petitioner: until a corporation has come into being, in this jurisdiction, by the issuance of a certificate of incorporation by the SEC, it cannot enter into any contract as a corporation. The franchise was granted to Morong Electric when it was not yet in esse is null and void SC: Petitioner's contention that Morong Electric did not yet have a legal personality when a municipal franchise was granted to it is correct. The juridical personality and legal existence of Morong Electric began only when its certificate of incorporation was issued by the SEC. o Before that date, or pending the issuance of said certificate of incorporation, the incorporators cannot be considered as de facto corporation, but the fact that Morong Electric had no corporate existence on the day the franchise was granted in its name does not render the franchise invalid, because later Morong Electric obtained its certificate of incorporation and then accepted the franchise in accordance with tshe terms and conditions thereof. This view is sustained by eminent American authorities (see doctrine) Thus, the incorporation of Morong Electric on October 17, 1962 and its acceptance of the franchise as shown by its action in prosecuting the application filed with the Commission for the approval of said franchise 1) perfected a contract between the respondent municipality and Morong Electric and 2) cured the deficiency pointed out by the petitioner in the application of Morong EIectric. The conclusion regarding the validity of the

franchise granted to Morong Electric is not incompatible with the holding of this Court in

Cagayan Fishing Development Co., Inc. vs. Teodoro Sandiko wherein Court held that a

corporation should have a full and complete organization and existence as an entity before it can enter into any kind of a contract or transact any business. o this Court did not say in that case that the rule is absolute or that under no circumstances may the acts of promoters of a corporation be ratified or accepted by the corporation if and when subsequently organized o there are exceptions such as the fact that American courts generally hold that a contract made by the promoters of a corporation on its behalf may be adopted, accepted or ratified by the corporation when organized. PSC decision affirmed. McQuillin: “The fact that a company is not completely incorporated at the time the grant is made to it by a municipality to use the streets does not, in most jurisdictions, affect the validity of the grant. But such grant cannot take effect until the corporation is organized…” Fletcher: “While a franchise cannot take effect until the grantee corporation is organized, the franchise may, nevertheless, be applied for before the company is fully organized. A grant of a street franchise is valid although the corporation is not created until afterwards.” Thompson (explains reason for these rules^): “…an ordinance granting a privilege to a corporation is not void because the beneficiary of the ordinance is not fully organized at the time of

the introduction of the ordinance. It is enough that organization is complete prior to the passage and acceptance of the ordinance. The reason is that a privilege of this character is a mere license to the corporation until it accepts the grant and complies with its terms and conditions.” xxx [IMPT] The ruling that a corporation should have a full and complete organization and existence as an entity before it can enter into any kind of a contract or transact any business is NOT ABSOLUTE. Under American jurisprudence, a contract made by the promoters of a corporation on its behalf may be adopted, accepted or ratified by the corporation when organized.

FERMIN CARAM, JR. and ROSE DE CARAM v. CA and ALBERTO V. ARELLANO 151 SCRA 372 (June 30, 1987) CRUZ, J.

1. 2.

3. 4.

5. 6.

The services of Barretto was requested to initiate the incorporation of Filipinas Orient Airways (FOA). Barretto was referred to as the “moving spirit” of said corporation because it was through his effort that it was created. Before FOA’s creation though, Barretto contracted with a third party, Alberto Arellano, for the latter to prepare a project study for the feasibility of creating a corporation like FOA. The project study was then presented to the would-be incorporators and investors. On the basis of said project study, Fermin Caram, Jr. and Rosa Caram agreed to be incorporators of FOA. Later however, Arellano filed a collection suit against FOA, Barretto, and the Carams. Arellano claims that he was not paid for his work on the project study. Lower Court: Orders the Carams to jointly and severally pay Arellano P50,000.00 for the preparation of the project study and his technical

Whether or not petitioners themselves are also personally liable for such expenses and, if so, to what extent? NO. The petitioners did not contract the services of Arellano. It was only the results of such services that Barretto and Garcia presented to them and which persuaded them to invest in the proposed airline.

GRANTED. Petitioners are not liable. The petitioners were not really involved in the initial steps that finally led to the incorporation of FAO, which were being directed by Barretto as the main promoter. It was he who was putting all the pieces together. The airline was eventually organized on the basis of the project study with the petitioners as major stockholders and, together with Barretto and Garcia, as principal officers. The petitioners were merely among the financiers whose interest was to be invited and who were in fact persuaded, on the strength of the project study, to invest in the proposed airline. There was no showing that FAO was a fictitious corporation and did not have a separate juridical personality, to justify making the petitioners, as principal stockholders thereof, responsible for its obligations. As a bona fide corporation, FAO should alone be liable for its corporate acts as duly authorized by its officers and directors.

7.

8.

services that led to the organization of the defendant corporation, plus P10,000.00 attorney’s fees - It was upon the request of Barretto and Garcia that Arellano handled the preparation of the project study which project study was presented to Caram so the latter was convinced to invest in the proposed airlines. - The project study was revised for purposes of presentation to financiers and the banks. It was on the basis of this study that defendant corporation was actually organized and rendered operational. - Garcia and Caram, and Barretto became members of the Board and/or officers of defendant corporation - All the other defendants who were involved in the preparatory stages of the incorporation must be liable The petitioners claim that this order has no support in fact and law because they had no contract whatsoever with the private respondent regarding the above-mentioned services. Their position is that as mere subsequent investors in the corporation that was later created, they should not be held solidarily liable with FOA, a separate juridical entity, and with Barretto and Garcia (their co-defendants in the lower court) who were the ones who requested the said services from Arellano.

The petition is rather hazy and seems to be flawed by an ambiguous ambivalence. It is unnecessary to examine at this time the rules on solidary obligations, which the parties-needlessly, as it turns out have belabored unto death.

DOCTRINE: Corporation Law; A bona fide corporation should alone be liable for its corporate acts duly authorized by its officers and directors. Petitioners cannot be held personally liable for the compensation claimed by private respondent for services performed by him in the organization of the corporation since petitioners did not contract such services.

Subscription Contracts (Sections 60 and 72 of the Corporation Code) Trillana vs. Quezon College, Inc. 93 Phil., 383 , June 27, 1953

Damasa Crisostomo subscribed 200 shares of capital stock with a par value of P100 each through a letter sent to the Board of Trustees of the Quezon College, enclosed with the letter are a sum of money as her initial payment and her assurance of full payment after she harvested fish. On October 26, 1948, Damasa Crisostomo passed away. As no

Whether or not the condition entered into by both parties are valid.

No, Under article 1115 of the old Civil Code which provides as follows: "If the fulfillment of the condition should depend upon the exclusive will of the debtor, the conditional obligation shall be void.

payment appears to have been made on the subscription mentioned in the foregoing letter, the Quezon College, Inc. presented a claim before the CFI of Bulacan in her testate proceeding, for the collection of the sum of P20,000, representing the value of the subscription to the capital stock of the Quezon College, Inc. which was then opposed by the administrator of the estate.

Bayla v. Silang Traffic Co. (1942)

Petitioners purchased the following: Sofronio T. Bayla.......

8 shares

P360

Venancio Toledo........

8 shares

375

Josefa Naval..............

15 shares

675

purchase price to be paid 5% upon the execution of the contract and the remainder in instalments of 5%, payable within the 1st month of each and every quarter starting July 1, 1935, w/ interest on deferred payments at 6%/annum until paid They also agreed to forfeit in favor of seller in case of default w/o court proceedings BOD resolution Aug 1, 1937: rescinding the agreement Petitioners filed an action in the CFI against Silang Traffic Co. Inc to recover certain sum of money w/c they had paid severally to the corp. on account of shares of stock they indiv. agreed to take and pay for under certain conditions Defenses: That the resolution is not applicable to the petitioners Sofronio T. Bayla, Josefa Naval, and Paz Toledo because on the date thereof "their subscribed shares of stock had already automatically reverted to the defendant, and the

W/N the subsequent BOD resolution is valid W/N under the contract between the parties the failure of the purchaser to pay any of the quarterly installments on the purchase price automatically gave rise to the forfeiture of the amounts already paid and the reversion of the shares to the corporation

As the appellant offered its stock for subscription on the term stated in a form letter, and Damasa Crisostomo (D.C.) applied for subscription fixing her own plan of payment, the relation, in the absence of acceptance by the appellant of the counter offer of D.C., had not ripened into an enforceable contract. There was imperative need for express acceptance on appellant's part, because the proposal of D. C. to pay the value of the subscription after she had harvested fish, was a condition obviously dependent upon her sole will and, therefore, facultative in nature, rendering the obligation void under article 1115 of the old Civil Code.” NO. CA reversed. Silang Traffic to pay petitioners 1. NO ▪ noted agreement is entitled "Agreement for Installment Sale of Shares in the Silang Traffic Company, Inc.,"; that while the purchaser is designated as "subscriber," the corporation is described as "seller" ▪ Whether a particular contract is a subscription or a sale of stock is a matter of construction and depends upon its terms and the intention of the parties ▪ subscription - mutual agreement of the subscribers to take and pay for the stock of a corporation ▪ purchase - independent agreement bet. the individual and the corp. to buy shares of stock from it at stipulated price ▪ rules governing subscriptions and sales of shares are different ▪ Corporation Law regarding calls for unpaid subscription and assessment of stock (sections 37-50) do not apply to a purchase of stock ▪ corporation has no legal capacity to release an original subscriber to its capital stock from the obligation to pay for his shares, is inapplicable to a contract of purchase of

installments paid by them had already been forfeited" that said resolution of August 1, 1937, was revoked and cancelled by a subsequent resolution



RTC: absolved defendant. BOD resolution cancelled Petitioners appealed





2.







Subscription Contracts (Sections 60 and 72 of the Corporation Code)

shares. The contract in question being one of purchase and not subscription as we have heretofore pointed out, we see no legal impediment to its rescission by agreement of the parties We may add that there is no intimation in this case that the corporation was insolvent, or that the right of any creditor of the same was in any way prejudiced by the rescission. The attempted revocation of said rescission by the resolution of August 22, 1937, was invalid, it not having been agreed to by the petitioners. NO The provision regarding interest on deferred payments would not have been inserted if it had been the intention of the parties to provide for automatic forfeiture and cancelation of the contract contract did not expressly provide that the failure of the purchaser to pay any installment would give rise to forfeiture and cancelation without the necessity of any demand from the seller Art. 1100 of the Civil Code: persons obliged to deliver or do something are not in default until the moment the creditor demands of them judicially or extrajudicially the fulfillment of their obligation, unless (1) the obligation or the law expressly provides that demand shall not be necessary in order that default may arise (2) by reason of the nature and circumstances of the obligation it shall appear that the designation of the time at which that thing was to be delivered or the service rendered was the principal inducement to the creation of the obligation.

Release from subscription obligation VELASCO VS. POIZAT G.R. No. L-11528

The plaintiff, as assignee in insolvency of "The Philippine Chemical Product Company" (Ltd.) is seeking to recover of the defendant, Jean M. Poizat, the sum of P1,500, upon a subscription made by him to the corporate stock of said company. It appears that the corporation in question was originally organized by several residents of the city of Manila, where the company had its principal place of business, with a capital of P50,000, divided into 500 shares. The defendant subscribed for 20 shares of the stock of the company, an paid in upon his subscription the sum of P500, the par value of 5 shares . The action was brought to recover the amount subscribed upon the remaining shares. It appears that the defendant was a stock holder in the company from the inception of the enterprise, and for sometime acted as its treasurer and manager. While serving in this capacity he called in and collected all subscriptions to the capital stock of the company, except the aforesaid 15 shares subscribed by himself and another 15 shares owned by Jose R. Infante. A meeting of the board of directors of the company was held at which a majority of the stock was presented. Upon this occasion two resolutions were adopted. The first was a proposal that the directors, or shareholders, of the company should make good by new subscriptions, in proportion to their respective holdings, 15 shares which had been surrendered by Infante. It seems that this shareholder had already paid 25 per cent of his subscription upon 20 shares, leaving 15 shares unpaid for, and an understanding had been reached by him and the management by which he was to be released from the obligation of his subscription, it being understood that what he had already paid should not be refunded. Accordingly the directors present at this meeting subscribed P1,200 toward taking up his shares, leaving a deficiency of P300 to be recovered by voluntary subscriptions from stockholders not present at the meeting. The other proposition was o the effect that Juan [Jean] M. Poizat, who was absent, should be required to pay the amount of his subscription upon the 15 shares for which he was still indebted to the company. The resolution further provided that, in case he should refuse to make

WON Poizat is liable upon this subscription?

Poizat is liable upon his subscription. Section 36 of the Corporation Law clearly recognizes that a stock subscription is subsisting liability from the time the subscription is made, since it requires the subscriber to pay interest quarterly from that date unless he is relieved from such liability by the bylaws of the corporation. The subscriber is as much bound to pay the amount of the share subscribed by him as he would be to pay any other debt, and the right of the company to demand payment is no less incontestable. The provisions of the Corporation Law (Act No. 1459) given recognition of two remedies for the enforcement of stock subscriptions. The first and most special remedy given by the statute consists in permitting the corporation to put up the unpaid stock for sale and dispose of it for the account of the delinquent subscriber. In this case the provisions of section 38 to 48, inclusive, of the Corporation Law are applicable and must be followed. Nothing in this Act shall prevent the directors from collecting, by action in any court of proper jurisdiction, the amount due on any unpaid subscription, together with accrued interest and costs and expenses incurred. The assignee of the insolvent corporation succeeds to all the corporate rights of action vested in the corporation prior to its insolvency; and the assignee therefore has the same freedom with respect to suing upon the stock subscription as the directors themselves would have had under section 49 above cited. There is another reason why the present plaintiff must prevail in this case. That reason is this: When insolvency supervenes upon a corporation and the court assumes jurisdiction to wind up, all unpaid stock subscriptions become payable on demand, and are at once recoverable in an action instituted by the assignee or receiver appointed by the court. It is now quite well settled that when the corporation becomes insolvent, with

such payment, the management of the corporation should be authorized to undertake judicial proceedings against him. When notification of this resolution reached Poizat through the mail it evoked from him a manifestation of surprise and pain, which found expression in a letter written by him in reply, dated July 27, 1914, and addressed to Velasco, as treasurer and administrator. In this letter Poizat states that he had been given to understand by some member of the board of directors that he was to be relieved from his subscription upon the terms conceded to Infante. The company soon went into voluntary insolvency, Velasco being named as the assignee. At the hearing of the Court of First Instance, judgment was rendered in favor of the defendant, and the complaint was dismissed. From this action the plaintiff has appealed.

proceedings instituted by creditors to wind up and distribute its assets, no call or assessment is necessary before the institution of suits to collect unpaid balances on subscription. It evidently cannot be permitted that a subscriber should escape from his lawful obligation by reason of the failure of the officers of the corporation to perform their duty in making a call; and when the original model of making the call becomes impracticable, the obligation must be treated as due upon demand. The better doctrine is that when insolvency supervenes all unpaid subscriptions become at once due and enforceable.

The circumstance that the board of directors in their meeting of July 13, 1914, resolved to release Infante from his obligation upon a subscription for 15 shares is no wise prejudicial to the right of the corporation or its assignee to recover from Poizat upon a subscription made by him. In releasing Infante the board transcended its powers, and he no doubt still remained liable on such of his shares as were not taken up and paid for by other persons.The general doctrine is that the corporation has no legal capacity to release an original subscriber to its capital stock from the obligation of paying for his shares, in whole or in part.The suggestion contained in Poizat's letter of July 27, 1914, to the effect that he understood that he was to be relieved upon the same terms as Infante is, for the same reason, of no merit as matter of defense, even if an agreement to that effect had been duly proved. Doctrines A stock subscription is a contract between the corporation and the subscriber, and courts will enforce it for or against either. No express promise to pay is necessary to make the

subscriber liable. The corporation has two remedies against the subscriber to the corporate shares, namely (1) to sell the stock for the account of the delinquent subscriber, and (2) to bring a legal action against him for the amount due. A corporation has no legal capacity to release a subscriber to its capital stock from the obligation to pay for his shares; and any agreement to this effect is invalid.

Philippine National Bank vs. Bitulok Sawmill, Inc. 23 SCRA 1366 (1968)

The Philippine Lumber Distributing Agency, Inc., according to the lower court, "was organized sometime in the early part of 1947 upon the initiative and insistence of the late President Manuel Roxas of the Republic of the Philippines who for the purpose, had called several conferences between him and the subscribers and organizers of the Philippine Lumber Distributing Agency, Inc." The purpose was praiseworthy, to insure a steady supply of lumber, which could be sold at reasonable prices to enable the war sufferers to rehabilitate their devastated homes. At the beginning, the lumber producers were reluctant to organize the cooperative agency as they believed that it would not be easy to eliminate from the retail trade the alien middlemen who had been in this business from time immemorial, but because the late President Roxas made it clear that such a cooperative agency would not be successful without substantial working capital which the lumber producers could not entirely shoulder, and as an inducement he promised and agreed to finance the agency by making the Government invest P9.00 by way of counterpart for every peso that the members would invest therein." Accordingly, "the late President Roxas instructed the Hon. Emilio Abello, then Executive Secretary and Chairman of the Board of Directors of the Philippine National Bank, for the latter to grant saidagency an overdraft in the original sum of P250,000.00 which was later increased to P350,000.00, which was approved by said Board of Directors of the Philippine National Bank on July 28, 1947, payable on or before April 30, 1958, with interest at the rate of 6% per annum, and secured by the chattel mortgages on the stock of lumber of said agency." The Philippine Government did not invest the P9.00 for every peso coming from defendant lumber producers. The loan extended to the Philippine Lumber Distributing Agency by the Philippine National Bank was not paid.

Whether or not the noncompliance with a plain statutory command, considering the persuasiveness of the plea that defendantsappellees would "not have subscribed to the capital stock" of the Philippine Lumber Distributing Agency "were it not for the assurance of the then President of the Republic that the Government would back it up by investing P9.00 for every peso" subscribed, a condition which was not fulfilled, such commitment not having been complied with, be justified.

NO. It would be unwarranted to ascribe to the late President Roxas the view that the payment of the stock subscriptions, as thus required by law, could be condoned in the event that the counterpart fund to be invested by the Government would not be available. Even if such were the case, however, and such a promise were in fact made, to further the laudable purpose to which the proposed corporation would be devoted and the possibility that the lumber producers would lose money in the process, still the plain and specific wording of the applicable legalprovision as interpreted by this Court must be controlling. It is a well-settled principle that with all the vast powers lodged in the Executive, he is still devoid of the prerogative of suspending the operation of any statute or any of its terms. DOCTRINES It is an established doctrine that subscriptions to the capital of a corporation constitute a fund to which creditors have a right to look for satisfaction of their claims and that the assignee in insolvency can maintain an action upon any unpaid stock subscription in order to realize assets for the payment of its debts. A corporation has no power to release an original subscriber to its capital stock from the obligation of paying for his shares, without a valuable consideration for such release; and as against creditors a reduction of the capital stock can take place only in the manner and under the conditions prescribed by the statute or the charter or the articles of incorporation

Formalities in Organizing Generally Government of the PI vs.

This is a petition in the Supreme Court of the extraordinary

Whether

or

not

the

Inasmuch as Act No. 1510 is the charter of Manila

Manila Railroad, 103 PHIL 757 (1929)

legal writ of mandamus presented by the Government of the Philippine Islands, praying that the writ be issued to compel the Manila Railroad Company and Jose Paez, as its manager, to provide and equip the telegraph poles of said company between the municipality of Paniqui, Province of Tarlac, and the Municipality of San Fernando, Province of La Union, with crosspieces for six telegraph wires belonging to the Government, which, it is alleged, are necessary for public service between said municipalities. The government of the Philippines entered into a contract with the manila rail road company under a special charter act no. 1510. The government of the Philippines is now demanding from the defendant that it should provide and equip its telegraph poles with crosspieces to carry six telegraph wires of the Government. This claim is based on the provisions of section 84 of act No. 1459. Act No. 1459 is the General Corporation Law and was adopted by the United States Philippine Commission on March 1, 1906. (Vol. 5, Pub. Laws, pp. 224-268.) Section 84 of the said Act provides: The railroad corporation shall establish along the whole length of the road a telegraph line for the use of the railroad. The posts of this line may be used for Government wires and shall be of sufficient length and strength and equipped with sufficient crosspiece to carry the number of wires which the Government may consider necessary for the public service. The establishment, protection, and maintenance of the wires and stations necessary for the public service shall be at the cost of the Government. (Vol. 5, P. L., p. 247.) The defense of the defendant is that it is not bound by the provisions of the corporation code because it has a charter of his own Act. No 1510. Under that act the government is entitled to place on the poles of the company four wires only.

provisions of the corporation law apply between the parties.

Railroad Company and constitute a contract between it and the Governmemnt, it would seem that the company is governd by its contract and not by the provisions of any general law upon questions covered by said contract. From a reading of the said charter or contract it would be seen that there is no indication that the Government intended to impose upon said company any other conditions as obligations not expressly found in said charter or contract. If that is true, then certainly the Government cannot impose upon said company any conditions or obligations found in any general law, which does not expressly modify said contract. Section 84 of the Corporation Law (Act No. 1459) was intended to apply to all railways in the Philippine Islands which did not have a special charter contract. Act No. 1510 applies only to the Manila Railroad Company, one of the respondents, and being a special charter of said company, its adoption had the effect of superseding the provisions of the general Corporation Law which are applicable to railraods in general. The special charter (Act No. 1510) had the effect of superseding the general Corporation Law upon all matters covered by said special charter. Said Act, inasmuch as it contained a special provision relating to the erection of telegraph and telephone poles, and the number of wires which the Government might place thereon, superseded the general law upon that question. Act No. 1510 of the United States Philippine Commission (vol. 5, P. L., pp. 350-358), and that under the provisions of said Act No. 1510 the Government is entitled to place on the poles of the company four wires only.

Rural Bank of Salinas, Inc. v. CA 210 SCRA 510

Clemente G. Guerrero, President of the Rural Bank of Salinas, Inc., executed a Special Power of Attorney in favor of his wife, Melania to sell or otherwise dispose of and/or mortgage 473 shares of stock of the Bank registered in his name (represented by the Bank's stock certificates nos. 26, 49 and 65), to execute the proper documents therefor, and to receive and sign receipts for the dispositions. Melania, as Attorney-in-Fact, executed a Deed of Assignment for 472 shares out of the 473 shares, in favor of private respondents Luz Andico (457 shares), Wilhelmina Rosales (10 shares) and Francisco Guerrero, Jr. (5 shares). Melania Guerrero presented to Rural Bank of Salinas the 2 Deeds of Assignment for registration with a request for the transfer in the Bank's stock and transfer book of the 473 shares of stock so assigned, the cancellation of stock certificates in the name of Clemente, and the issuance of new stock certificates in the name of the new owners thereof., Rural Bank denied such request. Melania filed with the SEC an action for mandamus against Rural Bank of Salinas, its President and Corporate Secretary. The Bank in their Answer with counterclaim alleged that upon the death of Clemente, his 473 shares of stock became the property of his estate, and his property and that of his widow should first be settled and liquidated in accordance with law before any distribution can be effected so that petitioners may not be a party to any scheme to evade payment of estate or inheritance tax and in order to avoid liability to any third persons or creditors of the late Clemente. Maripol Guerrero filed a motion for intervention (legally adopted daughter of the late Clemente and Melanie) stating that a Petition for the administration of the estate of Clemente had been filed but her motion was denied. She then filed before the CFI of Rizal, against Melanie for the annulment of the Deeds of Assignment for being fictitious, void or simulated. The Bank then filed a motion to dissmiss/suspend hearing pending resolution of the case for annulment. However, SEC denied such motion. SEC rendered a Decision granting the writ of Mandamus and directing petitioners to cancel stock certificates of the Bank, and to issue new certificates in the names of private respondents, except Melania Guerrero. Appealed to the CA

1. WON SEC has the power to adjudicate the case. -Yes 2. WON corporatons may by its board, its bylaws, or the act of its officers create restrictions in stock transfers.No. 3. WON the Bank being a corporation may refuse to transfer and register stocks. No.

1. Section 5 (b) of P.D. No. 902-A grants to the SEC the original and exclusive jurisdiction to hear and decide cases involving intracorporate controversies. An intracorporate controversy has been defined as one which arises between a stockholder and the corporation. There is no distinction, qualification, nor any exception whatsoever (Rivera vs. Florendo, 144 SCRA 643 [1986]). The case at bar involves shares of stock, their registration, cancellation and issuances thereof by petitioner Rural Bank of Salinas. It is therefore within the power of respondent SEC to adjudicate. 2. A corporation, either by its board, its by-laws, or the act of its officers, cannot create restrictions in stock transfers, because:. . Restrictions in the

traffic of stock must have their source in legislative enactment, as the corporation itself cannot create such impediment. By-laws are intended merely for the protection of the corporation, and prescribe regulation, not restriction; they are always subject to the charter of the corporation. The corporation, in the absence of such power, cannot ordinarily inquire into or pass upon the legality of the transactions by which its stock passes from one person to another, nor can it question the consideration upon which a sale is based. . . . (Tomson on Corporation Sec. 4137, cited in Fleisher vs. Nolasco, Supra). The only limitation imposed by Section 63 of the Corporation Code is when the corporation holds any unpaid claim against the shares intended to be transferred, which is absent here. 3. The right of a transferee/assignee to have stocks transferred to his name is an inherent right flowing from his ownership of the stocks. Respondent SEC correctly ruled in favor of the registering of the shares of stock in question in private respondent's names. Such ruling finds support under Section 63 of the Corporation Code,

but CA affirmed the decision of SEC.

to wit:

Sec. 63. . . . Shares of stock so issued are personal property and may be transferred by delivery of the certificate or certificates indorsed by the owner or his attorney-in-fact or other person legally authorized to make the transfer. No transfer, however, shall be valid, except as between the parties, until the transfer is recorded in the books of the corporation . . . The corporation's ministerial.

obligation

to

register

is

In transferring stock, the secretary of a corporation acts in purely ministerial capacity, and does not try to decide the question of ownership. (Fletcher, Sec. 5528, page 434). The duty of the corporation to transfer is a ministerial one and if it refuses to make such transaction without good cause, it may be compelled to do so by mandamus. (See. 5518, 12 Fletcher 394) Formalities in Organizing Articles of Incorporation, Procedure and Documentary Requirements, As to corporate name (Section 18) Red Line Transportation Thi Co. vs. Rural Transit Co. GR No. 41570 | Sept. 6, 1934

This is a petition for review of an order of the Public Service Commission granting to the Rural Transit Company, Ltd., a certificate of public convenience to operate a transportation service between Ilagan in the Province of Isabela and Tuguegarao in the Province of Cagayan, and additional trips in its existing express service between Manila Tuguegarao. On June 4, 1932, Rural Transit filed an application for certification of a new service between Tuguegarao and Ilagan with the Public Company Service Commission (PSC), since the present service is not sufficient Rural Transit further stated that it is a holder of a certificate of public convenience to operate a passenger bus service between Manila and Tuguegarao. Red Line opposed said application, arguing that they already hold a certificate of public convenience for Tuguegarao and Ilagan, and is rendering

Can the Public Service Commission authorize a corporation to assume the name of another corporation as a trade name?

NO The Rural Transit Company, Ltd., and the Bachrach Motor Co., Inc., are Philippine corporations and the very law of their creation and continued existence requires each to adopt and certify a distinctive name The incorporators "constitute a body politic and corporate under the name stated in the certificate." A corporation has the power "of succession by its corporate name." It is essential to its existence and cannot change its name except in the manner provided by the statute. By that name alone is it authorized to transact business. The law gives a corporation no express or

adequate service. They also argued that granting Rural Transit’s application would constitute a ruinous competition over said route. On Dec. 21, 1932, Public Service Commission approved Rural Transit’s application, with the condition that "all the other terms and conditions of the various certificates of public convenience of the herein applicant and herein incorporated are made a part hereof." A motion for rehearing and reconsideration was filed by Red Line since Rural Transit has a pending application before the Court of First Instance for voluntary dissolution of the corporation. A motion for postponement was filed by Rural Transit as verified by M. Olsen who swears "that he was the secretary of the Rural Transit Company, Ltd. During the hearing before the Public Service Commission, the petition for dissolution and the CFI’s decision decreeing the dissolution of Rural Transit were admitted without objection At the trial of this case before the Public Service Commission an issue was raised as to who was the real party in interest making the application, whether the Rural Transit Company, Ltd., as appeared on the face of the application, or the Bachrach Motor Company, Inc., using name of the Rural Transit Company, Ltd., as a trade name. However, PSC granted Rural Transit’s application for certificate of public convenience and ordered that a certificate be issued on its name. PSC relied on a Resolution in case No. 23217, authorizing Bachrach Motor to continue using Rural Transit’s name as its tradename in all its applications and petitions to be filed before the PSC. Said resolution was given a retroactive effect as of the date of filing of the application or April 30, 1930. PHILIPPINE FIRST INSURANCE COMPANY, INC. vs. MARIA CARMEN HARTIGAN, CGH, and O. ENGKEE

On June 1, 1953, plaintiff was originally named as 'The Yek Tong Lin Fire and Marine Insurance Co., Ltd’ an insurance corp. duly presented with the Security and Exchange Commissioner and before a Notary Public as provided in their articles of incorporation. Later amended its articles of incorporation and changed its name on May 26, 1961 as ‘Philippine First Insurance Co., Inc.’ pursuant to a certificate of the Board of Directors. The complaint alleges that: Philippine First

implied authority to assume another name that is unappropriated: still less that of another corporation, which is expressly set apart for it and protected by the law. If any corporation could assume at pleasure as an unregistered trade name the name of another corporation, this practice would result in confusion and open the door to frauds and evasions and difficulties of administration and supervision. In this case, the order of the commission authorizing the Bachrach Motor Co., Incorporated, to assume the name of the Rural Transit Co., Ltd. likewise incorporated, as its trade name being void. Accepting the order of December 21, 1932, at its face as granting a certificate of public convenience to the applicant Rural Transit Co., Ltd., the said order last mentioned is set aside and vacated on the ground that the Rural Transit Company, Ltd., is not the real party in interest and its application was fictitious.

May a Philippine corporation change its name and still retain its original personality and individuality as such?

YES. As can be gleaned under Sections 6 and 18 of the Corporation Law, the name of a corporation is peculiarly important as necessary to the very existence of a corporation. The general rule as to corporations is that each corporation shall have a name by which it is to sue and be sued and do all legal acts. The name of a corporation in this respect designates the corporation in the same manner as the name of an individual designates

Insurance Co., Inc., doing business under the name of 'The Yek Tong Lin Fire and Marine Insurance Co., Lt.' signed as co-maker together with defendant Maria Carmen Hartigan, CGH, to which a promissory note was made in favour of China Banking. Said defendant failed to pay in full despite renewal of such note. The complaint ends with a prayer for judgment against the defendants, jointly and severally, for the sum of P4,559.50 with interest at the rate of 12% per annum from November 23, 1961 plus P911.90 by way of attorney's fees and costs. Defendants admitted the execution of the indemnity agreement but they claim that they signed said agreement in favor of the Yek Tong Lin Fire and Marine Insurance Co., Ltd.' and not in favor of the plaintiff Philippine Insurance. They likewise admit that they failed to pay the promissory note when it fell due but they allege that since their obligation with the China Banking Corporation based on the promissory note still subsists, the surety who co-signed the promissory note is not entitled to collect the value thereof from the defendants otherwise they will be liable for double amount of their obligation, there being no allegation that the surety has paid the obligation to the creditor. In their special defense, defendants claim that there is no privity of contract between the plaintiff and the defendants and consequently, the plaintiff has no cause of action against them, considering that the complaint does not allege that the plaintiff and the 'Yek Tong Lin Fire and Marine Insurance Co., Ltd.' are one and the same or that the plaintiff has acquired the rights of the latter.

the person." Since an individual has the right to change his name under certain conditions, there is no compelling reason why a corporation may not enjoy the same right. There is nothing sacrosanct in a name when it comes to artificial beings. The sentimental considerations which individuals attach to their names are not present in corporations and partnerships. Of course, as in the case of an individual, such change may not be made exclusively. by the corporation's own act. It has to follow the procedure prescribed by law for the purpose; and this is what is important and indispensably prescribed — strict adherence to such procedure. A general power to alter or amend the charter of a corporation necessarily includes the power to alter the name of the corporation. Hence, a mere change in the name of a corporation, either by the legislature or by the corporators or stockholders under legislative authority, does not, generally speaking, affect the identity of the corporation, nor in any way affect the rights, privileges, or obligations previously acquired or incurred by it. Indeed, it has been said that a change of name by a corporation has no more effect upon the identity of the corporation than a change of name by a natural person has upon the identity of such person. The corporation, upon such change in its name, is in no sense a new corporation, nor the successor of the original one, but remains and continues to be the original corporation. It is the same corporation with a different name, and its character is in no respect changed. ... (6 Fletcher, Cyclopedia of the Law of Private Corporations, 224-225, citing cases.) As correctly pointed out by appellant, the approval by the stockholders of the amendment of its articles of incorporation changing the name "The Yek Tong Lin Fire & Marine Insurance Co., Ltd." to "Philippine First Insurance Co., Inc." on March 8, 1961, did not automatically change the

name of said corporation on that date. To be effective, Section 18 of the Corporation Law, earlier quoted, requires that "a copy of the articles of incorporation as amended, duly certified to be correct by the president and the secretary of the corporation and a majority of the board of directors or trustees, shall be filed with the Securities & Exchange Commissioner", and it is only from the time of such filing, that "the corporation shall have the same powers and it and the members and stockholders thereof shall thereafter be subject to the same liabilities as if such amendment had been embraced in the original articles of incorporation." It goes without saying then that appellant rightly acted in its old name when on May 15, 1961, it entered into the indemnity agreement, Annex A, with the defendant-appellees; for only after the filing of the amended articles of incorporation with the Securities & Exchange Commission on May 26, 1961, did appellant legally acquire its new name; and it was perfectly right for it to file the present case In that new name on December 6, 1961. Such is, but the logical effect of the change of name of the corporation upon its actions. Therefore, actions brought by a corporation after it has changed its name should be brought under the new name although for the enforcement of rights existing at the time the change was made. The change in the name of the corporation does not affect its right to bring an action on a note given to the corporation under its former name.

Universal Mills ● Corporation vs. Universal Textile Mills 78 SCRA 62 (1977)

This is an Exchange respondent changed as of

appeal from the order of the Securities and Commission granting a petition by the to have the petitioner’s corporate name be it is “confusingly and deceptively similar” to that the former.

Whether or not petioner’s trade name is confusingly similar with that of respondent’s

he corporate names in question are not identical, but they are indisputably so similar that even under the test of reasonable care and observation as the public generally are capable of using and may be expected to exercise” invoked by appellant. We are apprehensive confusion will



● ●



"Universal Textile Mills" was organized on Dec 1953 Another company, "Universal Hosiery Mills Corporation" (organized Oct 1954) manufactured hosiery and wearing apparel. It amended its articles of incorporation changing its name to Universal Mills Corporation Fire razed Universal Textile Mills' factory in Pasig. Petitioner Universal Mills alleged that as a result of the fire and because of news items appearing in various newspapers, it created uncertainty and confusion among its bankers, stockholders and customers. Petitioner filed said complaint to change corporate name.

usually arise, considering that x x x appellant included among its primary purposes the manufacturing, dyeing, finishing and selling of fabrics of all kinds” which respondent had been engaged for more than a decade ahead of petitioner.

SEC: Ordered Universal Mills to change its name on ground of being confusingly and deceptively similar. Going further:

"This SEC further takes cognizance of the fact that when the company filed the amendment changing its name to Universal Mills Corporation, it filed a written undertaking.. signed by its President, Mr. Cokiat, promising to change its name in the event that there is another person, firm or entity who has obtained a prior right to the use of such name or one similar to it. That promise is still binding upon the corporation & its responsible officers G.R. No. Respondent Iglesia ng Dios Kay Cristo Jesus, Haligi at 137592 December Suhay ng Katotohanan (Church of God in Christ Jesus, the 12, 2001 Pillar and Ground of Truth), is a non-stock religious society or corporation registered in 1936. Sometime in 1976, one ANG MGA KAANIB SA Eliseo Soriano and several other members of respondent IGLESIA NG DIOS KAY corporation disassociated themselves from the latter and KRISTO HESUS, H.S.K. SA succeeded in registering in 1977 a new non-stock religious BANSANG PILIPINAS, society or corporation, named Iglesia ng Dios Kay Kristo INC., petitioner, Hesus, Haligi at Saligan ng Katotohanan. vs. IGLESIA NG DIOS KAY Consequently, respondent corporation (Iglesia ng Dios Kay CRISTO JESUS, HALIGI AT Cristo Jesus) filed with the SEC a petition to compel the SUHAY NG Iglesia ng Dios Kay Kristo Hesus, Haligi at Saligan ng KATOTOHANAN, Katotohanan to change its corporate name, which petition respondent. was docketed as SEC Case No. 1774. The SEC rendered judgment in favor of respondent corporation (Iglesia ng Dios Kay Cristo Jesus), ordering the

Whether or not petitioner corporation's (Ang Mga Kaanib) corporate name is identical or confusingly or deceptively similar to that of respondent corporation's (Iglesia ng Dios Kay Cristo Jesus) corporate name.

Yes. The SEC has the authority to de-register at all times and under all circumstances corporate names which in its estimation are likely to spawn confusion. It is the duty of the SEC to prevent confusion in the use of corporate names not only for the protection of the corporations involved but more so for the protection of the public. Section 18 of the Corporation Code provides: Corporate Name. — No corporate name may be allowed by the Securities and Exchange Commission if the proposed name is identical or deceptively or confusingly similar to that of any existing corporation or to any other name already protected by law or is patently

Iglesia ng Dios Kay Kristo Hesus, Haligi at Saligan ng Katotohanan to change its corporate name to another name that is not similar or identical to any name already used by a corporation, partnership or association registered with the Commission.

It appears that during the pendency of SEC Case No. 1774, Soriano, et al., caused the registration in 1980 of petitioner corporation, Ang Mga Kaanib sa Iglesia ng Dios Kay Kristo Hesus, H.S.K, sa Bansang Pilipinas. The acronym "H.S.K." stands for Haligi at Saligan ng Katotohanan. Subsequently, respondent corporation (Iglesia ng Dios Kay Cristo Jesus) filed before the SEC a petition praying that petitioner corporation (Ang Mga Kaanib) be compelled to change its corporate name and be barred from using the same or similar name on the ground that the same causes confusion among their members as well as the public. The SEC rendered a decision ordering petitioner corporation (Ang Mga Kaanib) to change its corporate name. Petitioner corporation (Ang Mga Kaanib) appealed to the SEC En Banc. The SEC En Banc affirmed the above decision, upon a finding that petitioner corporation's (Ang Mga Kaanib) corporate name was identical or confusingly or deceptively similar to that of respondent corporation's (Iglesia ng Dios Kay Cristo Jesus) corporate name. Petitioner corporation (Ang Mga Kaanib) filed a petition for review with the CA. The CA affirmed the decision of the SEC En Banc. Petitioner corporation (Ang Mga Kaanib) claims that it complied with the aforecited SEC guideline by adding not only two but eight words to their registered name, to wit: "Ang Mga Kaanib" and "Sa Bansang Pilipinas, Inc.," which, petitioner corporation (Ang Mga Kaanib) argues, effectively distinguished it from respondent corporation (Iglesia ng

deceptive, confusing or is contrary to existing laws. When a change in the corporate name is approved, the Commission shall issue an amended certificate of incorporation under the amended name. Corollary thereto, the pertinent portion of the SEC Guidelines on Corporate Names states: (d) If the proposed name contains a word similar to a word already used as part of the firm name or style of a registered company, the proposed name must contain two other words different from the name of the company already registered; Parties organizing a corporation must choose a name at their peril; and the use of a name similar to one adopted by another corporation, whether a business or a nonprofit organization, if misleading or likely to injure in the exercise of its corporate functions, regardless of intent, may be prevented by the corporation having a prior right, by a suit for injunction against the new corporation to prevent the use of the name. In the case at bar, the additional words "Ang Mga Kaanib" and "Sa Bansang Pilipinas, Inc." in petitioner corporation's (Ang Mga Kaanib) name are, as correctly observed by the SEC, merely descriptive of and also referring to the members, or kaanib, of respondent corporation (Iglesia ng Dios Kay Cristo Jesus) who are likewise residing in the Philippines. These words can hardly serve as an effective differentiating medium necessary to avoid confusion or difficulty in distinguishing petitioner from respondent. This is especially so, since both petitioner and respondent corporations

Dios Kay Cristo Jesus).

are using the same acronym — H.S.K.; not to mention the fact that both are espousing religious beliefs and operating in the same place. Parenthetically, it is well to mention that the acronym H.S.K. used by petitioner corporation (Ang Mga Kaanib) stands for "Haligi at Saligan ng Katotohanan." Significantly, the only difference between the corporate names of petitioner and respondent are the words SALIGAN and SUHAY. These words are synonymous — both mean ground, foundation or support. Hence, this case is on all fours with

Universal Mills Corporation v. Universal Textile Mills, Inc., where the Court ruled that the

corporate names Universal Mills Corporation and Universal Textile Mills, Inc., are undisputably so similar that even under the test of "reasonable care and observation" confusion may arise.

G.R. No. L-15429 December 1, 1919 UY SIULIONG, MARIANO LIMJAP, GACU UNG JIENG, EDILBERTO CALIXTO and UY CHO YEE, petitioners, vs. THE DIRECTOR OF COMMERCE AND INDUSTRY, respondent.

Formalities in Organizing Articles of Incorporation, As to Purpose (Section 14 [2]) The purpose of this action is to obtain the writ Whether or not a of mandamus to require the respondent to file and register, corporation organized upon the payment of the lawful fee, articles of for commercial incorporation, and to issue to the petitioners as the purposes in the incorporators of a certain corporation to be known as Philippine Islands can be "Siuliong y Compañia, Inc.," a certificate under the seal of organized for more than the office of said respondent, certifying that the articles of one purpose? incorporation have been duly filed and registered in his office in accordance with the law. That prior to the presentation of the petition, petitioners associated together as partners, which partnership was known as "mercantil regular colectiva, under the name of "Siuliong y Cia.;" Petitioners have been members of said partnership of "Siuliong y Cia.," desired to dissolve the partnership and to form a corporation composed of the same persons as incorporators, to be known as "Siulong y Compañia, Incorporada;" That the purpose of said corporation, "Siuliong y Cia., Inc.," is to acquire the business of the partnership theretofore known as Siuliong & Co., and to

YES. Considering the purposes and objects of the

proposed articles of incorporation which are enumerated, we are of the opinion that it contains nothing which violates in the slightest degree any of the provisions of the laws of the Philippine Islands, and the petitioners are, therefore, entitled to have such articles of incorporation filed and registered as prayed for by them and to have issued to them a certificate under the seal of the office of the respondent, setting forth that such articles of incorporation have been duly filed in his office. (Sec. 11, Act No. 1459.) Therefore, the petition prayed for is hereby granted, and without any finding as to costs, it is so ordered. Doctrine

continue said business with some of its objects or purposes; An examination of the articles of incorporation of the said "Siuliong y Compañia, Incorporada" (Exhibit A) shows that it is to be organized for the purchase and sale, importation and exportation, of the products of the country as well as of foreign countries; To discount promissory notes, bills of exchange, and other negotiable instruments; The purchase and sale of bills of exchange, bonds, stocks, or joint account of mercantile and industrial associations and of all classes of mercantile documents; commissions, consignments;"xxx.. The respondent contends (a) that the proposed articles of incorporation presented for file and registry permitted the petitioners to engage in a business which had for its end more than one purpose; (b) that it permitted the petitioners to engage in the banking business, and (c) to deal in real estate, in violation of the Act of Congress of July 1, 1902. The petitioners, insisted that said proposed articles of incorporation do not permit it to enter into the banking business nor to engage in the purchase and sale of real estate in violation of said Act of Congress, expressly renounced in open court their right to engage in such business under their articles of incorporation, even though said articles might be interpreted in a way to authorize them to so to do.

That under the laws of the Philippine Islands, a corporation may be organized for "mercantile purposes" and to engage in such incidental business as may be necessary and advisable to give effect to, and aid in, the successful operation and conduct of the principal business; that all of the power and authority included in the articles of incorporation of Siuliong & Co., Inc., were only incidental to the' principal purpose of its proposed incorporation, to wit: "mercantile business."

Formalities in Organizing Articles of Incorporation, As to principal office (Section 14 [3]) No. L-22238. February 18, 1967. CLAVECILLA RADIO SYSTEM, petitioner and appellant, vs. HON. AGUSTIN ANTILLON, as City Judge of the Municipal Court of Cagayan de Oro City and NEW CAGAYAN GROCERY, respondents and appellees.

1.

2.

3. 4.

New Cagayan Grocery (NECAGRO) filed a complaint for damages against Clavecilla Radio system. They alleged that Clavecilla omitted the word “NOT” in the letter addressed to NECAGRO for transmittal at Clavecilla Cagayan de Oro Branch. NECAGRO alleged that the omission of the word “not” between the word WASHED and AVAILABLE altered the contents of the same causing them to suffer from damages. Clavecilla filed a motion to dismiss on the ground of failure to state a cause of action and improper venue. City Judge of CDO denied the MTD. Clavecilla filed a petition for prohibition with preliminary Injunction with

WON the present case against Clavecilla should be filed in Manila where it holds its principal office.

It is clear that the case from damages is based upon a written contract. Under par. (b)(3) Sec. 1 Rule 4 of the New Rules of Court, when an action is not upon a written contract then the case should be filed in the municipality where the defendant or any of the defendant resides or maybe served upon with summons. In corpo. Law, the residence of the corporation is the place where the principal office is established. Since Clavecilla’s principal office is in Manila, then the suit against it may properly be file in the City of Manila. As stated in Evangelista v. Santos, the laying of the venue of an action is not left to plaintiff’s

5.

6.

the CFI praying that the City Judge be enjoined from further proceeding with the case because of improper venue. CFI – dismissed the case and held that Clavecilla may be sued either in Manila (principal office) or in CDO (branch office). Clavecilla appealed to the SC contending that the suit against it should be filed in Manila where it holds its principal office.

caprice because the matter is regulated by the Rules of Court. Doctrines The residence of a corporation is the place where its principal office is established. It can be sued in that place, not in the place where its branch office is located. Where the action filed against a corporation in the inferior court is based on tort, it should be filed in the place where the corporation has its principal office, not in the place where it has its branch office. To allow an action against a corporation to be instituted in any place where a corporate entity has its branch offices would create confusion and work untold inconvenience to the corporation.

Formalities in Organizing Articles of Incorporation, As to Corporate Term (Section 11) Petitioner Alhambra Cigar and Cigarette Manufacturing Company, Inc. (hereinafter referred to simply as Alhambra) was duly incorporated under Philippine laws on January 15, 1912. By its corporate articles it was to exist for fifty (50) years from incorporation. Its term of existence expired on January 15, 1962. On that date, it ceased transacting business, entered into a state of liquidation. Thereafter, a new corporation. — Alhambra Industries, Inc. — was formed to carry on the business of Alhambra. On May 1, 1962, Alhambra’s stockholders, by resolution named Angel S. Gamboa trustee to take charge of its liquidation. On June 20, 1963 — within Alhambra’s three-year statutory period for liquidation – Republic Act 3531 was enacted into law. It amended Section 18 of the Corporation Law; it empowered domestic private corporations to extend their corporate life beyond the period fixed by the articles of incorporation for a term not to exceed fifty years in any one instance. Previous to Republic Act 3531, the maximum nonextendible term of such corporations was fifty years. On July 15, 1963, at a special meeting, Alhambra’s board of directors resolved to amend paragraph “Fourth” of its articles of incorporation to extend its corporate life for an

Whether or not the corporate life of a corporation be extended during the period of winding up or after it’s charter has already expired.

No. The common law rule, at the beginning, was rigid and inflexible in that upon its dissolution, a corporation became legally dead for all purposes. Statutory authorizations had to be provided for its continuance after dissolution “for limited and specified purposes incident to complete liquidation of its affairs”. Thus, the moment a corporation’s right to exist as an “artificial person” ceases, its corporate powers are terminated “just as the powers of a natural person to take part in mundane affairs cease to exist upon his death”. There is nothing left but to conduct, as it were, the settlement of the estate of a deceased juridical person. From July 15 to October 28, 1963, when Alhambra made its attempt to extend its corporate existence, its original term of fifty years had already expired (January 15, 1962); it was in the midst of the three-year grace period statutorily fixed in Section 77 of the Corporation

additional fifty years, or a total of 100 years from its incorporation. On August 26, 1963, Alhambra’s stockholders, representing more than two-thirds of its subscribed capital stock, voted to approve the foregoing resolution. On October 28, 1963, Alhambra’s articles of incorporation as so amended certified correct by its president and secretary and a majority of its board of directors, were filed with respondent Securities and Exchange Commission (SEC). On November 18, 1963, SEC, however, returned said amended articles of incorporation to Alhambra’s counsel with the ruling that Republic Act 3531 “which took effect only on June 20, 1963, cannot be availed of by the said corporation, for the reason that its term of existence had already expired when the said law took effect in short, said law has no retroactive effect.”

Law, thus: . SEC. 77. Every corporation whose charter expires by its own limitation or is annulled by forfeiture or otherwise, or whose corporate existence for other purposes is terminated in any other manner, shall nevertheless be continued as a body corporate for three years after the time when it would have been so dissolved, for the purpose of prosecuting and defending suits by or against it and of enabling it gradually to settle and close its affairs, to dispose of and convey its property and to divide its capital stock, but not for the purpose of continuing the business for which it was established. Plain from the language of the provision is its meaning: continuance of a “dissolved” corporation as a body corporate for three years has for its purpose the final closure of its affairs, and no other; the corporation is specifically enjoined from “continuing the business for which it was established”. The liquidation of the corporation’s affairs set forth in Section 77 became necessary precisely because its life had ended. For this reason alone, the corporate existence and juridical personality of that corporation to do business may no longer be extended. Silence of the law on the matter is not hard to understand. Specificity is not really necessary. The authority to prolong corporate life was inserted by Republic Act 3531 into a section of the law that deals with the power of a corporation to amend its articles of incorporation. (For, the manner of prolongation is through an amendment of the articles.) And it should be clearly evident that under Section 77 no corporation in a state of liquidation can act in any way, much less amend its articles, “for the purpose of continuing the

business for which it was established”. All these dilute Alhambra’s position that it could revivify its corporate life simply because when it attempted to do so, Alhambra was still in the process of liquidation. It is surely impermissible for us to stretch the law — that merely empowers a corporation to act in liquidation — to inject therein the power to extend its corporate existence. The pari materia rule of statutory construction, in fact, commands that statutes must be harmonized with each other. So harmonizing, the conclusion is clear that Section 18 of the Corporation Law, as amended by Republic Act 3531 in reference to extensions of corporate existence, is to be read in the same light as Republic Act 1932. Which means that domestic corporations in general, as with domestic insurance companies, can extend corporate existence only on or before the expiration of the term fixed in their charters. Benguet Consolidated Mining Co. vs. Pineda 98 Phil. 711 , March 28, 1956

Benguet Consolidated Mining Company was organized in 1903 under the Spanish Code of Commerce of 1886 as a sociedad anonima. It was agreed by the incorporators that Benguet Mining was to exist for 50 years. In 1906, Act 1459 (Corporation Law) was enacted which superseded the Code of Commerce of 1886. Act 1459 essentially introduced the American concept of a corporation. The purpose of the law, among others, is to eradicate the Spanish Code and make sociedades anonimas obsolete. In 1953, the board of directors of Benguet Mining submitted to the Securities and Exchange Commission an application for them to be allowed to extend the life span of Benguet Mining . Then Commissioner Mariano Pineda denied the application as it ruled that the extension requested is contrary to Section 18 of the Corporation Law of 1906 which provides that the life of a corporation shall not be extended by amendment beyond the time fixed in their original articles. CONTENTION oF BENGUET: Benguet Mining contends that

Whether or not Benguet Mining is correct.

The prohibition contained in section 18 of Act No. 1459, against extending the period of corporate existence by amendment of the original articles, was intended to apply, and does apply, to sociedades anonimas, already formed, organized and existing at the time of the effectivity of the Corporation Law (Act 1459) in 1906. The aforesaid statutory prohibition is valid and impairs no vested rights or constitutional inhibition where no agreement to extend the original period of corporate life was perfected before the enactment of the Corporation Law. A sociedad anónima, existing before the Corporation Law, that continues to do business as such for a reasonable time after its enactment, is deemed to have made its election and may not subsequently claim to reform into a corporation under section 75 of Act No. 1459. Particularly should this be the case

they have a vested right under the Code of Commerce of where it has asserted its privileges as such 1886 because they were organized under said law; that sociedad anónima before invoking its alleged right under said law, Benguet Mining is allowed to extend its life to ref orm into a corporation. by simply amending its articles of incorporation; that the prohibition in Section 18 of the Corporation Code of 1906 does not apply to sociedades anonimas already existing prior to the Law’s enactment; that even assuming that the prohibition applies to Benguet Mining, it should be allowed to be reorganized as a corporation under the said Corporation Law. Formalities in Organizing Articles of Incorporation, Grounds for Disapproval (Section 17) Asuncion vs. De Yriate, 28 PHIL 67 (1914)

This is an action to obtain a writ of mandamus to compel the chief of the division of archives of the Executive Bureau to file certain articles of incorporation. The chief of the division of archives, the respondent, refused to file the articles of incorporation, hereinafter referred to, upon the ground that the object of the corporation, as stated in the articles, was not lawful and that, in pursuance of Section 6 of Act No. 1459, they were not registerable. It is strongly urged on the part of the appellants that the duties of the defendant are purely ministerial and that he has no authority to pass upon the lawfulness of the object for which the incorporators propose to organize.

whether or not the chief of the division of archives has authority, under the Corporation Law, on being presented with articles of incorporation for registration, to decide not only as to the sufficiency of the form of the articles, but also as to the lawfulness of the purposes of the proposed corporation.

What the law does not permit cannot be obtained by indirection. The object of the proposed corporation is clearly repugnant to the provisions of the Municipal Code and the governments of municipalities as they have been organized thereunder. ('Act No. 82, Philippine Commission.) The judgment appealed from is affirmed, with costs against appellants. D.; MUNICIPALITIES; ORGANIZATION OF BARRIO INTO SEPARATE CORPORATION.— When articles of incorporation presented for registration show that the object of incorporators is to organize a pueblo or barrio of a given municipality into a separate corporation for the purpose of taking possession and having control of all municipal property within the pueblo or barrio so incorporated, and administer it exclusively for the benefit of the residents of that pueblo or barrio, said articles of incorporation show upon their face that the object of the incorporation is unlawful in that it seeks to deprive the municipality in which the pueblo or barrio is situated of its property and its citizens of the right of enjoying the same and would, if permitted, disrupt and destroy the government of the municipalities of the Islands and abrogate the laws relating- to the formation and government of municipalities

LOYOLA GRAND VILLAS HOMEOWNERS (SOUTH) ASSOCIATION, INC., petitioner, vs. HON. COURT OF APPEALS, HOME INSURANCE AND GUARANTY CORPORATION, EMDEN ENCARNACION and HORATIO AYCARDO, respondents. G.R. No. 117188 August 7, 1997

Formalities in Organizing BY LAWS W/N LGVHAI's failure to Loyola Grand Villas Homeowners Association, Inc. (LGVHAI) file its by-laws within the was organized on 8 February 1983 as the homeoenwers' period prescribed by association for Loyola Grand Villas. It was also registered as Section 46 of the the sole homeowners' association in the said village with Corporation Code had the Home Financing Corporation (which eventually became the effect of Home Insurance Guarantee Corporation ["HIGC"]). automatically dissolving However, the association was not able file its corporate by- the said corporation. laws. The LGVHAI officers then tried to registered its By-Laws in 1988, but they failed to do so. They then discovered that there were two other homeowners' organizations within the subdivision - the Loyola Grand Villas Homeowners (North) Association, Inc. [North Association] and herein Petitioner Loyola Grand Villas Homeowners (South) Association, Inc.["South Association]. Upon inquiry by the LGVHAI to HIGC, it was discovered that LGVHAI was dissolved for its failure to submit its by-laws within the period required by the Corporation Code and for its non-user of corporate charter because HIGC had not received any report on the association's activities. These paved the way for the formation of the North and South Associations. LGVHAI then lodged a complaint with HIGC Hearing Officer Danilo Javier, and questioned the revocation of its registration. Hearing Officer Javier ruled in favor of LGVHAI, revoking the registration of the North and South Associations. Petitioner South Association appealed the ruling, contending that LGVHAI's failure to file its by-laws within the period prescribed by Section 46 of the Corporation Code effectively automatically dissolved the corporation. The Appeals Board of the HIGC and the Court of Appeals both rejected the contention of the Petitioner affirmed the decision of Hearing Officer Javier.

No. The pertinent provision of the Corporation Code that is the focal point of controversy in this case states: Sec. 46. Adoption of by-laws. - Every corporation formed under this Code, must within one (1) month after receipt of official notice of the issuance of its certificate of incorporation by the Securities and Exchange Commission, adopt a code of by-laws for its government not inconsistent with this Code. Ordinarily, the word "must" connotes an imposition of duty which must be enforced. However, the word "must" in a statute, like "shall," is not always imperative. It may be consistent with an ecercise of discretion. If the language of a statute, considered as a whole with due regard to its nature and object, reveals that the legislature intended to use the words "shall" and "must" to be directory, they should be given that meaning. The legislative deliberations of the Corporation Code reveals that it was not the intention of Congress to automatically dissolve a corporation for failure to file the By-Laws on time. Moreover, By-Laws may be necessary to govern the corporation, but By-Laws are still subordinate to the Articles of Incorporation and the Corporation Code. In fact, there are cases where By-Laws are unnecessary to the corporate existence and to the valid exercise of corporate powers. The Corporation Code does not expressly provide for the effects of non-filing of By-Laws. However, these have been rectified by Section 6 of PD 902-

A which provides that SEC shall possess the power to suspend or revoke, after proper notice and hearing, the franchise or certificate of registration of corporations upon failure to file ByLaws within the required period. This shows that there must be notice and hearing before a corporation is dissolved for failure to file its By-Laws. Even assuming that the existence of a ground, the penalty is not necessarily revocation, but may only be suspension.

PMI Colleges v. NLRC 277 Scra 462 (1997)

In 1991, PMI Colleges hired the services of Alejandro Galvan for the latter to teach in said institution. However, for unknown reasons, PMI defaulted from paying the remunerations due to Galvan. Galvan made demands but were ignored by PMI. Eventually, Galvan filed a labor case against PMI. Galvan got a favorable judgment from the Labor Arbiter; this was affirmed by the National Labor Relations Commission. On appeal, PMI reiterated, among others, that the employment of Galvan is void because it did not comply with its by-laws. Apparently, the by-laws require that an employment contract must be signed by the Chairman of the Board of PMI. PMI asserts that Galvan’s employment contract was not signed by the Chairman of the Board.

Whether or not Galvan’s employment contract is void.

Pena vs. CA, 193 SCRA 7117 (1991)

PAMPANGA BUS CO., INC. (PAMBUSCO) is the owner of the three lots in dispute. PAMBUSCO mortgaged the lots to the Development Bank of the Philippines (DBP), which were later on foreclosed.

W/N there Peña entitled to the lots.

Rosita Peña was awarded the lots in a foreclosure sale for being the highest bidder. The certificate of sale was later issued to her and registered in her name.

is

By-Laws are indispensable to corporations, since they are required by law for an orderly management of corporations. However, failure to file them within the period prescribed does not equate to the automatic dissolution of a corporation. No. PMI Colleges never even presented a copy of the by-laws to prove the existence of such provision. But even if it did, the employment contract cannot be rendered invalid just because it does not bear the signature of the Chairman of the Board of PMI. By-Laws operate merely as internal rules among the stockholders, they cannot affect or prejudice third persons who deal with the corporation, unless they have knowledge of the same. In this case, PMI was not able to prove that Galvan knew of said provision in the by-laws when he was employed by PMI.

Yes. The by-laws of a corporation are its own private laws which substantially have the same effect as the laws of the corporation. They are in effect, written, into the charter. In this sense they become part of the fundamental law of the corporation with which the corporation and its directors and officers must comply.

Subsequently, the Board of Directors of PAMBUSCO, through three out of its five directors, issued a resolution to assign its right of redemption over the lots in favor of any interested party. The right of redemption was later on assigned to Marcelino Enriquez, who redeemed the property. Enriquez then sold the lots to spouses Rising T. Yap and Catalina Lugue-Yap. Meanwhile, a case involving the validity of the sale to the spouses Yap was pending, and despite the protestations of Peña as to validity of the PAMBUSCO's assignment of the right of redemption, the lots were somehow registered in the name of spouses Yap. Despite the registration of the lots to spouses Yap, Peña retained possession of the property.

Main Case: Spouses Yap sought to recover the possession of the lots from Peña. The latter countered that she is now the legitimate owner of the subject lands for having purchased the same in a foreclosure proceeding instituted by the DBP against PAMBUSCO and no valid redemption having been effected within the period provided by law. The defense was that since the deed of assignment executed by PAMBUSCO in favor of Enriquez was void ab initio for being an ultra vires act of its board of directors and for being without any valuable consideration, it could not have had any legal effect. (It should be noted that the by-laws of PAMBUSCO provide that four out of five directors must be present in a special meeting of the board to constitute a quorum, and that the corporation has already ceased to operate.) CFI ruled in favor of Petitioner Peña, but the same was overturned by the CA.

Apparently, only three (3) out of five (5) members of the board of directors of respondent PAMBUSCO convened by virtue of a prior notice of a special meeting. There was no quorum to validly transact business since it is required under its bylaws that at least four (4) members must be present to constitute a quorum in a special meeting of the board of directors. Under Section 25 of the Corporation Code of the Philippines, the articles of incorporation or by-laws of the corporation may fix a greater number than the majority of the number of board members to constitute the quorum necessary for the valid transaction of business. Any number less than the number provided in the articles or by-laws therein cannot constitute a quorum and any act therein would not bind the corporation; all that the attending directors could do is to adjourn. Moreover, the records show that respondent PAMBUSCO ceased to operate for about 25 years prior to the board meeting. Being a dormant corporation for several years, it was highly irregular, for a group of three (3) individuals representing themselves to be the directors of respondent PAMBUSCO to pass a resolution disposing of the only remaining asset of the corporation in favor of a former corporate officer. As a matter of fact, the three (3) alleged directors who attended the special meeting on November 19, 1974 were not listed as directors of respondent PAMBUSCO in the latest general information sheet. Similarly, the latest list of stockholders of respondent PAMBUSCO on file with the SEC does not show that the said alleged directors were among the stockholders of respondent PAMBUSCO, in contravention of the rule requiring a director to own one (1) share in their to qualify as director of a corporation.

Further, under the Corporation Law, the sale or disposition of any and/or substantially all properties of the corporation requires, in addition to a proper board resolution, the affirmative votes of the stockholders holding at least two-thirds (2/3) of the voting power in the corporation in a meeting duly called for that purpose. This was not complied with in the case at bar. At the time of the passage of the questioned resolution, respondent PAMBUSCO was insolvent and its only remaining asset was its right of redemption over the subject properties. Since the disposition of said redemption right of respondent PAMBUSCO by virtue of the questioned resolution was not approved by the required number of stockholders, the said resolution, as well as the subsequent assignment and sale, were null and void. Lastly, for lack of consideration, the assignment should be construed as a donation. Under Article 725 of the Civil Code, in order to be valid, such a donation must be made in a public document and the acceptance must be made in the same or in a separate instrument. In the latter case, the donor shall be notified of the acceptance in an authentic form and such step must be noted in both instruments. Since assignment to Enriquez shows that there was no acceptance of the donation in the same and in a separate document, the said deed of assignment is thus void ab initio. RECOGNITION AND DISREGARD OF CORPORATENESS Separate Juridical Personality Santos vs. National Labor Relations Commission [GR 101699, 13 March 1996]

Melvin D. Millena, on 1 October 1985, was hired to be the project accountant for Mana Mining and Development Corporation's (MMDC) mining operations in Gatbo, Bacon, Sorsogon. On 12 August 1986, Millena sent to Mr. Gil Abaño, the MMDC corporate treasurer, a memorandum calling the latter's attention to the failure of the company to

Whether Santos should be made solidarily liable with MMDC.

A corporation is a judicial entity with legal personality separated and distinct from those acting for and in its behalf and, in general, from the people comprising it. The rule is that obligations incurred by the corporation, acting through its directors, officers and employees, are

comply with the withholding tax requirements of, and to make the corresponding monthly remittances to, the Bureau of Internal Revenue (BIR) on account of delayed payments of accrued salaries to the company's laborers and employees. In a letter, dated 8 September 1986, Abaño advised Millena that it was the board's decision that it stop porduction (operation) in Sorsogon due to the upcoming rainy seasons and the deterioration of the peace and order in the said area; that the corporation will undertake only necessary maintenance and repair work and will keep overhead down to the minimum manageable level; and that the corporation will not need a project accountant until the corporaton resumes full-scale operations. Millena expressed "shock" over the termination of his employment. He complained that he would not have resigned from the Sycip, Gores & Velayo accounting firm, where he was already a senior staff auditor, had it not been for the assurance of a "continuous job" by MMDC's Eng. Rodillano E. Velasquez. Millena requested that he be reimbursed the "advances" he had made for the company and be paid his "accrued salaries/claims." The claim was not heeded. On October 1986, Millena filed with the NLRC Regional Arbitration, Branch No. V, in Legazpi City, a complaint for illegal dismissal, unpaid salaries, 13th month pay, overtime pay, separation pay and incentive leave pay against MMDC and its two top officials, namely, Benjamin A Santos (the President) and Rodillano A. Velasquez (the executive vicepresident). In his complaint-affidavit (position paper), submitted on 27 October 1986, Millena alleged, among other things, that his dismissal was merely an offshoot of his letter of 12 August 1986 to Abaño about the company's inability to pay its workers and to remit withholding taxes to the BIR. On 27 July 1988, Labor Arbiter Fructouso T. Aurellano, finding no valid cause for terminating complaint's employment, ruledthat a partial closure of an establishment due to losses was a retrenchment measure that rendered the employer liable for unpaid salaries and other monetary claims. The Labor Arbiter ordered Santos, et. al. to pay Millena the amount of P37,132.25 corresponding to the latter's unpaid

its sole liabilities. Nevertheless, being a mere fiction of law, peculiar situations or valid grounds can exist to warrant, albeit done sparingly, the disregard of its independent being and the lifting of the corporate veil. As a rule, this situation might arise a corporation is used to evade a just and due obligation or to justify a wrong, to shield or perpetrate fraud, to carry out similar other unjustifiable aims or intentions, or as a subterfuge to commit injustice and so circumvent the law. Without necessarily piercing the veil of corporate fiction, personal civil liability can also be said to lawfully attach to a corporate director, trustee or officer; to wit: When (1) He assents (a) to a patently unlawful act of the corporation, or (b) for bad faith or gross negligence in directing its affairs, or (b) for conflict of interest, resulting in damages to the corporation, its stockholders or other persons; (2) He consents to the issuance of watered stocks or who, having knowledge thereof, does not forthwith file with the corporate secretary his written objection thereto; (3) He agrees to hold himself personally and solidarily liable with the corporation; or (4) He is made, by a specific provision of law, to personally answer for his corporate action. The case of Santos is way of these exceptional instances. It is not even shown that Santos has had a direct hand in the dismissal of Millena enough to attribute to Santos a patently unlawful act while acting for the corporation. Neither can Article 289 of the Labor Code be applied since this specifically refers only to the imposition of penalties under the Code. It is undisputed that the termination of Millena's employment has, instead, been due, collectively, to the need for a further mitigation of losses, the onset of the rainy season, the insurgency problem, in Sorsogon and the lack of funds to further support the mining operation in Gatbo. It is basic that a corporation is invested by law with a personally separate and distinct from those of the persons composing it as well as from that of

salaries and advances: P5,400.00 for petitioner's 13th month pay; P3,340.95 as service incentive leave pay; and P5, 400.00 as separation pay. Santos, et. al. were further ordered to pay Millena 10% of the monetary awards as attorney's fees. Alleging abuse of discretion by the Labor Arbiter, the company and its co-respondents filed a "motion for reconsideration and /or appeal." 8 The motion/appeal was forthwith indorsed to the Executive Director of the NLRC in Manila. In a resolution, dated 04 September 1989, the NLRC affirmed the decision of the Labor Arbiter. A writ of execution correspondingly issued; however, it was returned unsatisfied for the failure of the sheriff to locate the offices of the corporation in the addressed indicated. Another writ of execution and an order of garnishment was thereupon served on Santos at his residence. Contending that he had been denied due process, Santos filed a motion for reconsideration of the NLRC's resolution along with a prayer for the quashal of the writ of execution and order of garnishment. He averred that he had never received any notice, summons or even a copy of the complaint; hence, he said, the Labor Arbiter at no time had acquired jurisdiction over him. On 16 August 1991, the NLRC dismissed the motion for reconsideration. Santos filed the petition for certiorari. Stockholders of F. Guanzon and Sons, Inc v. Register of Deeds of Manila (1962)

Sept 19, 1960: 5 stockholders of F Guanzon executed a certificate of liquidation of the assets of the corporation. By virtue of a resolution dissolving the corporation, they wish to distribute as liquidated dividends among themselves and in proportion to their shareholdings, the assets of the corporation, which includes real estate properties in Manila. The Register of Deeds however, upon presentment of the certificate of liquidation by the 5 stockholders, denied registration of the properties to be distributed on 7 grounds, 3 of which were questioned by the stockholders: (1) no statement of the # of parcels of land to be distributed (2) registration fees iao P430.50 (3) doc stamp tax iao P940.45 (4) court judgment approving the dissolution and directing disposition of the assets. The stockholders claim that the certificate of liquidation merely partitions/distributes the corporate assets among them because the corporation has already been dissolved. Hence

any, other legal entity to which it may be related. Mere ownership by a single stockholder or by another corporation of all nearly all of the capital stock of a corporation is not of itself sufficient ground for disregarding the separate corporate personally. Similar to the case of Sunio vs. National Labor Relations Commission, Santos should not have been made personally answerable for the payment of Millena's back salaries.

W/N the certificate of liquidation involves a mere distribution of corporate assets or a transfer or conveyance of property.

It is a transfer/conveyance of property. A corporation is a juridical person separate and distinct from the stockholders. Properties registered in the name of the corporation are owned by it as a separate entity. The shares held by stockholders are their personal property and not the corporation, and it only typifies an aliquot part of the corporation’s property or the right to share in the proceeds. The holder of such share is not the owner of any part of the capital of the corporation, nor is he entitled to possession of any definite portion of its assets, neither is he a co-owner. Liquidation by stockholders after a corporation’s dissolution is not mere partitioning of community property, but already a conveyance or transfer of title to them from the corporation.

they need not comply with the requirements imposed by the Register of Deeds and the Land Registration Authority. The LRA counters that the distribution of the corporate assets upon dissolution of the corporation, is ultimately a transfer/conveyance of property to the stockholders.

Manila Gas vs. CIR, 62 PHIL 895

The plaintiff is a corporation organized under the laws of the Philippine Islands. It operates a gas plant in the City of Manila and furnishes gas service to the people of the metropolis and surrounding municipalities by virtue of a franchise granted to it by the Philippine Government. Associated with the plaintiff are the islands Gas and Electric Company domiciled in New York, United States, and the General Finance Company domiciled in Zurich, Switzerland. Neither of these last mentioned corporations is resident in the Philippines. For the years 1930, 1931, and 1932, dividends in the sum of P1, 348,847.50 were paid by the plaintiff to the Islands Gas and Electric Company in the capacity of stockholders upon which withholding income taxes were paid to the defendant totalling P40, 460.03 For the same years interest on bonds in the sum of P411, 600 was paid by the plaintiff to the Islands Gas and Electric Company upon which withholding income taxes were paid to the defendant totalling P12, 348. Finally for the stated time period, interest on other indebtedness in the sum of P131, 644:90 was paid by the plaintiff to the Islands Gas and Electric Company and the General Finance Company respectively upon which withholding income taxes were

The distribution of the corporate properties to the SHs was deemed not in the nature of a partition among co-owners, but rather a disposition by the corporation to the SHs as opposite parties to a contract Properties registered in the name of the corporation are owned by it as an entity separate and distinct from its members; shares of stock are personal property, and NOT corporate property share of stock typifies an aliquot part of the corporation’s property, or the right to share in the proceeds to that extent when distributed holder of shares is not the owner of any part of the capital of the corporation, nor is he entitled to the possession of any definite portion of its property or assets 1. Appellant first contends that the dividends paid by it to its stockholders, the Islands Gas and Electric Company, were not subject to tax because to impose a tax thereon would be to do so on the plaintiff corporation, in violation of the terms of its franchise and would, moreover, be oppressive and inequitable. 2. Appellant contends that, as the Islands Gas and Electric Company and the General Finance Company are domiciled in the United States and Switzerland respectively, and as the interest on

1. The trial judge was of the opinion that the instant case was governed by our previous decision in the case of Philippine Telephone and Telegraph Co. vs. Collector of Internal Revenue ([1933], 58 Phil., 639). In this view we concur. It is true that the tax exemption provision relating to the Manila Gas Corporation herein before quoted differs in phraseology from the tax exemption provision to be found in the franchise of the Telephone and Telegraph Company, but the ratio decideridi of the two cases is substantially the same. As there held and as now confirmed, a corporation has a personality distinct from that of its stockholders, enabling the taxing power to reach the latter when they receive dividends from the corporation. It must be considered as settled in this jurisdiction that dividends of a domestic corporation, which are paid and delivered in cash to foreign corporations as stockholders, are subject to the payment of the income tax, the exemption clause in the charter of the corporation notwithstanding.

paid to the defendant totalling P3, 949.34. Some uncertainty existing regarding the place of payment, we will not go into this factor of the case at this point, except to remark that the bonds and other tokens of indebtedness are not to be found in the record. However, Exhibits E, F, and G, certified correct by the treasurer of the Manila Gas Corporation, purport to prove that the place of payment was the United States and Switzerland.

the bonds and other indebtedness earned by said corporations has been paid in their respective domiciles, this is not income from Philippine sources within the meaning of the Philippine Income Tax Law. Citing sections 10 (a) and 13 (e) of Act No. 2833, the Income Tax Law,

For the foregoing reasons, we are led to sustain the decision of the trial court and to overrule appellant's first assigned error.

2. The Solicitor-General answers with the observation that the cited decisions interpreted the Income Tax Law before it was amended by Act No. 3761 to cover the interest on bonds and other obligations or securities paid "within or without the Philippine Islands." Appellant rebuts this argument by "assuming, for the sake of the argument, that by the amendment introduced to section 13 of Act No. 2833 by Act No. 3761 the Legislature intended that interest received by non-residents is to be considered income from Philip- pine sources and so is subject to tax," but with the necessary sequel that the amendatory statute is invalid and unconstitutional as being beyond the power of the Legislature to enact. Taking first under observation the last point, it is to be observed that neither in the pleadings, the decision of the trial court, nor the assignment of errors, was the question of the validity of Act No. 3761 raised. Under such circumstances, and no jurisdictional issue being involved, we do not feel that it is the duty of the court to pass on the constitutional question, and accordingly will refrain from doing so. (CadwalladerGibson Lumber Co. vs. Del Rosario [1913], 26 Phil,, 192; Macondray & Co. vs. Benito and Ocampo, p. 137, ante; State vs. Burke [1912], 175 Ala., 561.) As to the applicability of the local cases cited and of the Porto Rican case of Domenech vs. United Porto Rican Sugar Co. ([1932], 62 F. [2d], 552), we need only observe that these cases announced good law, but that each case must be decided on its particular facts. In other words, in

the opinion of the majority of the court, the facts at bar and the facts in those cases can be clearly differentiated. Also, in the case at bar there is some uncertainty concerning the place, of payment, which under one view could be considered the Philippines and under another view the United States and Switzerland, but which cannot be definitely determined without the necessary documentary evidence before us. The approved doctrine is that no state may tax anything not within its jurisdiction without violating the due process clause of the constitution. The taxing power of a state does not extend beyond its territorial limits, but within such limits it may tax persons, property, income, or business. If an interest in property is taxed, the situs of either the property or interest must be found within the state. If an income is taxed, the recipient thereof must have a domicile within the state or the property or business out of which the income issues must be situated within the state so that the income may be said to have a situs therein. Personal property may be separated from its owner, and he may be taxed on its account at the place where the property is although it is not the place of his own domicile and even though he is not a citizen or resident of the state which imposes the tax. But debts owing by corporations are obligations of the debtors, and only possess value in the hands of the creditors. (Farmers Loan Co. vs. Minnesota [1930], 280 U. S., 204; Union Refrigerator Transit Co. vs. Kentucky [1905], 199 U.. S., 194; State Tax on Foreign- held Bonds [1873], 15 Wall, 300; Buck vs. Beach [1907], 206 U. S., 392; State ex rel. Manitowoc Gas Go. vs. Wis. Tax Comm. [1915], 161 Wis., Ill; United States Revenue Act of 1932, sec. 143.) These views concerning situs for taxation

purposes apply as well to an organized, unincorporated territory or to & Commonwealth having the status of the Philippines. Pushing to one side that portion of Act No. 8761 which permits taxation of interest on bonds and* other indebtedness paid without the Philippine Islands, the question is if the income was derived from sources within the Philippine Islands. In the judgment of the majority of the court, the question should be answered in the affirmative. The Manila Gas Corporation operates its business entirely within the Philippines. Its earnings, therefore, come from local sources. The place of material delivery of the interest to the foreign corporations paid out of the revenue of the domestic corporation is of no particular moment. The place of payment even if conceded to be outside of the country cannot alter the fact that the income was derived from the Philippines. The word "source" conveys only one idea, that of origin, and the origin of the income was the Philippines. In synthesis, therefore, we hold that conditions have not been provided which justify the court in passing on the constitutional question suggested; that the facts while somewhat obscure differ from the facts to be found in the cases relied upon, and that the Collector of Internal Revenue was justified in withholding income taxes on interest on bonds and other indebtedness paid to nonresident corporations because this income was received from sources within the Philippine Islands as authorized by the Income Tax Law. For the foregoing reasons, the second assigned error will be overruled. Before concluding, it is but fair to state that the writer's opinion on the first subject and the first

assigned error herein discussed is accurately set forth, but that his opinion on the second subject and the second assigned error is not accurately reflected, because on this last division his views coincide with those of the appellant. However, in the interest of the prompt disposition of this case, the decision has been written up in accordance with instructions received from the court. Judgment affirmed, with the costs of this instance assessed against the appellant. Magsaysay-Labrador vs. CA (1989) 180 SCRA 266

Adelaida Rodriguez-Magsaysay was the widow of Senator Genaro Magsaysay and the special administratix of his estate brought before the CFI of Olongapo in an action against Artemio Panganiban, Subic Land Corporation (SUBIC), Filipinas Manufacturer’s Bank (FILIMANBANK) and the Register of Deeds of Zambales. She alleged in her complaint that she and her husband acquired thru conjugal funds a parcel of land with improvements known as “Pequena Island” covered by TCT No. 3258. After the death of her husband, she discovered annotations at the back of the TCT. The first was that the land was acquired by her husband from his separate capital. Second was that there was also a registration of a Deed of Assignment in 1976 purportedly executed by the late Senator in favor of SUBIC. As a result, TCT 3258 was cancelled and TCT 22431 was issued in the name of SUBIC. And the third was a registration of Deed of Mortgage dated 1977 in the amount of P2.7M was executed by SUBIC in favor of FILMANBANK. Adelaida contended that the acts were void and done in an attempt to defraud the conjugal partnership considering that the land is conjugal. Her marital consent was not obtained to the annotation in the TCT3258. She also contended that the Register of Deeds changed the title without the approval of the Commissioner of Land Registration and that the late Senator did not gave his consent to the Deed of Assignment, and if he did, it was thru mistake, violence and intimidation. She further alleged that the assignment in favor of SUBIC was without consideration and consequently null and void.

Whether or petitioners stockholders has right to intervene.

not as the

No. The interest which entitles a person to intervene in a suit between other parties must be in the matter in litigation and of such direct and immediate character that the intervenor will either gain or lose by the direct legal operation and effect of judgment. Here, the interest of the petitioners, if exists at all, is indirect, contingent, remote, conjectural, consequential and collateral. Their interest is purely inchoate or in sheer expectancy of a right in the management of the corporation and to share in the profits thereof and in the properties and assets thereof on dissolution, after payment of the corporate debts and obligations. While a share of stock represents a proportionate or aliquot interest in the property of the corporation, it does not vest the owner thereof with any legal right or title to any of the property, his interest in the corporate property being equitable or beneficial in nature. Shareholders are in no legal sense the owners of corporate property, which is owned by the corporation as a distinct legal person. It was found also that no transfer transpired between petitioners and the late Senator. The transfer must be registered in the books of the corporation to affect third persons. And even assuming the arguendo that there was a valid transfer, petitioners are nonetheless barred from intervening inasmuch as their rights can be

Herein petitioners are the sisters of the late senator and filed a motion for intervention on the ground that their brother conveyed to them half of his shareholdings in SUBIC and as assignees of around 41% of the total outstanding shares of stocks, they have a substantial and legal interest in the subject matter of litigation and the success of the suit with respect to SUBIC. The court denied the motion contending that they had no legal interest and that SUBIC has a personality separate and distinct from its stockholders. On appeal, the CA affirmed the findings of the lower court, stating further that whatever claims the petitioner have against the late Senator or against SUBIC can be ventilated in a separate proceeding. Good Earth vs. CA, 194 SCRA 544 (1991)

A lease contract was entered into between ROCES and Good Earth Emporioum (GEE). A five-storey building was the subject of the said contract, which upon failure of the latter to pay its rentals, ROCES filed an ejectment case against the petitioner. The MTC of Manila rendered a decision ordering GEE and all persons under him to vacate the premises and surrender the same to ROCES and pay the plaintiffs the rental. GEE filed a motion to quash the writ of execution but the same was denied by the MTC for lack of merit. In 1987 the RTC of Manila reversed the decision of the MTC finding that the amount of P1 million evidenced by Exhibit "I" and another P1 million evidenced by the pacto de retro sale instrument were in full satisfaction of the judgment obligation. On further appeal, the CA reversed the decision of the RTC and reinstated the Resolution of the MTC of Manila. GEE’s m/r was denied, hence this petition.

ventilated and proceeding.

amply

protected

in

another

Doctrine: Shareholders are in no legal sense the owners of corporate property, which is owned by the corporation as a distinct legal person.

Whether or not there was full satisfaction of the judgment debt in favor of respondent corporation.

There is no indication in the receipt, that it was in payment, full or partial, of the judgment obligation. Likewise, there is no indication in the pacto de retro sale which was drawn in favor of Jesus Marcos Roces and Marcos V. Roces and not the respondent corporation, that the obligation embodied therein had something to do with petitioners' judgment obligation with respondent corporation. Article 1240 of the Civil Code of the Philippines provides that: “Payment shall be made to the person in whose favor the obligation has been constituted, or his successor in interest, or any person authorized to receive it.” In the case at bar, the supposed payments were not made to Roces-Reyes Realty, Inc. or to its successor in interest nor is there positive evidence that the payment was made to a person authorized to receive it. No such proof was submitted but merely inferred by the Regional Trial Court from Marcos Roces having signed the Lease Contract as President which was witnessed by Jesus Marcos Roces. On the other hand, Jesus Marcos Roces testified that the amount of P1 million evidenced by the receipt is the payment for a loan extended by him and Marcos Roces in favor of Lim Ka Ping. The assertion is home by the

receipt itself whereby they acknowledged payment of the loan in their names and in no other capacity. A corporation has a personality distinct and separate from its individual stockholders or members. As a consequence of the separate juridical personality of a corporation, the corporate debt or credit is not the debt or credit of the stockholder, nor is the stockholder’s debt or credit that of the corporation. The fact that at the time payment was made to the two Roces brothers, GEE was also indebted to respondent corporation for a larger amount, is not supportive of the Regional Trial Court's conclusions that the payment was in favor of the latter, especially in the case at bar where the amount was not receipted for by respondent corporation and there is absolutely no indication in the receipt from which it can be reasonably inferred, that said payment was in satisfaction of the judgment debt. Likewise, no such inference can be made from the execution of the pacto de retro sale which was not made in favor of respondent corporation but in favor of the two Roces brothers in their individual capacities without any reference to the judgment obligation in favor of respondent corporation. DOCTRINE: A corporation has a personality distinct and separate from its individual stockholders or members. Being an officer or stockholder of a corporation does not make one’s property also of the corporation, and vice-versa, for they are separate entities. Shareowners are in no legal sense the owners of corporate property (or credits) which is owned by the corporation as a distinct legal person.

Defective Incorporation.

De facto (Section 20 of the Corporation Code) Rationale for the doctrine FELIPE TAYKO vs. NICOLAS CAPISTRANO G.R. No. L-30188, October 2, 1928

This is a petition for a writ of prohibition enjoining the respondent judge from making cognizance of certain civil and criminal election cases in which the petitioners are parties. The petitioners allege that Capistrano was appointed judge of the CFI of Oriental Negros, to hold office during good behavior and until he should reach the age of 65 years; that he now has reached that age and, therefore, under the provisions of section 148 of the Administrative Code as amended, is disqualified from acting as a judge of the Court of First Instance. The petitioners further allege that in view of the many election protests and criminal cases for violation of the election law filed in the CFI of Oriental Negros arising in the from the last election, de la Costa was duly designated and acted as auxiliary judge. There was an understanding that de la Costa would hear and take cognizance of all election protests and criminal actions then pending or to filed arising from the said last general election, and that Capistrano would try and hear the ordinary cases pending. Notwithstanding the understanding, Capistrano tried and is still trying to take cognizance of the election protests an criminal actions in said court; declared in open court that he will try the criminal cases for the reason that de la Costa refused to try the same on the ground that the preliminary investigations were held before him, when, in truth and in fact, the d la Costa did not make the statement imputed to him and was and is still willing to try the election protests and criminal cases for violation of the election law pending in the court. Additionally that Capistrano, in spite of the fact that he was holding and is now pretending to hold the office of judge took great interest and active part in the filing of criminal charges against the petitioners to the unjustifiable extent of appointing a deputy fiscal, who then filed the proper informations, when the provincial fiscal refused to file criminal charges against the petitioners for violation of the

Whether or not Capistrano, upon reaching the age of 65, can still continue public office? Is he considered a de facto judge?

Briefly defined, a de facto judge is one who exercises the duties of a judicial office under color of an appointment or election thereto (Brown vs. O'Connell, 36 Conn., 432). He differs, on the one hand, from a mere usurper who undertakes to act officially without any color of right, and on the other hand, from a judge de jure who is in all respects legally appointed and qualified and whose term of office has not expired (State vs. Carroll, 38 Conn., 449; Denny vs. Matton, 2 Allen [Mass.], 361; Van Slyke vs. Farmers' Mut. Fire Ins. Co., 39 Wis., 390). Apart from any constitutional or statutory regulation on the subject there seems to be a general rule of law that an incumbent of an office will hold over after the conclusion of his term until the elction and qualification of a successor (22 R. C. L., pp. 554-5). When a judge in good faith remains in office after his title has ended, he is a de facto officer (Sheehan's Case, 122 Mass., 445). Applying the principles stated to the facts set forth in the petition before us, we cannot escape the conclusion that, on the assumption that said facts are true, the respondent judge must be considered a judge de facto. His term of office may have expired, but his successor has not been appointed, and as good faith is presumed, he must be regarded as holding over in good faith. The contention of counsel for the petitioners that the auxiliary judge present in the district must be considered the regular judge seems obviously erroneous. Accordingly, it is a well established principle, dating from the earliest period and repeatedly confirmed by an unbroken current of decisions, that the official acts of a de facto judge are just as valid for all purposes as those of a de

jure judge, so far as the public or third persons

election law for lack of sufficient evidence to sustain the same

who are interested therein are concerned. The principle is one founded in policy and convenience, for the right of no one claiming a title or interest under or through the proceedings of an officer having an apparent authority to act would be safe, if it were necessary in every case to examine the legality of the title of such officer up to its original source, and the title or interest of such person were held to be invalidated by some accidental defect or flaw in the appointment, election or qualification of such officer, or in the rights of those from whom his appointment or election emanated; nor could the supremacy of the laws be maintained, or their execution enforced, if the acts of the judge having a colorable, but not a legal title, were to be deemed invalid. As in the case of judges of courts of record, the acts of a justice de facto cannot be called in question in any suit to which he is not a party. Petition is sustained.

Finally, that Capistrano is neither a judge de jure nor de facto, but that he continues to hold the office of judge and pretends to be duly qualified and acting judge of the said province; and that he has tried, and continues to try, to act as such judge. Hence this petition.

Defective Incorporation. De facto (Section 20 of the Corporation Code) REQUISITES G.R. No. L-21114 November 28, 1967 FEDERICO FERNANDEZ, plaintiff-appellant, vs. P. CUERVA and CO., defendant-appellee.

Federico Fernandez was employed as salesman by P. Cuerva & Co. from March, 1949 to October, 1959. After his separation from the service, he filed a claim against P. Cuerva before the Regional Office of DOLE to recover unpaid salaries and commissions, and separation pay (July 26, 1960). During the pendency of said case, Fernandez again instituted a similar complaint with the CFI (December 17, 1962) alleging, among others, that the agreed upon increase of P100.00 a month in his salary (P200 -> P300) and 10% commission were not actually received by him. Apprently, there was a verbal understanding between him and P. Cuerva that the same would be retained by the latter as bond or deposit for the goods being handled by Fernandez. All in all, the total sum sought to be recovered by Fernandez is P11,570.00.

Did the filing by plaintiff of that claim with the DOLE RO suspend the running of the period of prescription?

YES It is true that the claim filed by plaintiff with the regional office of the Department of Labor is not a judicial demand in the same sense of the term "judicial demand" because the same was not instituted in a court of justice. However, pursuant to Reorganization Plan No. 20-A, each regional office of the Department of Labor was vested with original and exclusive jurisdiction over all cases affecting all money claims arising from violations of labor standards on working conditions to the exclusion of courts. Consequently, when Fernandez wanted to enforce his claim after his dismissal from the service, RO 4 of the DOLE was empowered to take cognizance of the claim. He could not institute the action to recover his claim in the court of

CFI dismissed the case mainly on the ground of prescription. Under the Minimum Wage Law, the action to recover the withheld commissions was already barred inasmuch as it was not brought within 3 years from the time the right of action accrued (Sec. 17). On his part, Fernandez mainly argue that the fact that his having filed a similar claim with Regional Office No. 4 of the Department of Labor had suspended the running of the prescriptive period insofar as his claim for refund of unauthorized deductions and withheld commissions was concerned. To this, P. Cuerva contends that inasmuch as plaintiff's claim was lodged with the regional office of the Department of Labor, which is not a court, the same could not be considered a judicial demand that would suspend the running of the prescriptive period.

justice. We believe that the filing of a claim before an administrative agency which is vested with authority to decide said claim would produce the effect of a judicial demand for the purpose of interrupting the running of the period of prescription. The purpose of the law on prescription and the statute of limitations is to protect the person who is diligent and vigilant in asserting his right, and conversely to punish the person who sleeps on his right. NOTE: While the said Sec. 25 of the reorganization plan was declared unconstitutional, Fernandez had filed his claim about one year before said declaration of unconstitutionality. The actual existence of a statute prior to such declaration is an operative fact, and may have consequences which cannot justly be ignored. It was precisely because Section 25 of Reorganization Plan No. 20-A was declared unconstitutional that Fernandez, without awaiting the action of Regional Office No. 4 of the Department of Labor on the claim that he filed on July 26, 1960, instituted his action in the CFI. The move of plaintiff was precisely intended to protect his right of action from the adverse effect of the decision of this Court. Considering that the running of prescriptive period was deemed interrupted, it is clear that Fernandez's action to enforce his claim was not yet barred by the statute of limitations when he filed his complaint in the CFI. Plaintiff's action may be considered as brought before the court still within the period of three years from the time his right of action accrued in accordance with the provisions of theMinimum Wage Law. And considering further that the amount sought to be recovered in the complaint is more than P10,000.00 (NOTE: P10,070.00), it follows that the CFI has the exclusive and original jurisdiction to entertain the action. NOTE: law applicable is the Minimum Wage Law

HALL v PICCIO 86 Phil 603, GR No L2598, June 29, 1950

On May 28, 1947, petitioners C. Arnold Hall and Bradley P. Hall, and respondents Fred Brown, Emma Brown, Hipolita D. Chapman and Ceferino S. Abella, signed and acknowledged in Leyte, the article of incorporation of the Far Eastern Lumber and Commercial Co., Inc., organized to engage in a general lumber business to carry on as general contractors, operators and managers, . Attached to the article was an affidavit of the treasurer stating that 23,428 shares of stock had been subscribed and fully paid with certain properties transferred to the corporation. The said articles of incorporation was filed in the office of SEC. Pending action of the articles of incorporation by SEC, the

W/N the court had jurisdiction to decree the dissolution of the company, because it being a de facto corporation, dissolution thereof may only be ordered in a quo warranto proceeding instituted in accordance with section 19 of the Corporation Law.

because the bond or deposit sought to be recovered by Fernandez was actually the sum total of the unauthorized deductions from his salaries and withheld commissions under Section 10 thereof. NOTE: A right of action accrues only from the moment the right to commence the action comes into existence, and prescription begins to run from that time. NOTE: while the amounts withheld by P. Cuerva were actually deductions from plaintiff's salaries and unpaid commissions, they were, however, constituted as a bond or a deposit to answer for any liability that he might incur in connection with the goods handled by him. It was, therefore, not feasible for Fernandez to demand every month or every payday, or during the period of his employment the return or refund of those amounts withheld, because the undertaking for which the bond or deposit was constituted was still subsisting. And so the right of plaintiff to commence an action for the return or refund of the amounts representing such bond or deposit would accrue only when the same was no longer needed, and the time when it was no longer needed only came in October 1959 when plaintiff was separated from the service. Having ceased to be employed by the defendant, the bond put up by plaintiff thereby became unnecessary or useless. Yes, the court has jurisdiction to take cognizance of the case! Section 20 of the Corporation Law does not apply in this situation First, not having obtained the certificate of incorporation, the Far Eastern Lumber and Commercial Co. — even its stockholders — may not probably claim "in good faith" to be a corporation. (Under our statue it is to be noted (Corporation Law, sec. 11) that it is the issuance of

respondents filed a civil case against the petitioners alleging that Far Eastern Lumber and Commercial Co was an unregistered partnership and that they wished it dissolved because of bitter dissension among the members, mismanagement and fraud by the managers and heavy financial losses. The court (thru Judge Piccio) ordered the dissolution of the company. Halls offered to file a counter bond for the discharge of the receiver but the judge refused to accept the offer and discharge the receiver.

a certificate of incorporation by the Director of the Bureau of Commerce and Industry which calls a corporation into being. The immunity if collateral attack is granted to corporations "claiming in good faith to be a corporation under this act." Such a claim is compatible with the existence of errors and irregularities; but not with a total or substantial disregard of the law. Unless there has been an evident attempt to comply with the law the claim to be a corporation "under this act" could not be made "in good faith." ) Second, this is not a suit in which the corporation is a party. This is a litigation between stockholders of the alleged corporation, for the purpose of obtaining its dissolution. Even the existence of a de jure corporation may be terminated in a private suit for its dissolution between stockholders, without the intervention of the state. SECTION 20 Sec. 20. De facto corporations. The due incorporation of any corporation claiming in good faith to be a corporation under this Code, and its right to exercise corporate powers, shall not be inquired into collaterally in any private suit to which such corporation may be a party. Such inquiry may be made by the Solicitor General in a quo warranto proceeding.

Benguet Consolidated Mining Co. vs. Pineda 98 Phil. 711 , March 28, 1956

Benguet Consolidated Mining Company was organized in 1903 under the Spanish Code of Commerce of 1886 as a sociedad anonima. It was agreed by the incorporators that Benguet Mining was to exist for 50 years. In 1906, Act 1459 (Corporation Law) was enacted which superseded the Code of Commerce of 1886. Act 1459 essentially introduced the American concept of a corporation. The purpose of the law, among others, is to eradicate the Spanish Code and make sociedades anonimas obsolete. In 1953, the board of directors of Benguet Mining submitted to the Securities and Exchange Commission an application for them to be

Whether or not Benguet Mining is correct.

The prohibition contained in section 18 of Act No. 1459, against extending the period of corporate existence by amendment of the original articles, was intended to apply, and does apply, to sociedades anonimas, already formed, organized and existing at the time of the effectivity of the Corporation Law (Act 1459) in 1906. The aforesaid statutory prohibition is valid and impairs no vested rights or constitutional inhibition where no agreement to extend the original period of corporate life was perfected before the enactment

Asia Banking Corporation v. Standard Products Co. (1924)

allowed to extend the life span of Benguet Mining . Then Commissioner Mariano Pineda denied the application as it ruled that the extension requested is contrary to Section 18 of the Corporation Law of 1906 which provides that the life of a corporation shall not be extended by amendment beyond the time fixed in their original articles. CONTENTION oF BENGUET: Benguet Mining contends that they have a vested right under the Code of Commerce of 1886 because they were organized under said law; that under said law, Benguet Mining is allowed to extend its life by simply amending its articles of incorporation; that the prohibition in Section 18 of the Corporation Code of 1906 does not apply to sociedades anonimas already existing prior to the Law’s enactment; that even assuming that the prohibition applies to Benguet Mining, it should be allowed to be reorganized as a corporation under the said Corporation Law. Corporation by estoppel (Sec 21 of the Corporation Code) Rationale for the doctrine

of the Corporation Law. A sociedad anónima, existing before the Corporation Law, that continues to do business as such for a reasonable time after its enactment, is deemed to have made its election and may not subsequently claim to reform into a corporation under section 75 of Act No. 1459. Particularly should this be the case where it has asserted its privileges as such sociedad anónima before invoking its alleged right to ref orm into a corporation.

1.

-

2.

The Stadard Products Co., Inc executed a promissory note through its president George H. Seaver in favour of Asia Banking Corporation or order. This is an action brought by Asia Banking Corporation to recover the sum of P24, 736.47 which is the balance due on such note.

Lower Court: Rendered judgment in favour of plaintiff for the sum demanded in the complaint, with interest on the sum of P24, 147.34 from November 1, 1923 at the rate of 10% per annum, and the costs. From this judgment, the defendant appeals to this court. At the trial, plaintiff failed to prove affirmatively the corporate existence of the parties, and Standard Products Co. argues that the lower court erred in finding that the parties were corporations with judicial personality. Doctrine: The general rule is that in the absence of fraud, a person who has contracted or otherwise dealt with an association in such a way as to recognize and in effect admit its legal existence as a corporate body is thereby

WON the judgment of the lower court is a reversible error despite Asia Baking’s failure to present evidence of its corporate existence (NO)

-

-

The general rule is that in the absence of fraud, a person who has contracted or otherwise dealt with an association in such a way as to recognize and in effect admit its legal existence as a corporate body is thereby estopped to deny its corporate existence in any action leading out of or involving such contract or dealing, unless its existence is attacked for causes which have arisen since making the contract or other dealing relied on as an estoppel. This doctrine applies to domestic as well as foreign corporations. Since Standard Products has recognized the corporate existence of Asia Banking by making a promissory note in its favour and making partial payments on the same, it is estopped from denying Asia Banking’s corporate existence. It is also denied from denying its own corporate existence. Under these circumstances, it was

estopped to deny its corporate existence in any action leading out of or involving such contract or dealing, unless its existence is attacked for causes which have arisen since making the contract or other dealing relied on as an estoppel.

Vda. De Salvatierra v. Hon. Garlitos and Refuerzo (1958) – Felix, J.

Vda. De Salvatierra owned a parcel of land in Leyte. She entered into a contract of lease with the Philippine Fibers Producers Co., Inc., allegedly a corporation “duly organized and represented by Segundino Refuerzo, the President.”Several obligations were provided in the contract, but these obligations were not complied with because Salvatierra filed with the CFI of Leyte a complaint against Philippine Fibers and Refuerzo for accounting, rescission and damages. She alleged that the defendants planted kenaf on 3 hectares of the leased property, and refused to render an accounting of income derived and to deliver her share. Such were in violation of the terms of their covenant. The defendants were declared in default for failure to file an answer. The lower Court granted Salvatierra’s prayer. The Court, upon motion of Salvatierra, issued a writ of execution. The Provincial Sheriff caused the attachment of 3 parcels of land in the name of Refuerzo. No property of the Philippine Fibers was found available for attachment. Refuerzo later filed a motion claiming that the decision was null and void with respect to him, there being no allegation in the complaint pointing to his personal liability. He prayed than an order be issued limiting such liability to the corporation. The lower Court granted it and ordered the sheriff to release all properties belonging to Refuerzo that might have already been attached. Salvatierra’s petition for relief was denied. She then instituted this petition for certiorari asking to nullify the order of the judge. Doctrines: While as a general rule a person who has contracted or dealt with an association in such a way as to recognize its existence as a corporate body is estopped from denying the same in an action arising out of such transaction or dealing, yet this doctrine may not be held to be applicable where fraud takes part in the said transaction.

unnecessary for Asia Banking to present other evidence of the corporate existence of either of the parties. DISPOSITIVE: Judgment appealed affirmed, with costs against appellant. 1.

2.

WON Refuerzo must be exonerated from any liability for the non-fulfillment of obligation imposed on the corporation. (NO) WON Refuerzo’s motion was filed within the prescriptive period. (NO)

from

is

RATIO: 1. While as a general rule a person who has contracted or dealt with an association in such a way as to recognize its existence as a corporate body is estopped from denying the same in an action arising out of such transaction or dealing, yet this doctrine may not be held to be applicable where fraud takes part in the said transaction. On plaintiff’s charge that she was unaware of the fact that Philippine Fibers had no juridical personality, Refuerzo gave no confirmation or denial and the circumstances surrounding the execution of the contract lead to the inescapable conclusion that Salvatierra was really made to believe that such corporation was duly organized in accordance with law. A corporation when registered has a juridical personality separate and distinct from its component members or stockholders and officers such that a corporation cannot be held liable for the personal indebtedness of a stockholder even if he should be its president and conversely, a stockholder or member cannot be held personally liable for any financial obligation by the corporation in excess of his unpaid subscription. o But this rule refers merely to registered corporations and cannot be made applicable to the liability of members of an unincorporated association. o Reason: since an organization which before the law is non-existent ahs no personality and would be incompetent to act and appropriate for itself the powers

A corporation when registered has a juridical personality separate and distinct from its component members or stockholders and officers such that a corporation cannot be held liable for the personal indebtedness of a stockholder even if he should be its president and conversely, a stockholder or member cannot be held personally liable for any financial obligation by the corporation in excess of his unpaid subscription. But this rule refers merely to registered corporations and cannot be made applicable to the liability of members of an unincorporated association.

-

and attribute of a corporation as provided by law; it cannot create agents or confer authority on another to act in its behalf. Thus, those who act or purport to act as its representatives or agents do so without authority and at their own risk o A person who acts as an agent without authority or without a principal is himself regarded as the principal, possessed of all the rights and subject to all the liabilities of a principal. Thus a person acting or purporting to act on behalf of a corporation which has no valid existence assumes such privileges and obligations and becomes personally liable for contracts entered into or for other acts performed as such agent. Refuerzo, as president of the unregistered corporation was the moving spirit behind the consummation of the lease agreement by acting as its representative, his liability cannot be limited or restricted to that imposed upon corporate shareholders. In acting on behalf of a corporation which he knew to be unregistered, he assumed the risk of reaping the consequential damages or resultant rights arising out of such transaction.

2. The decision was rendered June 8, 1955 whereas the motion filed by Refuerzo was dated January 31, 1956, or after the lapse of 7 months and 23 days, which is beyond the prescriptive period provided by Rule 38. Rule 38, Sec. 3. When petition filed; contents and verification. – “A petition provided for in either of the preceding sections of this rule must be verified, filed within sixty days after the petitioner learns of the judgment, order or other proceeding to be set aside, and not more than six months after such judgment or order was entered, or such proceeding was taken…”

-

-

The provision treats of 2 periods i.e. 60 days after petitioner learns of the judgment, and not more than 6 months after judgment or order was rendered, both of which must be satisfied. The nature of the relief and the purpose behind it, the periods fixed by the rule are notextendible and never interrupted; nor could it be subjected to any condition or contingency because it is of itself devised to meet a condition or contingency.

DISPOSITIVE: Order set aside and nullified. G.R. No. LJanuary 30, 1965 MARIANO A. ALBERT, plaintiffappellant, vs. UNIVERSITY PUBLISHING CO., INC., defendantappellee. Uy & Artiaga and Antonio M. Molina for plaintiffappellant. Aruego, Mamaril & Associates for defendantappellees. 19118

The University Publishing Co. Inc. through its President Jose Aruego entered into a contract with Mariano Albert whereby the corporation agreed to pay a certain sum in installments for the exclusive right to publish his revised commentaries in the RPC and for his share in the previous sale of the book’s first edit edition. The corporation failed to pay the second installment thereby making the whole amount due and demandable (i.e. there was an acceleration clause). Albert then sued the corporation. The lower court rendered judgment in favor of Albert and a writ of execution was issued against the corporation. Albert however, petitioned for a writ of execution against Aruego, as the real defendant, stating that there is no such entity as University Publishing Co. Inc. Albert annexed to his petition a certification from the SEC saying that their records contain no such registered corporation. The corporation countered by saying that Aruego is not a party to this case and that, therefore, Albert’s petition should be denied. The corporation countered by saying that Aruego is not a party to this case, and that therefore, Albert’s petition should be denied. The corporation, actually did not want Aruego to be declared a party to the present case is because there would be no need to institute a separate action against Aruego to be declared a party to the present case is because there would then be a need to institute a separate

Can the corporation invoke the doctrine of estoppel?

1.) The corporation cannot invoke the doctrine of estoppel. The fact of non-registration of the corporation has not been disputed because the corporation only raised the point that it and not Aruego is the party defendant thereby assuming that the corporation is an existing corporation with an independent juridical personality. HOWEVER, precisely on account of non- registration, it cannot be considered a corporation not even a corporation de facto. It has therefore no personality separate from Aruego; it cannot be sued independently. The estoppel doctrine has not been invoked and even if it had been, it is not applicable to the case at bar: (a) Aruego had represented a non-existing entity and induced not only Albert but also the court to believe in such representation (b) He signed the contract as president of the corporation stating that this was a corporation duly organized and existing under the laws of the Philippines. One who induced another to act upon his willful misrepresentation that a corporation was duly organized and existing under the law, cannot thereafter set up against his victim the principle of corporation by estoppel. 2.) Aruego is the real defendant as he had control over the proceedings. Had Aruego been named as

action against Aruego; and if this is done, Aruego can set up the defense of prescription under the Statute of Limitations.

party defendant instead of or together with the corporation, there would be no room for debate as to his personal liability. Since he was not so named, matters of due process have arisen. Parties to a suit are persons who have a right to control the proceedings, to make defense, to adduce and crossexamine witnesses and to appeal from a decision. In the case at bar, Aruego, was and in reality, the one who answered and litigated through his own firm as counsel. He was in fact, if not on name, the defendant. Clearly then Aruego had his day in court as the real defendant and due process of law has been substantially observed. 3.) Aruego is the real party in interest because he reaped the benefits from the contract.

Lim vs. Philippine Fishing Gear Industries Inc. [GR 136448, 3 November 1999]

Lim Tong Lim requested Peter Yao and Antonio Chuato engage in commercial fishing with him. The three agreed to purchase two fishing boats but since they do not have the money they borrowed from one Jesus Lim the brother of Lim Tong Lim. Subsequently, they again borrowed money for the purchase of fishing nets and other fishing equipments. Yao and Chua represented themselves as acting in behalf of “Ocean Quest Fishing Corporation” (OQFC) and they contracted with Philippine Fishing Gear Industries (PFGI) for the purchase of fishing nets amounting to more than P500k. However, they were unable to pay PFGI and hence were sued in their own names as Ocean Quest Fishing Corporation is a non-existent corporation. Chua admitted his liability while Lim Tong Lim refused such liability alleging that Chua and Yao acted without his knowledge and consent in representing themselves as a corporation.

Whether Lim Tong Lim is liable as a partner

Yes. It is apparent from the factual milieu that the three decided to engage in a fishing business. Moreover, their Compromise Agreement had revealed their intention to pay the loan with the proceeds of the sale and to divide equally among them the excess or loss. The boats and equipment used for their business entails their common fund. The contribution to such fund need not be cash or fixed assets; it could be an intangible like credit or industry. That the parties agreed that any loss or profit from the sale and operation of the boats would be divided equally among them also shows that they had indeed formed a partnership. The principle of corporation by estoppel cannot apply in the case as Lim Tong Lim also benefited from the use of the nets in the boat, which was an asset of the partnership. Under the law on estoppel, those acting in behalf of a corporation and those benefited by it, knowing it to be without valid existence are held liable as general partners. Hence, the question as to whether such was legally formed for unknown reasons is immaterial to the case.

Piercing the veil of corporate fiction Umali vs Court of Appeals 189 SCRA 529 [GR No. 89561 September 13, 1990]

Plaintiff Santiago Rivera is the nephew of plaintiff Mauricia Mur Vda. de Castillo. The Castillo family are the owners of parcel of land located in Lucena City which was given as security for a loan from the development Bank of the Philippines (DBP) for their failure to pay the amortization, foreclosure of the said property was about to be initiated. This problem was made known to Santiago Rivera, who proposed to them the conversion into subdivision of the four parcels of land adjacent to the mortgaged property to raise the necessary fund. The idea was accepted by the Castillo family and to carry out the project, a memorandum of agreement was executed by and between Slobec Realty and Development Inc. represented by its president Santiago Rivera and Castillo family. In this agreement, Santiago Rivera obliged himself to pay the Castillo family the sum of P70,000 immediately after the execution of the agreement and to pay additional amount of P40,000 after the property has been converted into a subdivision. Rivera, with agreement approached Mr. Modesto Cervantes, president of defendant Bormaheco and proposed to purchase from Bormaheco two tractors model D7 and D8 subsequently a sales agreement was executed on December 28, 1970. On January 3, 1971, Slobec, through Rivera, executed in favor of Bormaheco a chattel mortgage over the said equipment as security for the payment of the aforesaid balance of P180,000. As further security of the aforementioned unpaid balance, Slobec obtained from insurance corporation of the Philippines a security bond, with Insurance Corporation of the Philippines (ICP) as surety and Slobec as principal, in favor of Bormaheco, as borne out of by Exhibit 8. The aforesaid surety bond was in turn secured by an agreement of counter-guaranty with real estate mortgage executed by Rivera as President of Slobec and Mauricia Mur Vda. de Castillo, Buenaflor Castillo Umali, Bertilla Castillo-Rada, Victoria Castillo, Marietta Castillo and Leovina Castillo Jalbuena as mortgagors and insurance corporation of the Philippines as mortgagee. In this agreement, ICP guaranteed the obligation of Slobec with Bormaheco in the amount of P180,000. In giving the bond, ICP required that the Castillos mortgage to them the properties in question,

Whether or not the foreclosure is proper so as to apply the doctrine of piercing the veil of corporate entity.

No. Under the doctrine of piercing the veil of corporate entity, when valid grounds therefore exists, the legal fiction that a corporation is an entity with a juridical personality separate and distinct from its members or stockholders may be disregarded. In such cases, the corporation will be considered as a mere association of persons. The members or stockholders of the corporation will be considered as the corporation, that is, liability will attach directly to the officers and stockholders. The doctrine applies when the corporate fiction is used to defeat public convenience, justify wrong, protect fraud, or defend crime, on when it is made as a shield to confuse the legitimate issues or where a corporation is the mere alter ego or business conduit of a person, or where the corporation is so organized and controlled and its affairs are so conducted as to make it merely an instrumentality, agency, conduit or adjunct of another corporation. In the case at bar, petitioners seek to pierce the veil of corporate entity of Bormaheco, ICP and PM parts, alleging that these corporations employed fraud in causing the foreclosure and subsequent sale of the real properties belonging to petitioners while we do not discount the possibility of existence of fraud in the foreclosure proceeding, neither are we inclined to apply the doctrine invoked by petitioners in granting the relief sought. It is our considered opinion that piercing the veil of corporate entity is not the proper remedy in order that the foreclosure proceeding may be declared a nullity under the circumstances obtaining in the legal case at bar. The mere fact, therefore, that the business of two or more corporations are interrelated is not a justification for disregarding their separate personalities, absent sufficient showing that the

Koppel (Phil) vs. Yatco, 77 PHIL 496 (1946)

namely, four parcels of land covered by TCT in the name of the aforementioned mortgagors, namely TCT no. 13114, 13115, 13116, and 13117 all of the Register of Deeds of Lucena City. Meanwhile, for violation of the terms and conditions of the counter-guaranty agreement, the properties of the Castillos were foreclosed by ICP as the highest bidder with a bid of P285,212, a certificate of sale was issued by the provincial sheriff of Lucena City and TCT over the subject parcels of land were issued. Koppel Industrial Car and Equipment company (KICE), a foreign company not doing business in the Philippines, owned 995 shares out of the 1000 shares that comprise the capital stock of KPI, a domestic corporation licensed as commercial broker in the Philippines. The remaining 5 shares were owned by each of the officers of KPI. KICE is in the business of selling railway materials, machineries and supplies. Buyers in the Philippines, when interested, asked for price quotations from KPI, and KPI then cabled for the quotation desired from KICE. However, KPI quoted to the purchaser a selling price above the figures quoted by KICE. On the basis of these quotations, orders were placed by the local buyers. Between KICE and KPI, the arrangement nonetheless was that KICE controls how much share of the profits goes to KPI. For these transactions, the BIR treated KPI as a subsidiary of KICE and collected from KPI the merchants’ sales tax, which was a revenue law in force at the time the sales took place.

corporate entity was purposely used as a shield to defraud creditors and third persons of their rights.

1. W/N KPI did business with the local buyers as an agent of KICE and not as broker 2. W/N the application of “piercing the corporate veil” doctrine is proper

The present case is a petition for Certiorari and prohibition with prayer for the issuance of a writ of preliminary 1. injunction to prohibit the respondent Court of Industrial Relations from proceeding with the hearing of the contempt proceedings. 2. FACTS La Campana Starch Factory and La Campana Coffee Factory (La Campana for Brevity) are two separate entities

Yes. The facts that KICE unilaterally controls the amount of so-called “share in the profits” of KPI and that KICE owns an overwhelming majority (99.5%) of the capital stock of the KPI are sufficient to conclude that the latter is a mere dummy, agent or wholly-owned subsidiary of KICE. Such conclusion is based on the doctrine that courts may ‘pierce the corporate veil’ to uncover the true intents of these corporations.

2. Yes. With regards only to the transactions involved, KPI and KICE were treated as one and the same so that taxes could be rightly collected. The court has to disregard this “corporate fiction” to prevent KICE / KPI from evading its taxes by contravening the local internal revenue laws. The court did not deny legal personality to KPI; in fact, it had no power to hold so. The doctrine was used only to adjudge the rights and liabilities of each parties in these kind of transactions.

KPI paid the taxes under protest, demanded for refund and contended that KPI could not be liable for merchants’ sales tax because it was only acting as broker between KICE and the local buyers. The lower court dismissed the complaint and ruled in favor of the government. RICARDO TANTONGCO, petitioner, v. KAISAHAN NG MGA MANGGAGAWA SA LA CAMPANA (KKM) and THE HONORABLE COURT OF INDUSTRIAL RELATIONS, respondents

1.

ISSUE 1. WON La Campana ceased to exist upon the death of Ramon Tantongco; 2. WON the Doctrine of Piercing the Veil of Corporate Existence

RULING The Supreme Court DENIED the Petition for Certiorari and Prohibition. It ruled that La Camapana continued to exist despite the death of Ramon Tantongco. It further ruled that the Doctrine of Piercing the Veil of Corporate Existence is not applicable in the present case. Finally, it allowed the CIR to proceed with the contempt hearing.

GR No. L-13119 || September 22, 1959

run by a single management under the leadership of Ramon Tantongco. Kaisahan ng mga Manggagawa sa La Campana (Kaisahan for brevity), on the other hand, 3. is a labor union with members from the two companies. Sometime in June, 1951, representatives of Kaisahan approached the management of La Campana to demand higher wages and more benefits. A deadlock ensued since none of the parties is willing to give concessions. The dispute was certified to the Court of Industrial Relations (CIR). La Campana filed a motion to dismiss before the CIR claiming that the CIR has no jurisdiction because only those from the coffee factory were presenting the demands there were only 14 employees in said factory. This was done in light of the requirement that at least 31 employees should present the demands. The motion was denied by the CIR. According to the CIR, the Kaisahan was the one that presented the demands and not just the workers in the coffee factory. The Supreme Court affirmed the order of the CIR citing that although the two entities are separate, there is only one management. The entire membership of the Kaisahan is therefore to be counted and not simply those employed in the coffee factory. Additional incidental cases were filed by Kaisahan before the CIR including a petition for the reinstatement of some employees. Ramon Tantongco died some time in 1956. The administrator of the estate of Ramon Tantongco, herein petitioner Ricardo Tantongco, was ordered included as respondent in the cases pending before the CIR. The CIR rendered a decision on the incidental cases and ordered the reinstatement of the dismissed employees. When the employees reported to work, the management refused them admittance. Kaisahan then filed a petition to cite the management in contempt before the CIR. Hence this petition. CONTENTIONS Petitioner: The two companies ceased to exist upon the death of Ramon Tantongco. The Supreme Court held in GR No. L-5677 that La Campana and Ramon Tantongco are one based on the doctrine of piercing the veil of corporate existence. Therefore, the death of Ramon Tantongco meant the death of La Campana. Since La Campana already ceased to exist, the CIR no longer has jurisdiction over it. The claims should have been filed with

applies to the present case; and 3. WON the contempt of court proceedings in the CIR should proceed.

1 and 2 The death of Ramon Tantongco did not end the existence of La Campana. The Supreme Court applied the Doctrine of Piercing the Veil of Corporate Existence in GR no. L-5677 to avoid the use of technicality to defeat the jurisdiction of the CIR. In the said case, the Court determined that although La Campana are two separate companies, they are being managed by only one management. Furthermore, the workers of both factories were interchangeably assigned. In the present case, however, the Court ruled that despite the obvious fact that La Campana was run by the same people, they still are two different companies with separate personalities from Ramon Tantongco. La Campana was owned not only by Ramon but others as well including Ricardo Tantongco. Lastly, the Court ruled that petitioner is under estoppel and cannot claim that La Campana and Ramon are one and the same since he has represented La Campana as separate entities in numerous dealings.

3. Ricardo Tantongco should still face the contempt proceedings because under Section 6 of Commonwealth Act No. 143, “In case the employer (or landlord) committing any such violation or contempt is an association or corporation, the manager or the person who has the charge of the management of the business of the association or corporation and the officers of directors thereof who have ordered or authorized the violation of contempt shall be liable. . . .” Since Tantongco is the General Manager of La Campana, he is still obliged to appear at the contempt proceedings.

the probate court. Defendant: La Campana continues to exist despite the death of Ramon Tantongco. The CIR therefore has jurisdiction when it rendered its decision on the incidental cases. The non-compliance by La Campana therefore amounted to contempt of court. QUINTIN ROBLEDO, MARIO SINLAO, LEONARDO SAAVEDRA, VICENTE SECAPURI, DANIEL AUSTRIA, ET AL., petitioners, vs. THE NATIONAL LABOR RELATIONS COMMISSION, BACANI SECURITY AND ALLIED SERVICES CO., INC., AND BACANI SECURITY AND PROTECTIVE AGENCY AND/OR ALICIA BACANI, respondents. Ponente: Mendoza, J.

Robledo ET. Al. filed a Petition for Review of the Decision of NLRC, setting aside the decision of the Labor Arbiter, which held private respondents jointly and severally liable to the petitioners for overtime and legal holiday pay. Petitioners were former employees of Bacani Security and Protective Agency (BPSA). They were employed as security guards at different times during the period 1969 to December 1989 when BPSA ceased to operate. BPSA was a single proprietorship owned, managed, and operated by the late Felipe Bacani. On December 31, 1989, Felipe Bacani retired the business name and BSPA ceased to operate effective on that day. On Jan. 15, 1990 Felipe Bacani died. An intestate proceeding was instituted for the settlement of his estate before Pasig-RTC. Earlier, on Oct. 26, 1989, respondent Bacani Security and Allied Services Co., Inc. (BASEC) had been organized and registered as a corporation with SEC. The following were the incorporators with their respective shareholdings: ALICIA BACANI — 25,250 shares LYDIA BACANI — 25,250 shares AMADO P. ELEDA — 25,250 shares VICTORIA B. AURIGUE — 25,250 shares FELIPE BACANI — 20,000 shares On July 5, 1990, the petitioners filed a complaint with the DOLE for underpayment of wages and nonpayment of overtime pay and other accrued benefits, and for the return of their cash bond, which they posted, with BPSA. Made respondents were BSPA and BASEC. On March 1, 1992, the Labor Arbiter rendered a decision upholding the right of petitioners, finding the complainants entitled to their money claims to be paid by all the respondents’ solidarily. On appeal, the NLRC reversed the decision declaring that the Labor Arbiter is without jurisdiction and instead suggested that petitioners file their claims with Pasig-RTC where an intestate proceeding of Bacani’s estate was pending.

Whether Bacani Security and Allied Services, Inc. (BASEC) can be held liable for claims of petitioners against Bacani Security and Protective Agency (BSPA).

No. Petitioners contend that public respondent, NLRC, erred in setting aside the Labor Arbiter’s judgment on the ground that BASEC is the same entity as BSPA the latter being owned and controlled by one and the same family, the Bacani family. For this reason they urge that corporate fiction should be disregarded and BASEC should be held liable for the obligations of the defunct BSPA. As correctly found by the NLRC, BASEC is an entity separate and distinct from that of BSPA. BSPA is a single proprietorship owned and operated by Felipe Bacani. Hence, its debts and obligations were the personal obligations of its owner. Petitioner’s claims, which are based on these debts and personal obligations, did not survive the death of Felipe Bacani on Jan. 15, 1990 and should have been filed instead in the intestate proceedings involving his estate. The rule is settled that unless expressly assumed labor contracts are not enforceable against the transferee of an enterprise. The reason for this is that labor contracts are in personam. Consequently, it has been held that claims for backwages earned from the former employer cannot be filed against the new owners of an enterprise. 3Nor is the new operator of a business liable for claims for retirement pay of employees. It is apparent, therefore, that the doctrine of piercing the corporate veil has no application to this case where the purpose is not to hold the individual stockholders liable for the obligations of the corporation but, on the contrary, to hold the corporation liable for the obligations of a stockholder or stockholders. Piercing the veil of

Petitioners moved for reconsideration but their motion was denied for lack of merit. The case was elevated to the SC and was treated as a special civil action of certiorari to determine whether the NLRC committed a grave abuse of discretion in reversing the Labor Arbiter’s decision. Corporation Law: The doctrine of piercing the veil of corporate entity is used whenever a court finds that the corporate fiction is being used to defeat public convenience, justify wrong, protect fraud, or defend crime, or to confuse legitimate issues, or that a corporation is the mere alter ego or business conduit of a person or where the corporation is so organized and controlled and its affairs are so conducted as to make it merely an instrumentality, agency, conduit or adjunct of another corporation.

Gregorio Araneta, Inc. vs. Tuazon, 91 PHIL 786 (1952)

Piercing the veil of corporate fiction Fraud cases Paz Tuason de Paterno is the registered owner of a 40,703 MAIN ISSUE 1: Whether sq.m. big block of residential land in the district of Santa the deed of sale Mesa, Manila which was subdivided into city lots. (Exhibit A) is valid Most of these lots were occupied by lessees who had despite: contracts of lease which were to expire on Dec. 31,1952, The alleged agreement and carried a stipulation that in the event the owner and in the Promise to Buy lessor should decide to sell the property, the lessees were and Sell (Exhibit 1, par. to be given priority over other buyers if they should desire 8) that there was to be to buy their leaseholds. no execution of the Paz Tuason obtained from Jose Vidal several loans totalling deed of absolute sale to P90,098 and constituted a first mortgage on the aforesaid Gregorio Araneta, Inc., property to secure the debt. She obtained additional loans unless Vidal's mortgage of P30,000 and P20,000 upon the same security. was cancelled? (Wrong Each time, the previous contract of mortgage was renewed interpretation, Deed of and the amounts received were consolidated. Sale Valid) In the first novated contract (Jan. 1943), the time of Discrepancies between payment was fixed at 2 years and in the second (April the Promise to Buy and 1943) at 4 years. Sell and the Deed of

corporate entity means looking through the corporate form to the individual stockholders composing it. Here there is no reason to pierce the veil of corporate entity because there is no question that petitioners' claims, assuming them to be valid, are the personal liability of the late Felipe Bacani. It is immaterial that he was also a stockholder of BASEC. Indeed, the doctrine is stood on its head when what is sought is to make a corporation liable for the obligations of a stockholder. But there are several reasons why BASEC is not liable for the personal obligations of Felipe Bacani. For one, BASEC came into existence before BSPA was retired as a business concern. BASEC was incorporated on October 26, 1989 and its license to operate was released on May 28, 1990, while BSPA ceased to operate on December 31, 1989. Before, BSPA was retired, BASEC already existed. It is, therefore, not true that BASEC is a mere continuity of BSPA.

RATIO 1: Exhibit A Valid Despite Non-Cancellation of Mortgage LC: Contract was invalid based on Exhibit 1, par. 8 that there was to be no absolute sale to Gregorio Araneta, Inc., unless Vidal's mortgage was cancelled. SC: The contemplated execution of an absolute deed of sale was not contingent on the cancellation of Vidal's mortgage but as provided “once determined by Paz Tuason, may sell to Gregorio Araneta, Inc." The lots which could be sold to Gregorio Araneta, Inc. were definitely known by the expiry of the tenants' option to buy. Exhibit A was then in a condition to be made. Vidal's mortgage was not an obstacle to the sale.

There was also a separate written agreement entitled

"Penalidad del Documento de Novacion de Esta Fecha" ("Penalty Document Novation of This Date) which, unlike

the principal contracts, was not registered. Paz Tuason decided to sell the entire property for the net amount of P400,000 and entered into negotiations with Gregorio Araneta, Inc. for this purpose. Oct. 19, 1943: Contract called "Promesa de Compra y Venta" ("Promise to Buy and Sell" also Exhibit 1) was executed. It provided that subject to the preferred right of the lessees and that of Jose Vidal as mortgagee, Paz Tuason would sell to Gregorio Araneta, Inc. and the latter would buy for the said amount of P400,000 the entire estate under these terms. For the Tenants: 40% paid together with the letter of acceptance of the tenant 20 % at the execution of deed of sale agreement (Contract to Sell??) 40% at the execution of deed of final sale, which will be granted after the mortgage has been canceled Par. 8: 5% commission corresponding to Jose Araneta will be paid at the execution of the deed of sale agreement He was referred to as defendant's agent or broker "who acts in this transaction" and who as such was to receive a commission of 5 per cent, although the commission was to be charged to the purchasers Par. 13: Paz Tuason promised, in consideration of Jose

Araneta's services rendered to her, to assign to him all her right, title and interest to and in certain lots not embraced in the sales to Gregorio Araneta, Inc. or the tenants

At the expiry of the period (Dec. 31,1952), Paz Tuason will grant the corresponding deeds of sale to tenants who have decided to buy their lots. Any tenant who decides to buy the lot may choose to order the immediate execution in his favor of Deed of Final Sale if 50% of the price besides the 40% included in his letter of acceptance was paid and the remaining 10% paid immediately after mortgage was canceled For Gregorio Araneta Inc: Paz Tuason, acknowledges receipt of P190, 000 as an advance of the sale price from Gregorio Araneta, Inc.

Sale and allegations of fraud in the execution of the Deed of Absolute Sale? (YES, no fraud) Being a sale between the agent and his principal? (Jose Araneta is not an agent of Tuason but a BROKER, Deed of Sale valid) Being drawn by lawyers of the buyer who are at the same time lawyers of the seller? (Araneta firm is not the lawyer of Tuazon, Deed of sale valid)

An amount had been set aside to take care of it, and the parties were confident that the suit against the mortgagee would succeed. The only doubt in their minds was in the amount to which Vidal was entitled. The failure of the court to try and decide the case was not foreseen either. There was no undue rush on the part of Gregorio Inc., to push across the sale. The fact that similar deeds were given to the lessees who bought their leaseholds and these were never questioned by Tuason dispels any suspicion of bad faith on Gregorio Inc. If anyone was in a hurry it could have been Paz Tuason as she was pressed for cash. The payment of the mortgage was only an incident, or a necessary means to effectuate the sale. Otherwise she could have settled her mortgage obligation merely by selling a portion of her estate. Whatever the terms of Exhibit 1, Tuason and Gregorio Araneta Inc. were at perfect liberty to make a new agreement different from or even contrary to the provisions of that document. The validity of the subsequent sale must of necessity depend on what it said and not on the provisions of the promise to buy and sell. Exhibit A Valid: No Fraud, Substantial Compliance with Exhibit 1 Except in two particulars, Exhibit A was a substantial compliance with Exhibit 1 in furtherance of which Exhibit A was made. FIRST DIFFERENCE: 10 % of the purchase price should be paid only after Vidal's mortgage should have been cancelled. It is not onerous or unusual that the vendee should withhold a small portion of the purchase price before all the impediments to the final consummation of the sale had been removed. The tenants who had bought their lots had been granted the privilege to deduct as much as 40% Seller has already received from the tenant purchasers 90% of the purchase money. Had Gregorio Araneta, Inc. not insisted on charging

The amount that Paz Tuason receives in this act will be applied to pay her debt to Jose Vidal The lots which were not bought by the lessees will immediately be sold in favor of Gregorio Araneta, Inc. Gregorio Araneta, Inc., to pay the sale price as follows: 90% of upon execution of Deed of Final Sale discounting P190,000 already paid at the Execution of this Contract. Remaining 10% will be paid once the mortgage over property was canceled If P190, 000 exceeds the 90% of the price to be paid by Gregorio Araneta Inc., the balance will be paid immediately by Paz Tuazon, taking it from the amounts received from tenants on the sale of the lots Letters were sent the lessees giving them until Aug. 31, 1943, an option to buy the lots they occupied, at the price and terms stated in said letters. Most took advantage of the opportunity and after making the stipulated payments were given their deeds of conveyance. In the end, Lots 1, 8-16 and 18 (aggregate area of 14,810.20 sq.m.) remained unencumbered, except for the mortgage to Jose Vidal. Dec. 2, 1943: Paz Tuason and Gregorio Araneta, Inc. executed with regard to these lots an ABSOLUTE?? DEED OF SALE (Exhibit A) the terms of which, except in two respects, were similar to those of the sale to the lessees Price: P139,083.32 (90% is P125,174.99) Paz (Vendor) having received P190,000 from Gregorio, Inc (Vendee) upon the execution of "Promesa de Compra y Venta, returned the excess balance of P64,825.01 P190,000 was delivered by the Vendee to the Vendor by virtue of 4 checks issued by the Gregorio Inc. against BPI – 1 in favor of Paz (P13,476.62), 2 in favor of Jose Vidal (P143,150 + 30,000), 1 in favor of City Treasurer (3,373.38) Nov. 2, 1943: The return of P64,825.01 was made by Paz to Gregorio Inc. liquidated as follows: Payments from tenants Dumas, Sycip, Pabalan, del Rosario amounted to P68,563.21 from which 5% commissions of de Pabalan, Tuason, Dumas amounting to 3,244.97 and check to Paz amounting to 493.23 were deducted In view of the foregoing liquidation, the vendor acknowledges fully and unconditionally, having received

to Tuason the loss of the checks deposited with the court, the sale in question would have gone the smooth way of the sales to the tenants as declared by Dindo Gonzales, Paz's son in his testimony. SECOND DIFFERENCE: The stipulation by which Gregorio Araneta Inc. would hold Paz Tuason liable for the lost checks. (See above in Exhibit A-bold and underline) Difficult to believe that Paz Tuason was deceived into signing Exhibit A, in spite of the provision of which she and her son complain of. Paz Tuason had an able attorney (JPEnrile) who was assisting her in the suit against Vidal and a son (Dindo Gonzales) who is leading citizen and a business-man and knew the English language very well if she did not. She denied or at least pretended in her answer to be ignorant of the existence of Exhibit A, and only after she was confronted with the signed copy of the document on the witness did she spring up the defense of fraud. It would look as if she gambled on the chance that no signed copy of the deed had been saved from the war. From the unreasonableness and inequity of Exhibit A it is not to be presumed that she did not understand it. JOSE ARANETA: NOT AGENT BUT BROKER Paz Tuazon: Gregorio Inc.’s President, Jose Araneta is Tuazon’s agent and applying the principle stated in 18 C.J.S. 380: "The courts, at

law and in equity, will disregard the fiction of corporate entity apart from the members of the corporation when it is attempted to be used as a means of accomplishing a fraud or an illegal act”,

Jose Araneta and Gregorio Inc. is one and the same. Thus, Gregorio cannot buy Tuazon’s property. TC: Jose Araneta, Gregorio Inc’s President, was not Paz Tuason's agent or broker. However, hypothetically admitting the existence of an agency relation between Paz Tuason and Jose Araneta, Gregorio Araneta, Inc. was the purchaser and not

P125,174.99 of the present legal currency and hereby expressly declares that she will not hold the Vendee responsible for any loss that she might suffer due to the fact that 2 of the checks paid to her by the Vendee were issued in favor of Jose Vidal and the latter has, up to the present time, not yet collected the same. 10% balance of the purchase price (P13,908.33) not yet paid will be paid by the Vendee to the Vendor when the existing mortgage over the property sold by the Vendor to the Vendee is duly cancelled in the office of the Register of Deeds, or sooner at the option of the Vendee. Oct. 20, 1943: Before the execution of this Absolute deeds of Sale, Paz Tuason had already offered to Vidal the check for P143,150 mentioned in Exhibit A, in full settlement of her mortgage obligation Vidal refused to receive that check or to cancel the mortgage, contending that by the separate agreement before mentioned payment of the mortgage was not to be effected totally or partially before the end of 4 years from April, 1943. Oct.-Nov. 1943: Paz Tuason, through Atty. Enrile, commenced an action against Vidal but the record of that case was destroyed and no copy of the complaint was presented in evidence. They deposited with the clerk of court a check for P143,150 previously turned down by Vidal, another certified check for P12,932.61, also drawn by Gregorio Araneta, Inc., in favor of Vidal, and one ordinary check for P30,000 issued by Paz Tuazon. 3 checks were supposed to cover the whole indebtedness to Vidal including the principal and interest up to that time and the penalty provided in the separate agreement. The action against Vidal never came on for trial and the record and the checks were destroyed during the war operations in Jan. or Feb., 1945; and neither was the case reconstituted afterward. This failure of the suit for the cancellation of Vidal's mortgage, coupled with the destruction of the checks tendered to the mortgagee, the nullification of the bank deposit on which those checks had been drawn, and the tremendous rise of real estate value following the termination of the war, gave occasion to the breaking off

Jose Araneta citing the well-known distinction

between the corporation and its stockholders. The

sale to Gregorio Araneta, Inc. was not a sale to Jose Araneta, the agent or broker. SC as to TC Ruling: TC disregarded evidence. In par. 8 of Exhibit 1, Jose Araneta was referred to as Tuason’s agent or broker "who acts in this transaction" and who as such was to receive a commission of 5 %, while in par. 13, Tuason promised, in consideration of Jose Araneta's services rendered to her, to assign to him all her right, title and interest to and in certain lots not embraced in the sales to Gregorio Araneta, Inc. or the tenants. SC as to Corporate Theory: This principle does not fit in with the facts of the case. Gregorio Araneta, Inc. entered into the contract for itself and for its benefit as a corporation. The contract and the roles of the parties who participated therein were exactly as they purported to be and were fully revealed to the seller. There is no pretense, nor is there reason to suppose, that if Paz Tuason had known Jose Araneta to be Araneta, Inc's president, she would not have gone ahead with the deal. It would have made no difference, except for the brokerage fee, whether Gregorio Araneta, Inc. or Jose Araneta was the purchaser. SC Relation bet. Tuason&Araneta: Not w/n Art. 1459: Granting Jose & G.A.Inc. were identical and that the acts of one where the acts of the other, the relation between Tuason and Jose Araneta did not fall within the purview of article 1459 of the Spanish Civil Code. Art. 1459. The following persons cannot take by purchase, even at a public or judicial auction, either in person or through the mediation of another: An agent, any property of which the management or sale may have been intrusted to him Agency is defined in article 1709 in broad terms, and we have not come across any commentary or decision dealing directly with the precise meaning

the schemes outlined in Exhibits 1 and A Paz Tuason, after liberation, repudiated them for certain reasons. She alleges that Exhibit A is not valid because: There are discrepancies between the Promise to Buy and Sell (Exhibit 1) and Absolute Deed of Sale (Exhibit A) Exhibit 1, under par. 8, there was to be no absolute sale to Gregorio Araneta, Inc., unless Vidal's mortgage was cancelled. Exhibit A had no counterpart in Exhibit 1 by which Gregorio Araneta Inc. would hold Paz Tuason liable for the lost checks. No person in his or her right senses would knowingly have agreed to a covenant so iniquitous and unreasonable. There was undue rush on the part of Gregorio Inc., to push across the sale. Paz Tuason was deceived into signing Attys. Salvador Araneta and J. Antonio Araneta of Araneta & Araneta, who had drawn Exhibit A, did not inform her about its contents Being English, she had not read the deed of sale; that if she had not trusted the said attorneys she would not have been so foolish as to affix her signature to a contract so onesided. Technical objections are made against the deed of sale. Jose Araneta, since deceased, was defendant's agent and at the same time the president of Gregorio Araneta, Inc. The law firm of Araneta & Araneta who represent Gregorio Araneta, Inc. were her attorneys also. Gregorio Araneta, Inc. thus filed an action to compel Paz Tuason to deliver to the plaintiff a clear title to the lots described in Exhibit A free from all liens and encumbrances, and a deed of cancellation of the mortgage to Vidal. Vidal was summoned by order of the court, and filed a cross-claim against Paz Tuazon to foreclose his mortgage.

of agency as employed in article 1459. Manresa’s Opinion: Agent, in the sense there used, is one who accepts another's representation to perform in his name certain acts of more or less transcendency. Scaevola’s Opinion: The agent's incapacity to buy his principal's property rests in the fact that the agent and the principal form one juridicial person. In this connection, the fear that greed might get the better of the sentiments of loyalty and disinterestedness which should animate an administrator or agent, is the reason underlying various classes of incapacity enumerated in article 1459. American Courts: The law does not trust human nature to resist the temptations likely to arise of antogonism between the interest of the seller and the buyer. The ban of par. 2 of Art. 1459 connotes the idea of trust and confidence; and so where the relationship does not involve considerations of good faith and integrity the prohibition should not and does not apply. To come under the prohibition, the agent must be in a fiduciary with his principal. CASE AT BAR (Parang what is a broker): Tested by this standard, Jose Araneta was not an agent within the meaning of article 1459. He was to be nothing more than a go-between or middleman between the defendant and the purchaser, bringing them together to make the contract themselves. There was no confidence to be betrayed. Jose Araneta was not authorized to make a binding contract for the defendant. He was not to sell and he did not sell the defendant's property. He was to look for a buyer and the owner herself was to make, and did make, the sale. He was not to fix the price of the sale because the price had been already fixed in his commission. He was not to make the terms of payment because these, too, were clearly specified in his commission.

In fine, Jose Araneta was left no power or discretion whatsoever, which he could abuse to his advantage and to the owner's prejudice. Araneta&Araneta: Not Tuason’s Lawyers Since attorney Ponce Enrile was the defendant's lawyer in the suit against Vidal, it was not likely that she employed Atty. Salvador Araneta and J. Antonio Araneta as her attorneys in her dealings with Gregorio Araneta, Inc., knowing their identity with the buyer. The fact that Attys. Salvador and Araneta and J. Antonio Araneta drew Exhibits 1 and A could very well have been written in furtherance of Gregorio Araneta's own interest. Granting that Attorney Araneta and Araneta were attorneys for the Tuason, yet they were not forbidden to buy the property in question. Attorneys are only prohibited from buying their client's property which is the subject of litigation. (Art. 1459, No. 5, Spanish Civil Code) The questioned sale was effected before the subject thereof became involved in the present action. G.R. No. L-15121, August 31, 1962 Gregorio Palacio and Mario Palacio (minor) vs Fely Transportation Company Ponente: Regala

In their complaint, the Palacio alleged that Fely hired Alfredo Canillo as driver who negligently run over a child (Mario). Gregorio , the father of Mario is a welder and in the account of his child's injuries has abandoned his shop which is the family's source of income. Fely filed a motion to dismiss on the grounds that there is no cause of action against the company and that the cause of action is barred by prior judgment. But the court deferred the determination of the grounds alleged in the motion to dismiss until the trial of the case. The defendant then alleges (1) that complaint states no cause of action against defendant, and (2) that the sale and transfer of the jeep AC-687 by Isabelo Calingasan to the Fely Transportation was made on December 24, 1955, long after the driver Alfredo Carillo of said jeep had been convicted and had served his sentence.

Whether Fely Transportation can be held liable for the damages.

The Court agrees with this contention of the plaintiffs. Isabelo Calingasan and defendant Fely Transportation may be regarded as one and the same person. It is evident that Isabelo Calingasan's main purpose in forming the corporation was to evade his subsidiary civil liability resulting from the conviction of his driver, Alfredo Carillo. This conclusion is borne out by the fact that the incorporators of the Fely Transportation are Isabelo Calingasan, his wife, his son, Dr. Calingasan, and his two daughters.

Accordingly, defendants Fely Transportation and Isabelo Calingasan should be held subsidiarily liable for P500.00 which Alfredo Carillo was ordered to pay in the criminal case and which amount he could not pay on account of insolvency.

In view of the evidence presented, the lower court barred the judgment in the criminal case and held that the person subsidiarily liable to pay damages is Isabel Calingasan, the employer. PALAY, INC. vs. CLAVE G.R.No. L-56076

Pabalan and Lagdameo v. NLRC (G.R. No. 89879)

ON March 28, 1965, petitioner Palay, Inc., through its President, Albert Onstott executed in favor of private respondent, Nazario Dumpit, a Contract to sell a parcel of Land in Antipolo, Rizal owned by said corporation. The sale price was P23, 300.00 with 9% interest per annum, payable with a down payment of P4, 660.00 and monthly installments of P246.42 until fully paid. Paragraph 6 of the contract provided for automatic extrajudicial rescission upon default in payment of any monthly installment after the lapse of 90 days from the expiration of the grace period of one month, without need of notice and with forfeiture of all installments paid. Respondent Dumpit paid the down payment and several installments amounting to P13, 722.50. The last payment was made on December 5, 1967 for installments up to September 1967. On May 10, 1973, or almost six (6) years later, private respondent wrote petitioner offering to update all his overdue accounts with interest, and seeking its written consent to the assignment of his rights to a certain Lourdes Dizon. In response, petitioners informed respondent that his Contract to Sell had long been rescinded pursuant to paragraph 6 of the contract, and that the lot had already been resold. A complaint was filed by the respondent with the NHA for conveyance with an alternative prayer for refund. The NHA, in its resolution, ordered Palay, Inc. and Alberto Onstott in his capacity as President of the corporation, jointly and severally, to refund immediately to respondent the amount paid with 12% interest from the filing of complaint. Respondent Presidential Executive Assistant Clave affirmed the NHA resolution.

1. Whether the doctrine of piercing the veil of corporate fiction has application to the case. 2. Whether petitioner On Stott can be held solidarity liable with petitioner Corporation for the refund of the installment payments made by respondent Dump it.

The doctrine of piercing the veil of corporate fiction has no application to the case. Consequently, petitioner Onstott cannot be held solidarity liable with petitioner Corporation for the refund of the installment payments made by respondent Dumpit. A corporation is invested by law with a personality separate and distinct from those of the persons composing it. As a general rule, a corporation may not be made to answer for acts or liabilities of its stockholders or those of the legal entities to which it may be connected and vice versa. However, the veil of corporate fiction may be pierced when: it is used as a shield to further an end subversive of justice; or for purposes that could not have been intended by the law that created it; or to defeat public convenience, justify wrong, protect fraud, or defend crime; or to perpetrate fraud or con fuse legitimate issues; or to circumvent the law or perpetuate deception; or as an alter ego, adjunct or business conduit for the sole benefit of the stockholders. In this case however, there are no badges of fraud on the part of the petitioners. They had literally relied, although mistakenly, on paragraph 6 of the contract with respondent when they rescinded the contract to sell extra judicially. Although On Stott appears to be the controlling stockholder, there being no fraud, he cannot be made personally liable.

84 workers of the Philippine Inter-Fashion (PIF) filed a complaint against the latter for illegal transfer simultaneous with illegal dismissal in violation of the Labor Code. PIF was notified about the complaint and summons but hearings

Whether or not petitioners as officers may be held jointly and severally liable with the

NO. The settled rule is that the corporation is vested by law with a personality separate and distinct from the persons composing it, including its officers as

Del Rosario vs National Labor Relations Commission 187 SCRA 777 [GR No. 85416 July 24, 1990]

were continually re-set for failure of its officers (petitioners herein) to appear. Complainant workers thus moved to implead petitioners as officers of PIF in the complaint for their illegal transfer to a new firm. The Labor Arbiter ruled in favor of workers holding petitioners-officers jointly and severally liable with PIF to pay them their benefits. Petitioners’ appeal was dismissed.

corporation for its liability.

well as from that of any other legal entity to which it may be related. Thus, a company manager acting in good faith within the scope of his authority in terminating the services of certain employees cannot be held personally liable for damages. However, the legal fiction that a corporation has a personality separate and distinct from stockholders and members may be disregarded when the notion of legal entity is used as a means to perpetrate fraud or an illegal act or as a vehicle for the evasion of an existing obligation, the circumvention of statutes, and or (to) confuse legitimate issues the veil which protects the corporation will be lifted. In this particular case complainants did not allege or show that petitioners, as officers of the corporation deliberately and maliciously designed to evade the financial obligation of the corporation to its employees, or used the transfer of the employees as a means to perpetrate an illegal act or as a vehicle for the evasion of existing obligations, the circumvention of statutes, or to confuse the legitimate issues. Not one of the above circumstances has been shown to be present. Hence petitioners cannot be held jointly and severally liable with the PIF corporation under the questioned decision and resolution of the public respondent.

In POEA case no. 85-06-0394, the Philippine Overseas Employment Administration (POEA) promulgated a decision on February 4,1986 dismissing the complaint for money claims for lack of merit. The decision was appealed to the NLRC, which on April 30, 1987 reversed the POEA decision and ordered Philsa Construction and Trading Co.Ind and Ariel Enterprises (the foreign employer) to jointly and severally pay private respondent the peso equivalent of $16,039,000 salary differentials and $2,420.03 as vacation leave benefits. A writ of execution was issued by the POEA but it was returned unsatisfied incapable of satisfying the judgement. Private respondent moved for the issuance of an alias writ against the officers of Philsa. This motion was

Whether or not the POEA resolution is proper.

No. Under the law, a corporation is bestowed juridical personality, separate and distinct from its stockholders. But when the juridical personality of the corporation is used to defeat public convenience, justify wrong, protect or defend crime, the corporation shall be considered as a mere association of persons and its responsible officers and/or stockholders shall be individually liable. For the same reasons, a corporation shall be liable for obligations of a stockholder or a corporation and its successor-in-interest shall be considered as one and the liability of the former

opposed by the officers led by petitioners, the president and general manager of the corporation. However, POEA issued a resolution ordering the sheriff to execute against the properties of the petitioner and if insufficient, against the cash and/or surety bond of bonding company concerned for the full satisfaction of the judgement awarded.

shall attach to the latter. But for the separate juridical personality of a corporation to be disregarded, the wrong doing must be clearly and convincingly established. It cannot be presumed. Thus, at the time Philsa allowed its license to lapse in 1985 and even at the time it was delivered in 1986, there was yet no judgement in favor of private respondent. An intent to evade payment of his claims cannot therefore be implied from the expiration of Phila’s license and its delisting. Neither will the organization of Philsa International Placement and Services Corp. and its registration with the POEA as a private employment agency imply fraud since it was organized and registered in 1981, several years before private respondent filed his complaint with the POEA in 1985. The creation of the second anticipation of private respondent’s money claims and the consequent adverse judgement against Philsa. Likewise, substantially identity of the incorporators of the two corporations does not necessarily imply fraud.

Villa Rey Transit vs. Ferrer [GR L-23893, 29 October 1968]

[preceding case] Prior to 1959, Jose M. Villarama was an operator of a bus transportation, under the business name of Villa Rey Transit, pursuant to certificates of public convenience granted him by the Public Service Commission (PSC) in Cases 44213 and 104651, which authorized him to operate a total of 32 units on various routes or lines from Pangasinan to Manila, and vice-versa. On 8 January 1959, he sold the two certificates of public convenience to the Pangasinan Transportation Company, Inc. (Pantranco), for P350,000.00 with the condition, among others, that the seller (Villarama) "shall not for a period of 10 years from

Whether the stipulation, "SHALL NOT FOR A PERIOD OF 10 YEARS FROM THE DATE OF THIS SALE, APPLY FOR ANY TPU SERVICE IDENTICAL OR COMPETING WITH THE BUYER" in the contract between Villarama and Pantranco, binds the

Villarama supplied the organization expenses and the assets of the Corporation, such as trucks and equipment; there was no actual payment by the original subscribers of the amounts of P95,000.00 and P100,000.00 as appearing in the books; Villarama made use of the money of the Corporation and deposited them to his private accounts; and the Corporation paid his personal accounts. Villarama himself admitted that he mingled the corporate funds with his own money. These circumstances are strong persuasive

the date of this sale, apply for any TPU service identical or competing with the buyer." Barely 3 months thereafter, or on 6 March 1959: a corporation called Villa Rey Transit, Inc. (the Corporation) was organized with a capital stock of P500,000.00 divided into 5,000 shares of the par value of P100.00 each; P200,000.00 was the subscribed stock; Natividad R. Villarama (wife of Jose M. Villarama) was one of the incorporators, and she subscribed for P1,000.00; the balance of P199,000.00 was subscribed by the brother and sister-in-law of Jose M. Villarama; of the subscribed capital stock, P105,000.00 was paid to the treasurer of the corporation, who was Natividad R. Villarama. In less than a month after its registration with the Securities and Exchange Commission (10 March 1959), the Corporation, on 7 April 1959, bought 5 certificates of public convenience, 49 buses, tools and equipment from one Valentin Fernando, for the sum of P249,000.00, of which P100,000.00 was paid upon the signing of the contract; P50,000.00 was payable upon the final approval of the sale by the PSC; P49,500.00 one year after the final approval of the sale; and the balance of P50,000.00 "shall be paid by the BUYER to the different suppliers of the SELLER." The very same day that the contract of sale was executed, the parties thereto immediately applied with the PSC for its approval, with a prayer for the issuance of a provisional authority in favor of the vendee Corporation to operate the service therein involved. On 19 May 1959, the PSC granted the provisional permit prayed for, upon the condition that "it may be modified or revoked by the Commission at any time, shall be subject to whatever action that may be taken on the basic application and shall be valid only during the pendency of said application." Before the PSC could take final action on said application for approval of sale, however, the Sheriff of Manila, on 7 July 1959, levied on 2 of the five certificates of public convenience involved therein, namely, those issued under PSC cases 59494 and 63780, pursuant to a writ of execution issued by the Court of First Instance of Pangasinan in Civil Case 13798, in favor of Eusebio E. Ferrer against Valentin Fernando. The Sheriff

Corporation (the Villa Rey Transit, Inc.).

evidence showing that Villarama has been too much involved in the affairs of the Corporation to altogether negative the claim that he was only a part-time general manager. They show beyond doubt that the Corporation is his alter ego. The interference of Villarama in the complex affairs of the corporation, and particularly its finances, are much too inconsistent with the ends and purposes of the Corporation law, which, precisely, seeks to separate personal responsibilities from corporate undertakings. It is the very essence of incorporation that the acts and conduct of the corporation be carried out in its own corporate name because it has its own personality. The doctrine that a corporation is a legal entity distinct and separate from the members and stockholders who compose it is recognized and respected in all cases which are within reason and the law. When the fiction is urged as a means of perpetrating a fraud or an illegal act or as a vehicle for the evasion of an existing obligation, the circumvention of statutes, the achievement or perfection of a monopoly or generally the perpetration of knavery or crime, the veil with which the law covers and isolates the corporation from the members or stockholders who compose it will be lifted to allow for its consideration merely as an aggregation of individuals. Hence, the Villa Rey Transit, Inc. is an alter ego of Jose M. Villarama, and that the restrictive clause in the contract entered into by the latter and Pantranco is also enforceable and binding against the said Corporation. For the rule is that a seller or promisor may not make use of a corporate entity as a means of evading the obligation of his covenant. Where the Corporation is substantially the alter ego of the covenantor to the restrictive agreement, it can be enjoined from competing with the covenantee.

made and entered the levy in the records of the PSC. On 16 July 1959, a public sale was conducted by the Sheriff of the said two certificates of public convenience. Ferrer was the highest bidder, and a certificate of sale was issued in his name. Thereafter, Ferrer sold the two certificates of public convenience to Pantranco, and jointly submitted for approval their corresponding contract of sale to the PSC. Pantranco therein prayed that it be authorized provisionally to operate the service involved in the said two certificates. The applications for approval of sale, filed before the PSC, by Fernando and the Corporation, Case 124057, and that of Ferrer and Pantranco, Case 126278, were scheduled for a joint hearing. In the meantime, to wit, on 22 July 1959, the PSC issued an order disposing that during the pendency of the cases and before a final resolution on the aforesaid applications, the Pantranco shall be the one to operate provisionally the service under the two certificates embraced in the contract between Ferrer and Pantranco. The Corporation took issue with this particular ruling of the PSC and elevated the matter to the Supreme Court, which decreed, after deliberation, that until the issue on the ownership of the disputed certificates shall have been finally settled by the proper court, the Corporation should be the one to operate the lines provisionally. [present case] On 4 November 1959, the Corporation filed in the Court of First Instance of Manila, a complaint for the annulment of the sheriff's sale of the aforesaid two certificates of public convenience (PSC Cases 59494 and 63780) in favor of Ferrer, and the subsequent sale thereof by the latter to Pantranco, against Ferrer, Pantranco and the PSC. The Corporation prayed therein that all the orders of the PSC relative to the parties' dispute over the said certificates be annulled. The CFI of Manila declared the sheriff's sale of two certificates of public convenience in favor of Ferrer and the subsequent sale thereof by the latter to Pantranco null and void; declared the Corporation to be the lawful owner of the said certificates of public convenience; and ordered Ferrer and Pantranco, jointly and severally, to pay the Corporation, the sum of P5,000.00 as and for attorney's fees. The case against the PSC was

dismissed. All parties appealed. Piercing the veil of corporate fiction Alter ego G. C. ARNOLD vs. WILLITS & PATTERSON, LTD 44 Phil 634 (1923)

Arnold and Willits and Patterson, Ltd. entered into a contract by which plaintiff was appointed agent for a period of 5 years. A dispute arose as to the amount which plaintiff should receive for his services. Patterson retired and Willits became the sole owner of the assets of the firm. Willits then organized a corporation. He became exclusive owner except for a few stocks (nominal shares to qualify the directors) for organizational purposes. Another instrument was executed between Arnold and Willits. Such defined and specified the compensation of Arnold. Nothing shows that such was formally ratified or approved by the corporation. A statement of the corporation's account showed that there was due and owing the plaintiff a sum of money. The corporation's creditor's committee protested against such amount. Arnold filed suit to collect. Willits argued that the document was signed without the authority of the defendant corporation and also filed a counterclaim.

Whether plaintiff may collect from defendant corporation.

Yes. The proposition that a corporation has an existence separate and distinct from its membership has its limitations. It must be noted that this separate existence is for particular purposes. It must also be remembered that there can be no corporate existence without persons to compose it; there can be no association without associates. This separate existence is to a certain extent a legal fiction. Whenever necessary for the interests of the public or for the protection or enforcement of the rights of the membership, courts will disregard this legal fiction and operate upon both the corporation and the persons composing it. He continued his employment and rendered his services after the corporation was organized and the second document was signed just the same as he did before, and both corporations recognized and accepted his services. It was a one man corporation, and Willits, as the owner of all of the stock, was the force and dominant power which controlled them. After the document was signed it was recognized by Willits that the plaintiff's services were to be performed and measured by its term and provisions, and there never was any dispute between plaintiff and Willits upon that question. Statements of account were made and prepared by the accountant on the assumption that the document was in full force and effect as between the plaintiff and the defendant. Previous financial statements show upon their face that the account of plaintiff was credited with several small items on the same basis, and it was not until the 23d of March, 1921, that any objection was ever made by anyone.

The SC reverses. Arnold entitled to 68K plus half of 75K, representing PNs. Both Corp’s organized by Willits were a One Man Corporation. After the 2nd contract was signed it was recognized by Willits that Arnold’s services were to be performed by its terms and there never was any dispute between Arnold and Willits. Although a new corp was created, the new corp dealt with and treated Arnold as its agent in the same manner as the previous corp had, thus the new corp is bound by the contract which the old firm made. In fact, the 2nd contract protected Willits from a larger claim, which the accountant said, would be over 160K. Where a stock of a corporation is owned by one person whereby the corp functions only for the benefit of such individual owner, the corp and the individual should be deemed to be the same. Thus the corp is bound by the contract. LA CAMPANA COFFEE FACTORY v KAISAHAN NG MANGGAGAWA G.R. L-5677, May 25, 1953

Tan Tong since 1932 has been engaged in the buying and selling gawgaw under the trade name La Campana Gawgaw Packing. In 1950, Tan Tong and members of his family organized the family corporation. La Campana Coffee Factory with its principal office located in Gawgaw Packing. Prior to said information, Tan Tong entered into a CBA with the labor union of La Campana Gawgaw. Later on, his employees formed Kaisahan ng mga Manggagawa ng La Campana with an authorization from the DOLE to become an affiliate of the larger union. Kaisahan with 66 members presented a demand for higher wages and more privileges to La Campana Starch and Coffee Factory. The demand was not granted and the DOLE certified the issue to the CIR. La Campana filed a motion to dismiss alleging that the action was directed against two different entities with distinct personalities. This was denied, hence this petition.

W/N the jurisdiction case.

CIR over

has the

YES. La Compana Gawgaw and La Campana Factory are operating under one single management or as one business though with two trade names. The coffee factory is a corporation and by legal fiction, an entity separate and apart from the persons composing it namely, Tan Tong and his family. However, the concept of separate corporate personality cannot be extended to a point beyond reason and policy when invoked in support of an end subversive of this policy and will be disregarded by the courts. A subsidiary company which is created merely as an agent for the latter may sometimes be regarded as identical with the parent corporation especially if the stockholders or officers of the two corporations are substantially the same or their systems of operation unified. The facts showed that they had one management, one payroll prepared by the same person, Laborers were interchangeable, there is only one entity as shown by the signboard ad in trucks, packages and delivery forms and the same

place of business. The attempt to make the two factories appear as two separate businesses when in reality they are but one, is but a device to defeat the ends of the law and should not be permitted to prevail. WHY PIERCE? So that La Campana cannot evade the jurisdiction of CIR since La Campana Gawgaw has only 14 employees and only 5 are members of Kaisahan. Yutivo Sons Hardware Co vs CTA1 SCRA 160 (1961)

Yutivo is a domestic corporation engaged in importation and sale of hardware supplies and equipment. After the liberation in 1946, resumed its business and until 1946 bout a number of cars and trucks from General Motors (GM), an American corporation doing business in the Philippines. As importer, GM paid sales tax prescribed by the Tax Code on the basis of its selling price to Yutivo. Yutivo paid no further sales tax on its sales to the public. In June 1946, Southern Motors (SM) organized to engage in the business of selling cars, trucks and spare parts. One of its major subscribers is Yu Tiong Yee, a founder of Yutivo. After the incorporation of SM and until the withdrawal of GM from Phil, the cars and trucks were purchased by Yutivo from GM then sold by Yutivo to Sm and then SM sold these to the public. The same way that GM used to pay taxes on the basis of its sales to Yutivo, Yutivo paid taxes on the basis of its sales to SM. SM paid no taxes on its sales to the public. CIR made an assessment and charged Yutivo 1.8M as deficiency tax plus surcharge. Petitioner contested before CTA. CTA ruled that SM is a mere subsidiary or instrumentality of Yutivo, hence, its separate corporate existence must be disregarded.

WON Yutivo and SM are two separate entities.

Yes. It is an elementary and fundamental principle of corporation law that a corporation is an entity separate and distinct from its stockholders and from other corporation petitions to which it may be connected. However, "when the notion of legal entity is used to defeat public convenience, justify wrong, protect fraud, or defend crime," the law will regard the corporation as an association of persons, or in the case of two corporations merge them into one. Another rule is that, when the corporation is the "mere alter ego or business conduit of a person, it may be disregarded. However, the Court here held that they are inclined to rule that the Court of Tax Appeals was not justified in finding that SM was organized for no other purpose than to defraud the Government of its lawful revenues. In the first place, this corporation was organized in June, 1946 when it could not have caused Yutivo any tax savings. From that date up to June 30, 1947, or a period of more than one year, GM was the importer of the cars and trucks sold to Yutivo, which, in turn resold them to SM. During that period, it is not disputed that GM as importer, was the one solely liable for sales taxes. Neither Yutivo or SM was subject to the sales taxes on their sales of cars and trucks. The sales tax liability of Yutivo did not arise until July 1, 1947 when it became the importer and simply continued its practice of selling to SM. The decision, therefore, of the Tax Court that SM was

organized purposely as a tax evasion device runs counter to the fact that there was no tax to evade. Lidell & Co. vs. Collector, 2 SCRA 632 (1961)

The case is an appeal from the decision of the Court of Tax Appeals imposing a tax deficiencyliability of P1,317,629.61 on Liddell & Co., Inc.The petitioner, Liddell & Co. Inc., (Liddell & Co. for short) is a domestic corporation establish inthe Philippines on February 1, 1946. From 1946 until November 22, 1948 when the purpose clause of the Articles of Incorporation of Liddell & Co. Inc., was amended so as to limit its business activities toimportations of automobiles and trucks, Liddell & Co. was engaged in business as an importer and at thesame time retailer of Oldsmobile and Chevrolet passenger cars and GMC and Chevrolet trucks.On December 20, 1948, the Liddell Motors, Inc. was organized and registered with the Securitiesand Exchange Commission with an authorized capital stock of P100,000 of which P20,000 wassubscribed and paid for as follows: Irene Liddell wife of Frank Liddell 19,996 shares and Messrs. MarcialP. Lichauco, E. K. Bromwell, V. E. del Rosario and Esmenia Silva, 1 share each.Beginning January, 1949, Liddell & Co. stopped retailing cars and trucks; it conveyed theminstead to Liddell Motors, Inc. which in turn sold the vehicles to the public with a steep mark-up. Sincethen, Liddell & Co. paid sales taxes on the basis of its sales to Liddell Motors Inc. considering said sales asits original sales.The Collector of Internal Revenue argued that the Lidell Motors, Inc. was but an alter ego of Liddell & Co. and concluded that for sales tax purposes, those sales made by Liddell Motors, Inc. to thepublic were considered as the original sales of Liddell & Co. hence the imposition of tax deficiency. The CTA upheld the position taken by the Collector. Hence, this petition

Whether or not Lidell Motors, Inc. is an alter ego of Lidell & Co. making it liable for the said tax deficiency?

The Court held that Lidell Motors, Inc. is an alter ego of Lidell& Co. hence making it liable for tax deficiency .Liddell & Co. is wholly owned by Frank Liddell. As of the time of its organization, 98% of the capital stock belonged to Frank Liddell. The 20% paid-up subscription with which the company began its business was paid by him. The subsequent subscriptions to the capital stock were made by him and paid with his own money. As to Liddell Motors, Inc. SC is fully persuaded that Frank Liddell also owned it. He supplied the original capital funds. It is not proven that his wife Irene, ostensibly the sole incorporator of Liddell Motors, Inc. had money of her own to pay for her P20,000 initial subscription. Her income in the United States in the years 1943 and 1944 and the savings therefrom could not be enough to cover the amount of subscription, much less to operate an expensive trade like the retail of motor vehicles. The alleged sale of her property in Oregon might have been true, but the money received therefrom was never shown to have been saved or deposited so as to be still available at the time of the organization of the Liddell Motors, Inc.The evidence at hand also shows that Irene Liddell had scant participation in the affairs of Liddell Motors, Inc. There are quite a series of conspicuous circumstances that militate against the separate and distinct personality of Liddell Motors, Inc. from Liddell & Co.8 SC notice that the bulk of the business of Liddell & Co. was channeled through Liddell Motors, Inc. On the other hand, Liddell Motors, Inc. pursued no activities except to secure cars, trucks, and spare parts from Liddell & Co. Inc. and then sell them to the general public. These sales of vehicles by Liddell & Co. to Liddell Motors, Inc. for the most part were shown to have taken place on the same day that Liddell Motors,

Inc. sold such vehicles to the public. We may even say that the cars and trucks merely touched the hands of Liddell Motors, Inc. as a matter of formality. During the first six months of 1949, Liddell & Co. issued ten (10) checks payable to Frank Liddell which were deposited by Frank Liddell in his personal account with the Philippine National Bank. During this time also, he issued in favor of Liddell Motors, Inc. six (6) checks drawn against his personal account with the same bank. The checks issued by Frank Liddell to the Liddell Motors, Inc. were significantly for the most part issued on the same day when Liddell & Co. Inc. issued the checks for Frank Liddell9 and for the same amounts. It is of course accepted that the mere fact that one or more corporations are owned and controlled by a single stockholder is not of itself sufficient ground for disregarding separate corporate entities. Authorities10 support the rule that it is lawful to obtain a corporation charter, even with a single substantial stockholder, to engage in a specific activity, and such activity may co-exist with other private activities of the stockholder. If the corporation is a substantial one, conducted lawfully and without fraud on another, its separate identity is to be respected. Accordingly, the mere fact that Liddell & Co. and Liddell Motors, Inc. are corporations owned and controlled by Frank Liddell directly or indirectly is not by itself sufficient to justify the disregard of the separate corporate identity of one from the other. There is, however, in this instant case, a peculiar consequence of the organization and activities of Liddell Motors, Inc. Consistently with this view, the United States Supreme Court held that "a taxpayer may gain advantage of doing business thru a corporation if he pleases, but the revenue officers in proper cases, may disregard the separate corporate entity where it serves but as a shield for tax evasion and

treat the person who actually may take the benefits of the transactions as the person accordingly taxable." Thus, to allow a taxpayer to deny tax liability on the ground that the sales were made through another and distinct corporation when it is proved that the latter is virtually owned by the former or that they are practically one and the same is to sanction a circumvention of our tax laws. The decision appealed is modified. Lidell& Co. is declared liable only for the amount of Php 426, 811.67 with 25% surcharge for late payment and 6% interest thereon from the time the judgment becomes final. As it appears that, during the pendency of this litigation, appellant paid under protest the Govt, the total amount assessed by the Collector, the latter is hereby required to return the excess to the petitioner. Ramirez Telephone vs. Bank of America Guatson International vs. NLRC

Petitioners Guatson Travel and Tours, Inc. (hereinafter referred to as Guatson Travel), Philippine Integrated Labor Assistance Corp. (Philac) and Mercury Express International Courier Services, Inc. (MEREX) assail the Decision, rendered by the National Labor Relations Commission entitled “Jolly M. Almoradie vs. Guatson’s Travel Company, Philac and MEREX. In the questioned decision, the NLRC found that Mr. Henry Ocier’s (Vice-President and General Manager of petitioner Guatson Travel) actuation of threatening and forcing private respondent, Jolly M. Almoradie, to resign amounted to illegal dismissal and thus ordered petitioners to pay private respondent backwages. Jolly M. Almoradie was first employed by Mercury Express International Courier Service, Inc. (MEREX) in October, 1983 as Messenger. When it closed its operations, Almoradie was absorbed by MEREX’s sister company Philippine Integrated Labor Assistance Corp. (Philac), likewise as Messenger.

Whether Jolly Almoradie was indeed illegally dismissed by being forced to resign. YES

The petition is hereby DISMISSED for lack of merit. The SC agreed with the finding of NLRC that Jolly’s resignation was not voluntary. When Almoradie was promoted as Sales Representative he had caught the ire of management, so much so that he was issued no less than three memoranda in one day ordering him to answer certain charges. Why he was again promoted to the position of Account Executive after he was reverted back to the rank of a messenger from being a Sales Representative is rather intriguing, unless it was a scheme of management to really rid him from the company. Apparently, Almoradie is not cut out for a sales job, and hence could be dismissed or forced to resign for failing to make good on his job in sales. On the other hand, it would be difficult to dismiss him while being a messenger since he is a permanent employee and

In September, 1986, Almoradie was transferred to Guatson Travel, allegedly also a sister company of MEREX and Philac, as Liaison Officer. Thereafter, he was promoted to the position of Sales Representative. Almoradie was issued three separate memoranda asking him to explain the reason why he didn’t want to sell but he answered that it was not true. He said that he was hampered in his sales promotion and solicitation of customer, due to financial constraint considering that the kind and nature of work entails much expenses for which he shouldered with his personal money. He also stated that he liked her position as a messenger and preferred to be returned to his messenger position. Almoradie was reverted to the position of Messenger, yet sometime in September, 1988, he was again given the position of Account Executive, the nature of work of which is similar to that of a sales representative. Almoradie accepted the transfer with the understanding that he will solely discharge the duties of an account executive and will no longer be required to do messengerial work. On October 1, 1988, Almoradie was allegedly summoned by Henry Ocier to his office and was there and then forced by the latter to resign. Ocier taunted Almoradie with threats that it he will not resign, he will file charges against him which would adversely affect his chances of getting employed in the future. Ocier allegedly even provided the pen and paper on which Almoradie wrote and signed the resignation letter dictated by Ocier himself. Subsequently, Almoradie filed a complaint for Illegal Dismissal. The Labor Arbiter, however Dismissed. Upon Almoradie’s appeal, the NLRC reversed the decision of the Labor Arbiter on his finding that complainant was not forced to resign, anchoring its conclusion to the fact that Almoradie was a permanent employee who has been working for the Ocier’s for five long years; that he was receiving a fairly good salary considering that he is single; that he had no potential employer at the time of his resignation; that there was no evidence to show that Mr. Henry Ocier was indeed not in town on October 1, 1988, when he allegedly forced

there would not be enough basis to make him resign Henry Ocier did not only say that he will file charges against Almoradie and that he has a good lawyer but he even threatened to block his future employment should the latter not file his resignation. This threat is not farfetched. We uphold the NLRC. The three companies are owned by one family, such that majority of the officers of these companies are the same. The companies are located in one building and use the same messengerial service. Moreover, there was no showing that private respondent was paid separation pay when he was absorbed by Philac upon closure of Merex; nor was there evidence that he resigned from Philac when he transferred to Guatson Travel. Under the doctrine of piercing the veil of corporate fiction, when valid ground exists, the legal fiction that a corporation is an entity with a juridical personality separate and distinct from its members or stockholders may be disregarded. Where there is a finding of illegal dismissal, the employee is entitled to both reinstatement and award of backwages from the time the compensation was withheld, in this case in 1988, up to a maximum of three years. The Decision of the NLRC is hereby MODIFIED to the extent that the award of backwages should be computed based on a three-year period, while the separation pay of one month for every year of service should be computed from the time petitioner was employed by Merex and should include the threeyear period as backwages.

Almoradie to resign. Concept Builders vs NLRC GR 108734; 29 May 1996

Petitioner Concept Builders, Inc., a domestic corporation engaged in the construction business. Private respondents were employed by said company as laborers, carpenters and riggers. However, they were illegally dismissed. Aggrieved, private respondents filed a complaint for illegal dismissal. The Labor Arbiter rendered judgment ordering petitioner to reinstate private respondents and to pay them back wages. It became final and executory. The alias Writ of Execution cannot be enforced by the sheriff because all the employees inside petitioner’s premises at 355 Maysan Road, Valenzuela, Metro Manila, claimed that they were employees of Hydro Pipes Philippines, Inc. (HPPI) and not by petitioner. Thus, NLRC issued a break-open order against Concept Builders and HPPI.

Whether the piercing the veil of corporate entity is proper

Yes. It is a fundamental principle of corporation law that a corporation is an entity separate and distinct from its stockholders and from other corporations to which it may be connected. But, this separate and distinct personality of a corporation is merely a fiction created by law for convenience and to promote justice. So, when the notion of separate juridical personality is used to defeat public convenience, justify wrong, protect fraud or defend crime, or is used as a device to defeat the labor laws, this separate personality of the corporation may be disregarded or the veil of corporate fiction pierced. This is true likewise when the corporation is merely an adjunct, a business conduit or an alter ego of another corporation. The conditions under which the juridical entity may be disregarded vary according to the peculiar facts and circumstances of each case. No hard and fast rule can be accurately laid down, but certainly, there are some probative factors of identity that will justify the application of the doctrine of piercing the corporate veil, to wit: 1. Stock ownership by one or common ownership of both corporations. 2. Identity of directors and officers. 3. The manner of keeping corporate books and records. 4. Methods of conducting the business. The SEC en banc explained the “instrumentality rule” which the courts have applied in disregarding the separate juridical personality of corporations as follows: Where one corporation is so organized and controlled and its affairs are conducted so that it is, in fact, a mere instrumentality or adjunct of the other, the fiction of the corporate entity of the “instrumentality” may be disregarded. The control necessary to invoke the rule is not majority or even complete stock control but such domination of

instances, policies and practices that the controlled corporation has, so to speak, no separate mind, will or existence of its own, and is but a conduit for its principal. It must be kept in mind that the control must be shown to have been exercised at the time the acts complained of took place. Moreover, the control and breach of duty must proximately cause the injury or unjust loss for which the complaint is made. The test in determining the applicability of the doctrine of piercing the veil of corporate fiction is as follows: 1. Control, not mere majority or complete stock control, but complete domination, not only of finances but of policy and business practice in respect to the transaction attacked so that the corporate entity as to this transaction had at the time no separate mind, will or existence of its own; 2. Such control must have been used by the defendant to commit fraud or wrong, to perpetuate the violation of a statutory or other positive legal duty or dishonest and unjust act in contravention of plaintiff’s legal rights; and 3. The aforesaid control and breach of duty must proximately cause the injury or unjust loss complained of. The absence of any one of these elements prevents “piercing the corporate veil.” In applying the “instrumentality” or “alter ego” doctrine, the courts are concerned with reality and not form, with how the corporation operated and the individual defendant’s relationship to that operation. Clearly, petitioner ceased its business operations in order to evade the payment to private respondents of back wages and to bar their reinstatement to their former positions. HPPI is obviously a business conduit of petitioner corporation and its emergence was skillfully orchestrated to avoid the financial liability that already attached to petitioner

corporation.

G.R. No. L-28694 May 13, 1981 TELEPHONE ENGINEERING & SERVICE COMPANY, INC., petitioner, vs. WORKMEN'S COMPENSATION COMMISSION, PROVINCIAL SHERIFF OF RIZAL and LEONILA SANTOS GATUS, for herself and in behalf of her minor children, Teresita, Antonina and Reynaldo, all surnamed GATUS, respondents

Piercing the veil of corporate fiction Equity cases Petitioner is a domestic corporation engaged in the business Whether or not TESCO of manufacturing telephone equipment. It has a sister is liable for the death company, the Utilities Management Corporation (UMACOR), claim of the deceased. with offices in the same location. UMACOR is also under the management of Jose Luis Santiago. UMACOR employed the late Pacifica L. Gatus as Purchasing Agent. Then was detailed with petitioner company. He reported back to UMACOR and after 2 years he contracted illness and died of "liver cirrhosis with malignant degeneration." Respondent Leonila S. Gatus, filed a "Notice and Claim for Compensation" with Workmen's Compensation Commission sub-office, alleging therein that her deceased husband was an employee of TESCO, and that he died of liver cirrhosis. On August 9, 1967, and Office wrote petitioner transmitting the Notice and for Compensation, and requiring it to submit an Employer's Report of Accident or Sickness pursuant to Section 37 of the Workmen's Compensation Act (Act No. 3428). An "Employer's Report of Accident or Sickness" was thus submitted with UMACOR indicated as the employer of the deceased. The Report was signed by Jose Luis Santiago. In answer, the employer stated that it would not controvert the claim for compensation, and admitted that the deceased employee contracted illness "in regular occupation." On the basis of this Report, the Acting Referee awarded death benefits plus burial expenses in favor of the heirs of Gatus. TESCO filed its "Motion for Reconsideration and/or Petition to Set Aside Award" alleging as grounds therefor, that the admission made in the "Employer's Report of Accident or Sickness" was due to honest mistake and/or excusable negligence on its part, and that the illness for which compensation is sought is not an occupational disease, hence, not compensable under the law. The Motion for Reconsideration was denied. Meanwhile, the Provincial Sheriff of Rizal levied on and attached the properties of TESCO and scheduled the sale of the same at public auction. Thus petition for "Certiorari with Preliminary Injunction" seeking to annul the award and to enjoin the Sheriff from levying and selling its properties at public

Viewed in the light of these criteria, we note that it is only in this Petition before us that petitioner denied, for the first time, the employer-employee relationship. Although respect for the corporate personality as such, is the general rule, there are exceptions. In appropriate cases, the veil of corporate fiction may be pierced as when the same is made as a shield to confuse the legitimate issues. While, indeed, jurisdiction cannot be conferred by acts or omission of the parties, TESCO'S denial at this stage that it is the employer of the deceased is obviously an afterthought, a devise to defeat the law and evade its obligations. This denial also constitutes a change of theory on appeal which is not allowed in this jurisdiction. Moreover, issues not raised before the Workmen's Compensation Commission cannot be raised for the first time on appeal. For that matter, a factual question may not be raised for the first time on appeal to the Supreme Court. 20 WHEREFORE, this Petition is hereby dismissed.

auction.

[ GR No. L-23586, Mar 20, 1968 ] A. D. SANTOS v. VENTURA VASQUEZ

Respondent Ventura Vasquez was petitioner's taxi driver. Sometime on December 22 or 23, 1961, at about 11:00 a.m., while driving petitioner's taxicab, he vomitted blood. Aside from his hemoptysis, he suffered back pains, fever and headache. He reported to petitioner the fact of his having vomitted blood. He was sent to petitioner company's physician, Dr. Roman, who treated him and sent him to Sto. Tomas Hospital where he was confined for six days. Thereafter, he was admitted at the Quezon Institute. There he stayed until March 19, 1962 under the medical care of Dr. Mario Lirag. Dr. Lirag diagnosed his ailment as pulmonary tuberculosis, moderately advanced in both lungs. Upon his discharge on March 19, 1962, he was clinically improved. His X-ray examination, however, showed the same finding, i.e., PTB, moderately advanced. He has not resumed work. Offshoot of the foregoing is respondent's claim filed on May 9, 1962 with the Workmen's Compensation Commission. The hearing officer affirmed the his claim and order the petitioner to pay all his claims.

(1) Respondent's claim should have been dismissed for his failure to file the notice of injury and claim for compensation required by Section 24 of the Workmen's Compensation Act; and (2) The claim for compensation is directed against Amador Santos, not against petitioner.

1. Sickness manifested itself on December 22 or 23, 1961. Claim was filed on May 9, 1962. Petitioner argues that by Section 24 of the Workmen's Compensation Act, the claims should be thrown out of court. Because, according to petitioner, such claim was not filed within two months following illness. Petitioner's case must fail. Stabilized jurisprudence is that failure of the employer to file with the Commission notice of controversion set forth in the second paragraph of Section 45 of the Workmen's Compensation Act is a waiver of the defense that the claim for compensation was not filed within the statutory period and a forfeiture of the employer's right to controvert the claim. Petitioner here knew of respondent's illness. Yet, it did not controvert respondent's right to compensation. Constructively, such failure is an admission that the claim is compensable. 2. Petitioner's averment that respondent driver had no cause of action against petitioner is equally without merit. Respondent's claim for compensation herein is directed against petitioner A.D. Santos, Inc. Petitioner, in answer to the claim, categorically admitted that claimant was its taxi driver. Add to this the fact that the claimant contracted pulmonary tuberculosis by reason of his said employment. And respondent's cause of action against petitioner is complete. But petitioner cites the fact that respondent driver, in the course of his testimony, mentioned that he worked for the City Cab operated by Amador Santos. This will not detract from the validity of respondent's right to compensation. For, the truth is that really at one time Amador Santos was the sole owner and operator of the City Cab. It was subsequently transferred to petitioner A.D. Santos, Inc. in which Amador Santos was an officer. The mention by respondent of Amador Santos as his employer in the course of his testimony, in the

words of this Court in Sugay vs. Reyes, G.R. No. L20451, December 28, 1964, "should not be allowed to confuse the facts relating to employer-employee relationship," for "when the veil of corporate fiction is made as a shield to perpetrate a fraud and/or confuse legitimate issues (here, the relation of employer-employee), the same should be pierced."

The Court of Appeals found that the Park Rite Co., Inc., a Philippine whether the individual stockholders corporation, was originally organized on or about April 15, 1947, with maybe held liable for a capital stock of 1,500 shares at P1.00 a share. The corporation obligations contracted by leased from Rafael Perez Rosales y Samanillo a vacant lot on Juan the corporation. Luna street (Manila) which it used for parking motor vehicles for a consideration. It turned out that in operating its parking business, the corporation occupied and used not only the Samanillo lot it had leased but also an adjacent lot belonging to the respondents-appellees Padilla, without the owners' knowledge and consent. When the latter discovered the truth around October of 1947, they demanded payment for the use and occupation of the lot. The corporation (then controlled by petitioners Cirilo Parades and Ursula Tolentino, who had purchased and held 1,496 of its 1,500 shares) disclaimed liability, blaming the original incorporators, McConnel, Rodriguez and Cochrane. Whereupon, the lot owners filed against it a complaint for forcible entry in the Municipal Court of Manila on 7 October 1947 (Civil Case No. 4031). RTC- Judgment was rendered in due course on 13 November 1947, ordering the Park Rite Co., Inc. to pay P7,410.00 plus legal interest as damages from April 15, 1947 until return of the lot. Restitution not having been made until 31 January 1948, the entire judgment amounted to P11,732.50. Upon execution, the corporation was found without any assets other than P550.00 deposited in Court. After their application to the judgment credit, there remained a balance of P11,182.50 outstanding and unsatisfied. Now the judgment creditors then filed suit in the Court Manila against the corporation and its stockholders, to recover from them, jointly unsatisfied balance of the judgment, plus costs.

of First Instance of past and present and severally, the legal interest and

RTC- The Court of First Instance denied recovery; CA- Court of Appeals (CA-G.R. No. 8434-R) reversed, finding that the corporation was a mere alter ego or business conduit of the principal stockholders that controlled it for their own benefit, and adjudged them responsible for the amounts demanded by the lot owners, Defendant-appellee RICARDO RODRIGUEZ is hereby ordered to pay to the plaintiffs-appellants Dominga de los Reyes and Sabino Padilla the sum of P1,742.64 with legal interest thereon from the time of the

filing of the complaint and until it is fully paid. In addition thereto the defendants-appellees Cirilo Paredes, Ursula Tolentino and Ricardo Rodriguez shall pay the costs proportionately in both instances.

Related Documents

Corp Of Or Posting
October 2019 10
Syllabus
November 2019 17
Syllabus
October 2019 18
Syllabus
April 2020 1
Syllabus
November 2019 13

More Documents from ""